Bí quyết giải toán Số học THCS theo chủ đề - Huỳnh Kim Linh

525 91 0
Bí quyết giải toán Số học THCS theo chủ đề - Huỳnh Kim Linh

Đang tải... (xem toàn văn)

Tài liệu hạn chế xem trước, để xem đầy đủ mời bạn chọn Tải xuống

Thông tin tài liệu

• Tính chất 9.. Chúng ta vận dụng linh hoạt các tính chất cơ bản này trong nhiều các bài toán về chia hết.. Mặt khác 5 số nguyên liên tiếp luôn có một số chia hết cho 5 nên tích chúng c[r]

(1)

2𝑝 − 1

20192020 ∶ 19

BÍ QUYẾT THEO CHỦ ĐỀ

Giải tốn số học THCS

(2)

HUỲNH KIM LINHNGUYỄN QUỐC BẢO

BÍ QUYẾT Giải tốn số học THCS

THEO CHỦ ĐỀ

(3)(4)

Lêi giíi thiƯu Các em học sinh thầy giáo, cô giáo thân mến !

Cuốn sách Bí giải tốn số học THCS được tác giả biên soạn nhằm giúp em học sinh học tập tốt mơn Tốn THCS THPT sau

Các tác giả cố gắng lựa chọn tập thuộc dạng điển hình, xếp thành hệ thống để bồi dưỡng học sinh giỏi lớp THCS Sách viết theo chủ đềtương ứng với vấn đề quan trọng thường đề thi học sinh giỏi tốn THCS, vào lớp 10 chun mơn toán cảnước Mỗi chủđềđược viết theo cấu trúc lý thuyết cần nhớ, dạng toán thường gặp, tập rèn luyện hướng dẫn giải giúp em học sinh nắm vững kiến thức đồng thời rèn luyện kiến thức học Mỗi chủđề có ba phần:

A Kiến thc cn nh: Phần tóm tắt kiến thức bản, kiên thức bổ sung cần thiết đểlàm sở giải tập thuộc dạng chuyên đề

B Mt s ví d: Phần đưa ví dụ chọn lọc, tiêu biểu chứa đựng kĩ phương pháp luận mà chương trình địi hỏi

Mỗi ví dụthường có: Lời giải kèm theo nhận xét, lưu ý, bình luận phương pháp giải, sai lầm thường mắc nhằm giúp học sinh tích lũy thêm kinh nghiệm giải toán, học toán

C Bài tp vn dng:

Phần này, tác giảđưa hệ thống tập phân loại theo dạng tốn, tăng dần độ khó cho học sinh giỏi Có tập trích từcác đề thi học sinh giỏi Toán đề vào lớp 10 chuyên Toán Các em cố gắng tự giải Nếu gặp khó khăn có thểxem hướng dẫn lời giải cuối sách

Các tác giả hi vong sách tài liệu có ích giúp em học sinh nâng cao trình độvà lực giải tốn, góp phần đào tạo, bồi dưỡng học sinh giỏi cấp THCS Mặc dù có nhiều cố gắng biên soạn song sách khó tránh khỏi sai sót Chúng tơi mong nhận ý kiến đóng góp bạn đọc

Trong q trình soạn sách xin chân thành cảm ơn Thầy Trần Thanh Trà - Trường THCS Chu Văn An, quận Ngô Quyền, tỉnh Hải Phòng; Thầy Lưu Lý Tưởng - Trường THCS Văn Lang, TP Việt Trì, Phú Thọ; Thầy Phạm Văn Vượng - Trường THCS Nhữ Bá Sỹ, tỉnh Thanh Hóa, Cơ Quế Thị Lan Trường THCS Diễn Mỹ, Diễn Châu, Nghệ An tặng nhiều tài liệu đềthi quý để tác giả kham khảo

(5)(6)

CH

UY

ÊN

Đ

S

H

C

A.KiÕn thøc cÇn nhí I Ước bội

1) Định nghĩa ước bội

Ước: Số tự nhiên d ≠0được gọi ước số tự nhiên a a chia hết cho d Ta

nói d là ước a

Nhận xét: Tập hợp ước a Ư( ) {a = dN d a: | }

Bội: Số tự nhiên m được gọi bội a≠0 m chia hết cho a hay a

ước số m

Nhận xét: Tập hợp bội a(a≠0)là B a( ) {= 0; ; ; ;a a ka},kZ

2) Tính chất:

- Số bội số nguyên khác Số ước số nguyên - Các số -1 ước số nguyên

- NếuƯ( ) { }a = 1;a a số nguyên tố

- Số lượng ước số : Nếu dạng phân tích thừa số nguyên tố số tự nhiên A a b cx y z … số lượng ước A (x+1)(y+1)(z+1) …

Thật ước A số có dạng mnp…trong đó:

mx+1 cách chọn (là

1, , , ,a aax) ny+1 cách chọn (là1, , , ,b b2 … by)

pz+1 cách chọn (là1, , , ,c c2 … cz),…

Do đó, số lượng ước A (x+1)(y+1)(z+1) II Ướcchung vàbộichung

1) Định nghĩa

Ước chung (ƯC): Nếu hai tập hợp Ư(a) Ư(b) có phần tử chung phần tửđó gọi ước số chung a và b Kí hiệu ƯC(a; b)

CH

Đ

1 CÁC BÀI TOÁN V

(7)

CH

IN

H

P

H

C

K

T

H

I H

C S

IN

H

GI

I C

ẤP

H

AI

Nhận xét: Nếu ƯC(a b; ) { }= a b nguyên tố

Ước chung lớn (ƯCLN): Số dNđược gọi ước số chung lớn a b (a b; ∈Z) d phần tử lớn tập hợp ƯC(a; b) Kí hiệu ước chung lớn a b ƯCLN(a; b) (a;b) gcd(a;b)

Bội chung (BC): Nếu hai tập hợp B(a) B(b) có phần tử chung phần tử gọi bội số chung a b Kí hiệu BC(a; b)

Bội chung nhỏnhất (BCNN): Số m≠0được gọi bội chung nhỏ a b m

là số nhỏ khác tập hợp BC(a; b) Kí hiệu bội chung nhỏ a b BCNN(a; b) [ ]a b; lcm(a;b)

2)Cách tìm ƯCLN BCNN

a) Muốn tìn ƯCLN hai hay nhiều số lớn ,ta thực bước sau : Phân tích số thừa số nguyên tố

2.- Chọn thừa số nguyên tố chung

3.- Lập tích thừa sốđã chọn, thừa số lấy với sốmũ nhỏ Tích ƯCLN phải tìm

Ví dụ:

30=2.3.5, 20=2 5⇒ƯCLN(30; 20) =2.5=10

Chú ý :

- Nếu sốđã cho khơng có thừa số ngun tốchung ƯCLN chúng - Hai hay nhiều số có ƯCLN gọi số nguyên tố

- Trong số cho, số nhỏ ước số cịn lại ƯCLN số cho số nhỏ

b) Muốn tìm BCNN hai hay nhiều số lớn , ta thực ba bước sau : 1- Phân tích số thừa số nguyên tố

2- Chọn thừa số nguyên tố chung riêng

3- Lập tích thừa sốđã chọn , thừa số lấy với sốmũ lớn chúng Tích BCNN phải tìm

Ví dụ:

30=2.3.5, 20=2 5⇒BCNN(30; 20) =2 3.52 =60

Chú ý:

- Nếu sốđã cho đôi nguyên tố BCNN chúng tích số Ví dụ: BCNN(5 ; ; 8) = = 280

- Trong sốđã cho, số lớn bội số cịn lại BCNN sốđã cho số lớn Ví dụ: BCNN(12 ; 16 ; 48) = 48

3) Tính chất

(8)

CH

UY

ÊN

Đ

S

H

C

● Nếu (a a1; 2; ;an)=1thì ta nói số a a1; 2; ;an nguyên tố

● Nếu (am;ak)= ∀ ≠1, m k m k,{ } {, ∈ 1; 2; ;n} ta nói số a a1; 2; ;an đơi nguyên tố

c∈ƯC (a; b) thì a b; ( )a b;

c c c

  =     ● d ( )a b; a b;

d d   = ⇔ =

  ● (ca cb; ) ( )=c a b;

● ( )a b; =1 ( )a c; =1thì(a bc; )=1 ● (a b c; ; ) ( )=( a b c; ; )

● Cho a> >b

- Nếu a=b q ( )a b; =b

- Nếu a=bq+r r( ≠0) ( ) ( )a b; = b r;

Một số tính chất bội chung nhỏ nhất: ● Nếu [ ]a b; =M M M;

a b

  =

 

 

● [a b c; ; ] [ ]=  a b c; ; 

● [ka kb, ]=k a b[ ], ;

● [ ]a b; (a b; )=a b

4) Thuật tốn Euclid việc tính nhanh ƯCLN BCNN

“Thut toán Euclid” thuật toán cổ biết đến, từ thời Hy Lạp cổđại, sau Euclid (ơ –clit) hệ thống phát triển nên thuật

toán mang tên ông Về số học, “Thuật toán Euclid” thuật toán để xác định ước số chung lớn (GCD – Greatest Common Divisor) phần tử thuộc vùng Euclid (ví dụ: số ngun) Khi có ƯCLN ta tính nhanh BCNN Thuật tốn khơng u cầu việc phân tích thành thừa số số nguyên

Thuật tốn Oclitdùng đểtìm ƯCLN sốnguyên Đểtìm ƯCLN hai số nguyên a b ta dùng cách chia liên tiếp hay gọi “vòng lặp” sau:

Bước 1: Lấy a chia cho b:

Nếu a chia hết cho b ƯCLN(a,b) = b

(9)

CH

IN

H

P

H

C

K

T

H

I H

C S

IN

H

GI

I C

ẤP

H

AI

Bước 2:Lấyb chia cho số dư r: Nếu b chia hết cho r ƯCLN(a, b) = r

Nếu b chiar dư r1 (r1 ≠0) làm tiếp bước

Bước 3:Lấy r chia cho số dư r1: Nếu r chia cho r1 dư ƯCLN(a, b) = r1 Nếu r chia r1 dư r2 (r1 ≠0) làm tiếp bước

Bước 4: Lấy r1 chia cho số dư r2 : Nếu r1 chia hết cho r2 ƯCLN(a, b) = r2 Nếu r1 cho cho r2 dư r3 (r3 ≠0 ) làm tiếp

như đến sốdư

Sdư cuối khác dãy chia liên tiếp như ƯCLN (a,b)

Ví dụ: Tính ước số chung lớn 91 287

• Trước hết lấy 287 (số lớn số) chia cho 91: 287 = 91.3 + 14 (91 14 sẽđược dùng cho vòng lặp kế) Theo thuật tốn Euclid, ta có ƯCLN(91,287) = ƯCLN(91,14)

Suy tốn trởthành tìm ƯCLN(91,14) Lặp lại quy trình phép chia khơng cịn sốdư sau:

91 = 14.6 + 7(14 sẽđược dùng cho vòng lặp kế)

14 = 7.2 (khơng cịn sốdư suy kết thúc, nhận 7 làm kết quả) Thật vậy: = ƯCLN(14,7) = ƯCLN(91,14) = ƯCLN(287,91) Cuối ƯCLN(287,91) =7

Tính BCNN nhanh nhất

Để việc giải tốn BCNN ƯCLN nhanh, Nếu biết áp dụng “Thut toán Euclid” : Biết rằng: hai số nguyên a, b có BCNN [ a,b] ƯCLN (a,b)

[ ] ( ) [ ] ( ) ( ) [ ]

, , , , ,

, ,

a b a b

a b a b a b a b a b

a b a b

= ⇒ = =

Nghĩa là: Tích số nguyên a b = ƯCLN (a,b) x BCNN (a,b)

Ví dụ: có a = 12; b = 18 suy ƯCLN (12,18) = thì: BCNN (12,18) = (12 x 18) : = 36

a b

b r1 q

1

r r2 q1

r q2 ……

n

r − (a, b)

(10)

CH

UY

ÊN

Đ

S

H

C

Nếu làm theo cách phân tich thừa số nguyên tố phải tính: 12 = 22 x 3; 18 = x 32 suy BCNN (12,18) = 22 x 32 = 36

Nhận xét: Với cặp số ngun có nhiều chữ số việc phân tích thừa số nguyên tố nhiều thời gian; lấy tích số bấm máy tính cầm tay nhanh dễhơn 5) Phân số tối giản

a

b phân số tối giải (a b, )=1 Tính chất:

i) Mọi phân sốkhác có thểđưa phân số tối giản ii) Dạng tối giản phân số

iii)Tổng (hiệu) số nguyên phân số tối giản phân số tối giản

B. CÁC DẠNG TOÁN THƯỜNG GẶP

Dạng 1: Các toán liên quan tớisố ước số

* Cơ sởphương pháp:Nếu dạng phân tích thừa số nguyên tố số tự nhiên A a b cx y z … số lượng ước A (x+1)(y+1)(z+1) …

Thật ước A số có dạng mnp…trong đó: mx+1 cách chọn (là

1, , , ,a aax) ny+1 cách chọn (là1, , , ,b b2 … by)

pz+1 cách chọn (là1, , , ,c c2 … cz),…

Do đó, số lượng ước A (x+1)(y+1)(z+1) * Ví dụminh họa:

Bài tốn 1. Tìm sốước số 96

18

Hướng dẫn giải

Ta có : 96 ( )2 96 192 96

18 = =3

Vậy số ước số 96

18 (96 192 1+ )( + =) 97.193 18721.=

(11)

CH

IN

H

P

H

C

K

T

H

I H

C S

IN

H

GI

I C

ẤP

H

AI

Hướng dẫn giải

Giả sử

k a a a

k

n= p p p với pi nguyên tố aiN*

n số phương a a1, 2, ,aklà số chẵn (a1 +1)(a2 +1 ) (ak +1) số lẻ

Mặt khác (a1+1)(a2 +1 ) (ak +1) số số ước n, tốn chứng

minh

Bài toán 3. Một số tự nhiên n tổng bình phương số tự nhiên liên tiếp Chứng minh rằng n khơng thểcó 17 ước số

Hướng dẫn giải

Tổng bình phương số tự nhiên liên tiếp có dạng :

( )2 ( )2

1

n= m− +m + m+ = m + khơng thể số phương

Nếu n có 17 ước số n là số phương (bài tốn 1), vơ lí Từ suy điều phải chứng minh

Dạng 2: Tìm số nguyênn để thỏa mãn điều kiện chia hết

* Cơ sởphương pháp: Tách số bị chia thành phần chứa ẩn số chia hết cho số chia phần nguyên dư, sau để thỏa mãn chia hết số chia phải ước phần sốngun dư, từ ta tìm số nguyên n thỏa mãn điều kiện

* Ví dụminh họa:

Bài tốn 1. Tìm số tựnhiên n để(5n + 14) chia hết cho (n + 2)

Hướng dẫn giải Ta có 5n + 14 = 5.(n + 2) +

Mà 5.(n + 2) chia hết cho (n + 2)

Do (5n + 14) chia hết cho (n +2) ⇔4 chia hết cho (n + 2) ⇔(n + 2) ước ⇔(n +2) ∈{1;2;4}

⇒ n ∈{0;2}

Vậy với n ∈{0; 2}thì (5n + 14) chia hết cho (n + 2)

Bài tốn 2. Tìm số tựnhiên n để

3 15

+ + n

n số tự nhiên

Hướng dẫn giải Để

3 15 + +

n n

(12)

CH

UY

ÊN

Đ

S

H

C

⇒ [(n + 15) - (n + 3)] chia hết cho (n + 3) ⇔12 chia hết cho (n +3)

⇔(n + 3) Ư(12) = {1; 2; 3; 4; 6; 12} ⇔n ∈{0; 1; 3; 9}

Vậy với n ∈{0; 1; 3; 9}thì

3 15 + +

n n

số tự nhiên

Bài tốn 3. Tìm số tựnhiên n để n2+ 3n +  n +

Hướng dẫn giải

Ta có: n2+ 3n +  n +

Suy ra: n (n + 3) +  n + ⇔  n + => n + ∈ Ư(6) = {1; 2; 3; 6} => n = 0; n = Bài tốn 4. Tìm sốngun n để phân số 4n

2n

+

− có giá trị số nguyên

Hướng dẫn giải

Ta có: 4n

2n

+ − =

4n n(2n 1) 7

n

2n 2n 2n

− + = − + = +

− − −

Vì n nguyên nên để 4n

2n

+

− nguyên

7

2n 1− nguyên

=> 2n – ∈ Ư(7) = {–7; –1; 1; 7}

⇔ 2n ∈{– 6; 0; 2; 8} ⇔ n ∈ {– 3; 0; 1; 4} Vậy với n ∈ {– 3; 0; 1; 4} 4n

2n

+

− có giá trị số nguyên

Bài tốn 5. Tìm số tựnhiên n để biểu thức sau số tự nhiên:

2 17

2 2

n n n

B

n n n

+ +

= + −

+ + +

Hướng dẫn giải

Ta có:

2 17 2 17 19

2 2 2

n n n n n n n

B

n n n n n

+ + + + + − +

= + − = =

+ + + + +

4( 2) 11 11

2

n

n n

+ +

= = +

+ + Để B số tự nhiên 11

2

(13)

CH

IN

H

P

H

C

K

T

H

I H

C S

IN

H

GI

I C

ẤP

H

AI

Do n + > nên n + = 11 ⇒n = Vậy n = B ∈ N

Bài tốn 6 Tìm k nguyên dương lớn để ta có số ( )

1 23

k n

k

+ =

+ sốnguyên dương

Hướng dẫn giải

Ta có: ( ) ( )( )

2 2

1 23 21 484 484

1 ,

23 23 23 23

k k k k k

n k k Z

k k k k

+

+ + + + − +

= = = = − + ∈

+ + + + n

sốnguyên dương k+23 | 484, k+23>23

Ta có 484 = 222 = 4.121= 44.21 23 121 98 23 44 21

k k

k k

+ = =

 

⇒ ⇒

+ =  =

 Với k = 98, ta có n = 81

Với k = 21, ta có n = 11

Vậy giá trị k lớn thỏa mãn yêu cầu tốn 98

Dạng 3: Tìm số biết ƯCLN chúng * Cơ sởphương pháp:

* Nếu biết ƯCLN(a, b) = K a = K.m b = K.n với ƯCLN(m; n) = (là điều kiện số m, n cần tìm) , từđó tìm a b

* Ví dụminh họa:

Bài tốn Tìm hai số tự nhiên a, b, biết rằng: a + b = 162 ƯCLN(a, b) = 18

Hướng dẫn giải Giả sử ab

Ta có: a+ =b 162, (a b, )=18

Đặt 18

18

a m

b n

=  

=

 với (m, n)=1,mn

Từ a+ =b 162⇒18(m+n)=162⇒ + =m n

Do ( m, n ) = 1, lập bảng:

m

n

a 18 36 loai 72

(14)

CH

UY

ÊN

Đ

S

H

C

Kết luận: Các số cần tìm là: (18;144 ; 36;126 ; 72;90) ( ) ( )

Bài tốn Tìm hai số nhỏhơn 200, biết hiệu chúng 90 ƯCLN 15

Hướng dẫn giải Gọi hai số cần tìm a, b (a b, ∈N a b; , <200)

Ta có: a− =b 90; (a b, )=15

Đặt ( )

( )

( )

,

15 ,

15 90

15

m n

a m m n

m n

b n m n

 =

=  =

 ⇒ ⇒

  

− =

= − =

  

  

Lại có: , 200 15 200 13

15 200 13

m m

a b

n n

< ≤

 

 

< ⇒ ⇒

< ≤

 

 

m n a b

13 195 105

11 65 75

7 85 15

Vậy: (a b, ) (= 195;105 , 65;75 , 85;15 ) ( ) ( )

Bài toán Tìm hai số tự nhiên có tích 432 ƯCLN

Hướng dẫn giải

Ta có: ab=432;(a b, )=6(ab)

Đặt a=6 ,m b=6n với (m, n) = m n ⇒36mn=432⇒mn=12

Ta được:

m n a b

1 12 72

3 18 24

Vậy (a, b) (= 6;72 , 18, 24) ( )

Bài tốn Tìm hai số a, b biết 7a=11b ƯCLN(a; b)= 45

Hướng dẫn giải

(15)

CH

IN

H

P

H

C

K

T

H

I H

C S

IN

H

GI

I C

ẤP

H

AI

TừƯCLN(a; b) = 45 ( ) ( )

1 1

1

45

; 1,

45

a a

a b a b

b b

= 

⇒ = = ≥

Mà: 1

1

11

11 11

7

7

a a

a

b

b b

= 

= ⇒ = ⇒ 

=

 (a b1; 1)=1=>

45.11 495 45.7 315

a b

= =

 = =

 Vậy hai số a,b cần tìm a = 495 b = 315

Dạng 4: Các toán phối hợp BCNN số với ƯCLN chúng * Cơ sởphương pháp:

* Nếu biết BCNN (a, b) = K ta gọi ƯCLN(a; b) = d a = m.d b = n.d với ƯCLN(m; n) = (là điều kiện số m, n cần tìm) , từđó tìm a b

* Ví dụminh họa:

Bài tốn Cho a=1980,b=2100 a) Tìm ( )a b, [ ]a b,

b) So sánh [ ]a b, ( )a b, với ab Chứng minh nhận xét hai số tự nhiên ab khác 0tùy ý

( Nâng cao phát triển lớp tập – Vũ Hữu Bình)

Hướng dẫn giải

a) 2 2

1980=2 5.11, 2100=2 3.5 ƯCLN(1980, 2100)

2 3.5 60

= =

( ) 2

1980, 2100 7.11 69300

BCNN = =

b) [1980, 2100 1980, 2100] ( )=1980.2100( 4158000) Ta chứng minh

rằng [ ]a b, ( )a b, =a b

Cách Trong cách giải này, thừa số riêng coi thừa số

chung, chẳng hạn a chứa thừa số 11,bkhông chứa thừa số 11 coi bchứa

thừa số11 với sốmũ Với cách viết này, ví dụ ta có:

2

1980=2 5.7 11

2

2100=2 3.5 7.11

(1980, 2100)là tích thừa số chung với số mũ nhỏ 2 0

2 5.7 11 =60 [1980, 2100] tích thừa số chung với sốmũ lớn 2 7.112 2 =69300.

(16)

CH

UY

ÊN

Đ

S

H

C

[ ]a b, ( )a b, =a b ( )1

Khi phân tích thừa số nguyên tố, thừa số nguyên tố hai vế ( )1

chính thừa số nguyên tố có ab Ta chứng tỏ hai vế chứa

thừa số nguyên tốnhư với sốmũ tương ứng

Gọi plà thừa số nguyên tố tùy ý thừa số nguyên tố Giả sử số mũ ptrong a x,số mũ p bytrong x ycó thể

Khơng tính tổng qt, giả sử xy Khi vế phải (1) chứa p với số mũ x+y Còn vế trái, [a, b] chứa p với số mũ x, (a, b) p với số mũ y nên vế trái chứa p với số mũ x+y

Cách Gọi d =( , )a b a=da b', =db′ (1), ( ', ')a b =1

Đặt ab m

d = ( )2 , ta cần chứng minh [ ]a b, =m

Để chứng minh điều này, cần chứng tỏ tồn số tự nhiên x, y cho

m=ax, m=by (x, y) =

Thật từ (1) (2) suy '

.b

m a ab

d

= = ,

'

.a

m b ba

d

= = Do đó, ta chọn ' '

, ,

x=b y=a ( )x y, =1 ( ' ')

,

a b = Vậy ab [ ]a b, ,

d = tức [ ]a b, ( )a b, =ab

Bài tốn Tìm hai số tự nhiên biết ƯCLN chúng 10, BCNN chúng 900

Hướng dẫn giải

Gọi số phải tìm a b, giả sử ab Ta có ( , )a b =10 nên a=10a', b = 10b', ' '

( , )a b =1,a′≤b' Do ab=100 ' 'a b (1) Mặt khác ab=[ ]a b, ( , )a b =900.10=9000 (2) Từ (1) (2) suy a b' '=90 Ta có trường hợp :

'

a

'

b 90 45 18 10

Suy ra:

a 10 20 50 90

(17)

CH IN H P H Ụ C K Ỳ T H I H Ọ C S IN H GI Ỏ I C ẤP H AI

Bài tốn Tìm hai số tự nhiên a, b cho tổng ƯCLN BCNN 15

Hướng dẫn giải Giả sử a < b

Gọi d =ƯCLN( a; b) ( ) ( )

1 1

1

, ;

a d a

a b a b b d b

= 

⇒ = < =

 , d < 15

Nên BCNN(a; b) = a b d1 .1

Theo ta có: d+a b d1 =15=>d(1+a b1 1)=15=> ∈d U( ) {15 = 1;3;5;15}, Mà d < 15, Nên

TH1 :

1

1

1

1 14

14 14

a a

d a b

b b

= ⇒ = 

= ⇒ = ⇒ 

= ⇒ =

1 2 7 a a b b = ⇒ =   = ⇒ = 

TH2 :

1

1

1

3

4 12

a a

d a b

b b = ⇒ =  = ⇒ = ⇒  = ⇒ = 

TH3 :

1

1

1

5

2 10

a a

d a b

b b

= ⇒ = 

= ⇒ = => 

= ⇒ = 

Vậy cặp số(a; b) cần tìm : (1;14), (2; 7), (3; 12), ( 5; 10) đảo ngược lại

Dạng 5: Các toán liên quan đến hai số nguyên tố nhau

* Cơ sở phương pháp: Để chứng minh hai số nguyên tố nhau, ta chứng minh chúng có ƯCLN =

* Ví dụminh họa:

Bài tốn 1.Chứng minh rằng:

a) Hai số tự nhiên liên tiếp (khác 0) hai số nguyên tố b) Hai số lẻ liên tiếp hai số nguyên tố

c) 2n + 3n + (nN ) hai số nguyên tố

Hướng dẫn giải

a) Gọi d ∈ ƯC (n , n + 1) ⇒(n+ −1) n d ⇒1d ⇒ =d Vậy n n + hai số nguyên tố

cùng

b) Gọi d ∈ ƯC (2n + 1, 2n + 3) ⇒(2n+ −3) (2n+1)d ⇒2d ⇒ ∈d { }1; Nhưng d ≠2vì d ước số lẻ Vậy d =

Vậy (2n + 1) (2n + 3) hai số nguyên tố

c) Gọi d ∈ ƯC (2n + 1,3n + 1) ⇒3(2n+ −1) 2(3n+1)d⇒1d ⇒ =d

(18)

CH

UY

ÊN

Đ

S

H

C

Bài toán Cho a b hai số nguyên tố Chứng minh sốsau hai số nguyên tố nhau:

a) a a + b b) a2 a + b c) ab a + b Hướng dẫn giải

a) Gọi d∈ƯC(a, a + b) ⇒(a+b)−a d ⇒b d Ta lại có: a d ⇒ ∈d ƯC(a, b), d =

(vì a b hai số nguyên tố nhau) Vậy (a, a + b) =

b) Giả sử a2 a + b chia hết cho số nguyên tố d a chia hết cho d, b chia hết cho d Như a b chia hết cho số nguyên tố d, trái với giả thiết (a, b) =

Vậy a2và a + b hai số nguyên tố

c) Giả sử ab a + b chia hết cho số nguyên tố d Tồn hai thừa số a b, chẳng hạn a, chia hết cho d, b chia hết cho d, trái với (a, b) = Vậy (ab, a + b) =

Bài tốn Tìm số tự nhiên n để số: 9n +24 3n + số nguyên tố nhau?

Hướng dẫn giải Giả sử 9n + 24 3n + chia hết cho số nguyên tố d

Ta có (9n+24) (−3 3n+4)d ⇒12d ⇒ ∈d { }2;3 Điều kiện để (9n + 24, 3n + 4) =

2,

dd ≠ Ta dễ thấy d ≠3 3n + khơng chia hết cho Muốn d ≠2

trong hai số 9n + 24 3n +4 không chia hết cho Ta thấy 9n+ 24 số lẻ suy n lẻ, 3n + lẻ suy n lẻ

Vậy để (9n + 24, 3n + 4) = n phải số lẻ

Bài tốn Tìm n để18n + 31n +7 hai số nguyên tố

Hướng dẫn giải Gọi ƯCLN( 18n + ; 21n + 7)= d, d ∈N*

Khi ta có : ( )

( ) ( ) ( )

7 18 18

126 42 126 21 21

21 21

n d

n d

n n d d

n d n d

+ 

+

 ⇒ ⇒ + − + ⇒

 + 

+

 

 

 

 

( ) {21 1; 3; 7; 21}

d U

(19)

CH IN H P H Ụ C K Ỳ T H I H Ọ C S IN H GI Ỏ I C ẤP H AI

Do 21n + 7d, Mà 21n + không chia hết cho 3, nên d = d =7 Để hai số18n + 21n + hai số nguyen tốthì d khác hay 18n + 3/ ⇒7 18n + -2 1/7 ⇒ 18n - 18/7 ⇒ 18( n - 1)/7 ⇒ n - 1/7 ⇒ n - 1≠7k ⇒ n ≠7k +

Vậy n ≠7k + với k số tựnhiên 18n + 21n + hai số nguyên tố

Dạng 6: Các toán phân số tối giản

* Cơ sởphương pháp: Một phân số tối giản tử số mẫu số có ước chung lớn

* Ví dụminhhọa:

Bài toán Chứng minh

3

n n

+

+ phân số tối giản với số tự nhiên n

Hướng dẫn giải Gọi d ước chung (2n + 3) (3n + 4) Suy ra:

( )

( ) ( ) ( )

3

2

3 3

2

3

n d

n d

n n d d d

n d n d +  +  ⇒ ⇒ + − + ⇒ ⇒ ∈  +  +          Ư(1)

Mà Ư(1) = −{ }1;1 ⇒ ∈ −d { }1;1 Vậy

3

n n

+

+ phân số tối giản

Bài toán Chứng minh 21

14

n n

+

+ phân số tối giản với số tự nhiên n

Hướng dẫn giải Cách 1: Gọi (2n + 4, 14n + 3) = d ( )

( ) ( )

21

7 14 3

14

n d n n n d +  ⇒ ⇒ + ⇒ + +      Từ(1) (3) suy 1d ⇒ =d

Vậy 21

14

n n

+

+ phân số tối giản với số tự nhiên n

Cách 2: Giả sử phân số 21

14

n n

+

+ chưa tối giản

Suy 21n + 14n + có ước số chung nguyên tố d (21 4) (14 3)

14

n n n d

n d

⇒ + − + = +

⇒ +

(20)

CH

UY

ÊN

Đ

S

H

C

Do đó: (14n+ −3) (14n+ =1) 1d,vơ lý

Vậy toán chứng minh

Bài toán Chứng minh 22

3

n

n n

+

+ + phân số tối giản với số tự nhiên n

Hướng dẫn giải Ta viết lại:

( )( )

2 3

1

3

n n

n n

n n

+ = +

+ +

+ +

Do n + n + hai số tự nhiên liên tiếp nên nguyên tố ⇒(n+1,n+2)=1

Suy tổng chúng (n + 1) + (n + 2) = 2n + tích chúng ( )( )

1

n+ n+ =n + n+ nguyên tố

Vậy phân số 22 ,

3

n

n N

n n

+

+ + phân số tối giản

Bài toán Định n để

2

n n

+

− phân số tối giản với n số tự nhiên

Hướng dẫn giải Để

2

n n

+

− phân số tối giản (n + 8, 2n – 5) =

Giả sử d ước nguyên tố 2n – n + Suy ra: | n ( )( )1

| n

d d

+ 

 −

 Từ (1) (2) suy ra: d| 2(n+8) (= 2n− +5) 21 ( )3

Do d | 21⇒ =d 3,

Muốn cho phân số tối giản điều kiện cần đủ (n + 8) không chia hết cho Do đó: n≠3k+1,n≠7m−1với k m, ∈N

Vậy n≠3k+1và n≠7m−1là điều kiện cần tìm để phân số

2

n n

+

− tối giản

Dạng 7: Tìm ƯCLN biểu thức số

* Ví dụminh họa:

Bài tốn Tìm ƯCLN 2n−1 9n+4(n∈). Hướng dẫn giải

(21)

CH

IN

H

P

H

C

K

T

H

I H

C S

IN

H

GI

I C

ẤP

H

AI

Vì 2n−1 17 ⇒2n−18 17 ⇔2(n 9) 17−  ⇔ −n 17 ⇔ =n 17k+9 với kN Nếu n=17k + 2n - 117 9n + = 9(17k + 9)+ = Bội 17 + 8517 (2n - 1,9n + 4) = 17

Nếu n≠17k+9 2n - khơng chia hết cho 17 (2n - 1,9n + 4) =

Bài tốn Tìm ƯCLN ( 1)

2

n n+

2n+1(n∈*). Hướng dẫn giải Gọi d∈ƯC ( 1), 2 1

2

n n

n

 + 

+

 

  n n( +1)d 2n+1d

Suy n(2n+ −1) (n n+1)d tức n d2 .

Từ n n( +1)d n d2 suy n d Ta lại có 2n+1d, 1dnên d =1

Vậy ƯCLN ( 1)

2

n n+

và 2n +

Dạng 8: Liên hệ phép chia có dư với phép chia hết, ƯCLN, BCNN

* Cơ sởphương pháp:

* Nếu số tự nhiên a chia cho số tự nhiên bđược sốdư ka – k b

* Nếu a ⋮ b a ⋮ c mà ƯCLN(a, b) = ⇒ a chia hết cho tích b.c (a, b, c ∈ N) * Nếu a ⋮ b a ⋮ c mà a số nhỏ ⇒ a = BCNN(a, b) (a, b, c ∈ N) * Nếu a ⋮ b m ⋮ b mà b lớn ⇒ b = Ư CLN(a, m) (a, b, m ∈ N)

* Ví dụminh họa:

Bài toán Bạn Nam nghĩ số có chữa số, bớt sốđó số 7, bớt số số8, bớt sốđó 10 số 9, Hỏi bạn Nam nghĩ sốnào?

Hướng dẫn giải Gọi x số bạn Nam nghĩ, Điều kiện: 99< <x 1000

Theo ta có:

8 7

9 8 7;8;9 (7;8;9) 10 9

x x

x x x x BC

x x

− −

 

 − ⇒ − ⇒ − ⇒ − ∈

 

 −  −

 

 

  

 

{ } { }

1 0;504;1008; 1;505;1009;

x− ∈ ⇒ ∈x , Mà 99 < x < 1000 nên x = 505 Vậy số có ba chữ số mà bạn Nam nghĩ 505

(22)

CH

UY

ÊN

Đ

S

H

C

Hướng dẫn giải

Theo ta có: ( )

3 2

5 , , 10 (3;5; 7)

7 14

a m a m a

a n m n p N a n a a BC

a p a p a

= + = + −

  

 = + ∈ ⇒ = + ⇒ − ⇒ − ∈

  

 = +  = +  −

  

  

Vì a nhỏ nên 2a - nhỏ khác hay 2a - = BCNN( 3; 5; 7) = 105 ⇒2a = 106 ⇒a = 53

Vậy số tự nhiên nhỏ cần tìm 53

Bài tốn Tìm số tự nhiên nhỏ chia cho 5, 7, có sốdư theo thứ tựlà 3, 4,

Hướng dẫn giải Gọi số tự nhiên cần tìm a Theo ta có:

( )

5 10

7 , , 14 (9;5; 7)

9 18 10

a m a m a

a n m n p N a n a a BC

a p a p a

= + = + −

  

 = + ∈ ⇒ = + ⇒ − ⇒ − ∈

  

 = +  = +  −

  

  

Vì a nhỏ nên 2a - nhỏ khác hay 2a - = BCNN( 9; 5; 7) = 315 ⇒2a = 316 ⇒a = 158

Vậy số tự nhiên nhỏ cần tìm 158

Bài tốn Linh Mai mua số hộp bút chì màu, số bút đựng hộp lớn Kết quảLinh có 15 bút chì màu Mai có 18 bút chì màu hỏi hộp có bút?

Hướng dẫn giải

Gọi số bút hộp a Điều kiện: aN a, <15 a >1

Theo ta có : 15 a 18 a, Nên a ước chung 15 18 Và a phải lớn nhỏhơn 15 ⇒ kết quảđược a =

Bài toán Hai lớp 6A 6B tham gia phong trào tết trồng cây, em tròng số nhau, kết lớp 6A trồng 132 vag 6B 135 Hỏi lớp có học sinh

Hướng dẫn giải

Gọi số em trồng a, Điều kiện: aN a, <132,a>1

Theo ta có: 132a 135a ta thấy a UC∈ (132;135)={ }1;3

Vậy a = 3, Khi lớp 6A có 132: = 44 học sinh lớp 6B có 135: = 45 học sinh

(23)

CH IN H P H Ụ C K Ỳ T H I H Ọ C S IN H GI Ỏ I C ẤP H AI

kết bạn tham gia phân công đứng thành hàng dọc cho hàng có số bạn thi mơn nhau.Hỏi phân học sinh đứng thành hàng?

Hướng dẫn giải

Gọi số học sinh đứng hàng a Điều kiện : aN a, <72và a >

Vì hàng có số học sinh mơn nên ta có: 96 a ;120 a 72 a ,

Để có hàng số học sinh phải lớn hay a lớn

Hay a = ƯCLN ( 96; 120; 72) = 24, Vậy số hàng cần tìm : (96 + 120 + 72): 24 = 12 hàng

Dạng 9: Tìm ƯCLN hai số thuật tốn Ơ-clit

* Cơ sởphương pháp:

a) Trường hợp b a| (a, b) = b

b) Trường hợp b a| giả sửa = bq + c (a, b) = (b, c)

Thuật toán Euclid Giả sử:

1

1 2

1 2 3

2 1

1 , , , ,

n n n n n n

n n n

a bq r r b

b r q r r r

r r q r r r

r r q r r r

r r q

− − − −

= + < < = + < <

= + < < = + < < =

Thuật toán Euclid phải kết thức với sốdư

1

n

r+ ≠

Theo b) ta có

( ) ( ) (a b, = b r, = r r1, 2)= = (rn−1,rn)=rn

Vậy ƯCLN(a, b) sốdư cuối khác thuật tốn Euclid

* Ví dụminh họa:

Bài toán 1. Dùng thuật toán Euclid để chứng minh : ( )

3 1,

n + n + n + n =

Hướng dẫn giải

Ta có ( )

3

n + n + = n + n n+n +

( )

3

2

2

1 1

n n n n

n n n

n n

+ = + + + = + = +

Vậy ( )

3 1,

n + n + n + n =

a b

b r1 q

1

r r2 q1

r q2 ……

n

r − (a, b)

(24)

CH

UY

ÊN

Đ

S

H

C

Bài toán Cho hai số tự nhiên a b a( >b)

a) Chứng minh a chia hết cho b ( , )a b =b

b) Chứng minh a không chia hết cho b ƯCLN hai số ƯCLN

của số nhỏ sốdư phép chia số lớn cho số nhỏ c) Dùng nhận xét để tìm ƯCLN(72, 56)

(Nâng cao phát triển lớp tập 1) Hướng dẫn giải

a) Mọi ước chung a b hiển nhiên ước b Đảo lại, a chia hết cho b

nên b ước chung a b Vậy ( , )a b =b

b) Gọi r số dư phép chia a cho b a( >b) Ta có a=bk+r k( ∈N), cần chứng

mình ( , )a b =( , ).b r

Thật vậy, a b chia hết cho d r chia hết cho d, ước chung

a b ước chung b r (1) Đảo lại b r chia hết cho d a chia hết cho d, ước chung b r ước chung a b (2) Từ (1)

(2) suy tập hợp ước chung a b tập hợp ước chung b r Do hai số lớn hai tập hợp nhau, tức ( , )a b =( , ).b r

c) 72chia 56 dư 16 nên (72, 56)=(56,16) ;

56chia 16 dư nên (56,16)=(16,8) ;

16chia hết (16,8)=8 Vậy (72, 56)=8

Nhận xét : Giảsử a không chia hết cho b a chia cho br1, b chia cho r1 dư

2,

r r1 chia cho r2dư r3, ,rn−2chia cho rn−1dư r rn, n−1chia cho rn dư 0( dãy số b r r, , , 1 2 rn

dãy số tự nhiên giảm dần nên số phép chia hữu hạn q trìnhtrên kết thức với số dư 0) Theo chứng minh ví dụ ta có

( ) ( ) (a b, = b r, = r r1, 2)= (rn−1,rn)=rnrn−1 chia hết cho rn

Như UCLN a b( , ) số chia cuối dãy phép chia liên tiếp a cho b, b cho r r1, cho r2, , r r1, , số dư phép chia theo thứ tự

Trong thực hành ngườita đặt tính sau :

Việc thực dãy phép chia liên tiếp gọi thuật toán Ơ clit 72 56

56 16

16

(25)

CH

IN

H

P

H

C

K

T

H

I H

C S

IN

H

GI

I C

ẤP

H

AI

Trường hợp tìm ƯCLN ba số, ta tìm ƯCLN hai số tìm UCLN kết với số thứ ba

Bài toán Tìm ƯCLN( a,b) biết a số gồm 1991 chữ số2; b số gồm chữ số

Hướng dẫn giải

Ta có: 1991 chia dư 7, cịn chia dư Theo thuật tốn Ơ- Clít:

(a, b) =   =   =  =

1991 sè sè sè sè sè

( 22 ,22 2) (22 2,22 2) (22 2,2) 2.

Bài tốn Tìm ƯCLN

a) 

2004 sè

11 1 11111111 b) 123456789 987654321

(Chuyên đề bồi dưỡng học sinh giỏi toánTHCS phần số học- Nguyễn Vũ Thanh)

Hướng dẫn giải

a) Gọi  

2004

;

11 11

a= b= Ta có 2000 8 nên    

2000 8

2000

11 11 111 11

b

=

  

 

Do  ( ) ( ) ( )

2000

11 0000 1111 1111 , ,1111 1111 1111 so

a= + =bq+ ⇒ a b = b = do b

b) Gọi a = 987654321; b = 123456789 Ta có:

( ) ( ) ( )

8 , b , 9

a= b+ ⇒ a = b = dob

C BÀI TẬP ÁP DỤNG

Câu Tìm sốchia thương phép chia có số bị chia 145, sốdư 12 biết thương khác (sốchia thương số tự nhiên)

Câu Hãy viết số108 dạng tổng số tự nhiên liên tiếp lớn Câu 3. Tìm số tựnhiên n để3n + chia hết cho n –

Câu 4. Tìm a ∈Nđể a + bội a –

Câu 5. Tìm số tựnhiên cho 4n - chia hết cho 2n –

Câu Tìm sốnguyên n để:

5+n −2n chia hết cho n−2 Câu Tìm số nguyên nđể:

4

n + chia hết cho n+2 Câu8 Tím tất sốnguyên n để phân số

2

n n

+

(26)

CH

UY

ÊN

Đ

S

H

C

Câu Tìm số tự nhiên có ba chữ số, biết tăng gấp n lần cộng chữ số với n ( n số tự nhiên, gồm nhiều chữ số)

Câu 10 Tìm số tự nhiên a biết 264 chia cho a dư 24, 363 chia cho a dư 43

Câu 11 Tìm số tự nhiên a biết 398 chia cho a dư 38 , cịn 450 chia cho a dư 18

Câu 12 Có 100 90 bút chì thưởng cho số học sinh, cịn lại vởvà 18 bút chì khơng đủchia Tính số học sinh thưởng

Câu 13 Phần thưởng cho học sinh lớp học gồm 128 vở, 48 bút chì, 192 nhãn Có thể chia nhiều thành phần thưởng nhau, phần thưởng gồm vở, bút chì, nhãn vở?

Câu 14 Tìm số tự nhiên a nhỏ cho a chia cho 3, cho 5, cho số dư theo thứ tự 2, 3, 4

Câu 15 Một thi chạy tiếp sức theo vòng tròn gồm nhiều chặng Biết chu vi đường tròn là330m , chặng dài75m , địa điểm xuất phát kết thúc chỗ

Hỏi thi có chặng?

Câu 16. Tìm số tự nhiên có ba chữ số, cho chia cho 17, cho 25 sốdư theo

thứ tự 8 16

Câu 17. Tìm số tư nhiên n lớn có ba chữ số, cho n chia cho 8 dư 7, chia cho 31 dư 28.

Câu 18 Nếu xếp số sách vào túi1 0 vừa hết, vào túi 12

thừa cuốn, vào túi 18 thừa 8 biết số sách khoảng từ 715 đến 1000. Tính sốsách đó?

Câu 19 Hai lớp , 6A Bcùng thu nhặt số giấy vụn Trong lớp 6A,một bạn

thu 25kg, cịn lại bạn thu 10kg Tính số học sinh lớp, biết số giấy

lớp thu khoảng từ 200kg đến 300kg

Câu 20 Có hai đồng hồ(có kim kim phút) Trong ngày, thứ chạy nhanh phút, thứ hai chạy chậm phút Cảhai đồng hồđược lấy lại xác Hỏi sau bao lâu, cảhai đồng hồ lại chạy xác?

Câu 21.Tìm hai số tự nhiên biết rằng:

a) Hiệu chúng 84, ƯCLN 28, số khoảng từ 300 đến 440 b) Hiệu chúng 48, ƯCLN 12

Câu 22 Tìm hai số tự nhiên biết ƯCLN chúng 36 tổng chúng 432

Câu 23 Tìm hai số tự nhiên biết tích chúng 864 ƯCLN Câu 24 Chứng minh 14n + 21n + (n ∈N )là hai số nguyên tố

Câu 25. Chứng minh 2n + 6n + hai số nguyên tố

(27)

CH

IN

H

P

H

C

K

T

H

I H

C S

IN

H

GI

I C

ẤP

H

AI

Câu 27 Cho a b hai số nguyên tố Chứng minh số sau hai số nguyên tố nhau:

a) b a b− (a>b);

b) 2

a +b ab

Câu 28 Chứng minh số c nguyên tố với a với b c nguyên tố với tích ab

Câu 29 Tìm số tự nhiên n cho: a) 4n−5 chia hết cho 13;

b) 5n+1chia hết cho 7;

c) 25n+3 chia hết cho 53

Câu 30 Tìm số tự nhiên n để số sau nguyên tố nhau: a) 4n+3 2n+3;

b) 7n+13 2n+4;

c) 9n+24 3n+4;

d) 18n+3 21n+7.

Câu31 Chứng minh có vơ số số tự nhiên n để n+15 n+72 hai số nguyên tố

cùng

Câu 32 Cho ( )a b, =1 Tìm :

( )

) ,

a a b a b+ − b) (7a+9 ,3b a+8b)

Câu 33 Tìm a, b biết: a) [ ]a b, +( )a b, =55; b) [ ]a b, −( )a b, =5; c) [ ]a b, +( )a b, =35

Câu 34.Tìm ƯCLN số sau thuật toán Ơ-clit: a) (187231,165148 ;)

b)  

100 chu so chu so (11 ,11 1). 

Câu 35 Tìm [n n; +1;n+2]

Câu 36 Tìm *

n∈ biết n< 30 để số 3n+4 5n+1 có ước chung lớn

Câu 37 Tìm số nguyên nđể phânsố

2

n n

+

+ có giá trị số nguyên

(28)

CH

UY

ÊN

Đ

S

H

C

bến sau 56 phút lại sau phút Xe thứ ba quay bến sau 48 phút sau phút lại Hỏi ba xe lại xuất phát từ bến lần thứ hai vào lúc giờ?

Câu 39 Chứng minh với sốnguyên dương n phân số

6 + +

n

n tối giản

Câu 40 Cho phân số: ( )

+

= ∈

+ 

n

P n

n

a) Chứng tỏ phân số P phân số tối giản

b) Với giá trị n phân số P có giá trị lớn nhất?

Câu 41 Tìm hai sốnguyên dương biết a + 2b = 48 ƯCLN(a; b) +3.BCNN(a; b)= 114 Câu 42.Cho (a, b) = 1, tìm (11a + 2b, 18a + 5b)

Câu 43 Chứng minh (a, b) = (5a + 3b, 13a + 8b)

Câu 44 Cho ba số tự nhiên a, b, c nguyên tốcùng đôi Chứng minh (ab + bc + ca, abc) =

Câu 45. Tìm tất các số tự nhiên a, b nguyên tố biết rằng:

2

a b

a ab b 73

+ =

− +

Câu 46. Cho m, n∈N, 1≤m<n Chứng minh rằng: (22n +1, 22n + =1)

Câu 47. Cho 1≤m, n∈N Tìm ( m n ) −1, −1

Câu 48 Tìm hai số tự nhiên a b, biết: ƯCLN ( , )a b =15 vàBCNN a b( , )=300;

Câu 49 Cho a∈Z , tìm (a, a+2)

Câu 50. Cho a, m số nguyên lớn Chứng minh :

( ) ( )

1 m , ,

a a aa m a

+ + + + − = −

Câu 51 Chứng minh a, b, c số lẻ , , ( , , )

2 2

a b b c c a

a b c

+ + +

  =

 

 

Câu 52 Tổng số tự nhiên a a1, 2, ,a49bằng 999 Hỏi ước số chung lớn chúng nhận giá trị lớn ?

Câu 53 Cho (a, b) = 1, tìm (11a + 2b, 18a + 5b) Câu 54 Cho (m, n) = Tìm ( 2)

,

m+n m +n

Câu 55 Chứng minh phân số sau tối giản với nZ

( )

21

) n ; ) n

(29)

CH

IN

H

P

H

C

K

T

H

I H

C S

IN

H

GI

I C

ẤP

H

AI

Câu 56 Tìm số nguyên nđể phân số sau tối giản

18 3

) ; )

21 7

n n

a b

n n

+ +

+ +

Câu 57.Tìm hai số tự nhiên a, b thỏa mãn a+ =b 128 ( )a b, =16 Câu 58.Tìm ƯCLN ab+ba 33 với a + b không chia hết cho

Câu 59 Chứng minh số tự nhiên có ba chữ số tận 136 có ước sốdương

Câu 60 Chứng minh knlm =1 (ma+nb ka, +lb) ( )= a b,

Câu 61 Tìm ƯCLN tất số có chữ sốđược viết chữ số 1, 2, 3, …, sốđó chữ sốđều khác

Câu 62 Cho (a, b) = tìm ƯCLN 2a + b a(a + b)

Câu 63 Chứng minh phân số sau tối giản với n số nguyên

2

12 15

) ; )

30 30 21 13

n n n

a b

n n n

+ + +

+ + +

Câu 64 Tìm sốnguyên n để phân số 13

2

n n

+

− tối giản

Câu 65. Chứng minh

5n +1 6

n

3

n

tối giản

Câu 66. Tìm số tự nhiên nhỏ để phân số sau tối giản : , , , 31

9 10 33

n+ n+ n+

Câu 67 Tìm số tự nhiên a, b biết ab=360,[ ]a b, =60

Câu 68 Tìm số tự nhiên nhỏ chia cho 2, 3, 4, 5, 6, 7, 8, có sốdư 1, 2, 3, 4, 5, 6, 7,

Câu 69.Tìm tất cặp số ( )a b; nguyên dương thỏa mãn hai điều kiện:

i) a b, khác 1và ước số chung lớn a b,

ii) Số N =ab ab( +1 2)( ab+1) có 16 ước số nguyên dương

Câu 70 Xác định số nguyên tố p q, cho p2 − pq+2q2 2p2 + pq+q2 số

nguyên tố

Câu 71.Tìm tất số tự nhiên khác 0: a b, cho: ( )a b, =1 2 2

25

a b a b

+ =

+

(Thi học sinh giỏi lớp TP Hồ Chí Minh năm 1992 – 1993) Câu 72 Cho m, n hai số nguyên tố Tìm ước chung lớn m n +

(30)

CH

UY

ÊN

Đ

S

H

C

Câu 73.Tìm tất cặp số nguyên dương a, b thỏa mãn 4a 1+ 4b 1− nguyên tố nhau, đồng thời a b+ ước 16ab 1+

Câu 74.Tìm tất cặp số ( )a b; nguyên dương thỏa mãn hai điều kiện:

i) a b, khác 1và ước số chung lớn a b,

ii) Số N =ab ab( +1 2)( ab+1) có 16 ước số nguyên dương

(Trích đề học sinh giỏi toán Đăk Lăknăm học 2017-2018) Câu 75 Cho hai số tự nhiên m n thoảmãn

m n n

m+1+ +1

số nguyên Chứng minh ước chung lớn m n không lớn m+n

(Trích đề học sinh giỏi lớp Hải Dươngnăm học 2004-2005) Câu 76.Cho ba sốnguyên dương a b c, , đôi khác đồng thời thỏa mãn điều kiện:

i) a ước b c bc+ + ,

ii) b ước a+ +c ac,

iii) c ước a b ab+ + ,

a) Hãy ba số (a b c, , )thỏa mãn điều kiện

b) Chứng minh a b c, , không thểđồng thời số nguyên tố

(31)

CH

IN

H

P

H

C

K

T

H

I H

C S

IN

H

GI

I C

P H

AI

A. KiÕn thøc cÇn nhí

1 Định nghĩa phép chia.

Cho hai số nguyên a b b ≠ ta ln tìm hai số nguyên q r cho a bq r , với = + 0≤ ≤r b −1 Trong a số bị chia, b số chia, q thương,

r số dư

Khi a chia cho b số dư r∈{0;1; 2; ;b −1}

• Nếu r 0= a bq , ta nói a chia hết cho b hay b chia hết a Ký hiệu: = a b hay

b a

Vậy a chia hết cho b tồn số nguyên q cho a bq = • Nếu r 0≠ , ta nói a chia b có số dư r

2 Một số tính chất cần nhớ

• Tính chất Mọi số ngun khác ln chia hết cho

• Tính chất Nếu a bb ca c

• Tính chất Nếu a bb aa = ±b

• Tính chất Nếu a.b m  (b,m)=1 a m  • Tính chất Nếu a mb m (a±b m)

• Tính chất Nếu a m , a n (m n, )=1 a mn

• Tính chất Nếu a bc dac bd

• Tính chất Trong n số nguyên liên tiếp tồn số nguyên chia hết cho n

• Tính chất Nếu a b 0− ≠ với a, b số tự nhiên (anbn)(ab) (nN)

• Tính chất 10 Nếu a b 0+ ≠ với a, b, n số tự nhiên n số lẻ (an +bn)(a+b) 3.Một số dấu hiệu chia hết

Đặt A a a a a a , với = n n 1− 2 0 a ;a ; ;a ;a ;an n 1− chữ số Khi ta có dấu hiệu chia

hết sau:

CH

Đ

2 QUAN HỆ CHIA HẾT

(32)

CH

UY

ÊN

Đ

S

H

C

{ }

0

2 0; 2; 4; 6;8

A a a

•  ⇔  ⇔ ∈

( 1 )

3 n n

A a a aa

•  ⇔ + + + + 

1

4

A a a

•  ⇔ 

{ }

0

5 0;5

A a a

•  ⇔  ⇔ ∈

2

8

A a a a

•  ⇔ 

( 1 )

9 n n

A a a aa

•  ⇔ + + + + 

( ) ( )

11 11

A a a a a

•  ⇔ + + − + + 

1

25 25

A a a

•  ⇔ 

2

125 125

A a a a

•  ⇔ 

B CÁC DẠNG TOÁN THƯỜNG GẶP

Dạng 1: Sử dụng tính chất n số ngun liên tiếp có số chia hết

cho n (n ≥ 1)

* Cơ sởphương pháp: Sử dụng tính chất như: tích hai số nguyên liên tiếp chia hết cho 2, tích ba số nguyên liên tiếp chia hết cho chia hết cho Chúng ta vận dụng linh hoạt tính chất nhiều toán chia hết

Bài toán 1. Chứng minh rằng:

a) Tích số nguyên liên tiếp chia hết cho b) Tích số chẵn liên tiếp chia hết cho c) Tích số nguyên liên tiếp chia hết cho 120

Hướng dẫn giải

a) Trong số nguyên liên tiếp có số chia hết cho số chia hết tích số nguyên liên tiếp chia hết cho (do (2, 3) = 1)

b)Hai số chẵn liên tiếp có dạng 2n (2n + 2) với n Z∈

Do tích hai số ngun liên tiếp có dạng 4n(n + 1) Do n n + hai số nguyên liên tiếp nên n n 2( + )

Vì 4n n 8( + )

c)Ta có 120 = 3.5.8

(33)

CH

IN

H

P

H

C

K

T

H

I H

C S

IN

H

GI

I C

P H

AI

5 số nguyên liên tiếp có số chẵn liên tiếp nên theo ý b) ta có tích số ngun liên tiếp chia hết cho

Mặt khác số nguyên liên tiếp ln có số chia hết tích chúng chia hết cho

Vậy tích số nguyên liên tiếp chia hết cho 120

Chú ý: Tổng qt ta có tích n số tự nhiên liên tiếp chia hết cho n!

Bài tốn 2.Chứng minh tích số chẵn liên tiếp chia hết cho 48 Hướng dẫn giải

Ba số chẵn liên tiếp có dạng 2n, (2n + 2) (2n + 4) với n Z∈

Do tích hai số ngun liên tiếp có dạng 8n(n + 1)(n + 2)

Do n, (n + 1) (n + 2) số nguyên liên tiếp nên n n n 6( + )( + )

Vì n n n 2( + )( + )=6m m Z( ∈ )

Do tích số chẵn liên tiếp 8n n n 2( + )( + )=48m 48 Vậy toán chứng minh

Bài toán 3. Chứng minh với số nguyên n n n3−

chia hết cho

Hướng dẫn giải

Ta có:

( ) ( ) ( )

3− = − =1 −1 +1 n n n n n n n

Biểu thức tích số nguyên liên tiếp nên số chia hết cho 2, số chia hết cho mà (2, 3) = nên (n n3− )6

Bài toán 4. Chứng minh với số nguyên lẻ n

1

− − +

n n n chia hết cho 128

Hướng dẫn giải

Ta có:

( ) ( ) ( )( ) ( ) (2 )

6− 4− 2+ =1 − −1 2− =1 −1 4− =1 2−1 +1

n n n n n n n n n n

n số lẻ nên đặt n = 2k + 1 (k N∈ ) Ta có:

( 2 )2 ( )2 ( 2 )2 ( )

1  1 4 4 

− = + −  = + = + 

 

n k k k k k

(34)

CH

UY

ÊN

Đ

S

H

C

Mặt khác: n2 + =1 2( k+1)2 + =1 4k2+4k+ =2 2( k2+2k+1 2)

Do n n n6− 4− 2+ =1 (n2−1) (2 n2+1 128)

(đpcm)

Chú ý: Bình phương số lẻ số lẻ

Dạng 2: Phân tích thành nhân tử

* Cở sở phương pháp: Để chứng minh A(x) chia hết cho p ta phân thích A x( )=D x p( ) ,

cịn khơng thể đưa phân tích ta viết p=k q

Nếu ( )k q, =1 ta chứng minh A(x) chia hết cho k q

Nếu ( )k q, ≠1 ta viết A(x) = B(x).C(x) rồi chứng minh B(x) chia hết cho k C(x) chia hết

cho q

* Ví dụminh họa:

Bài tốn 1. Cho a, b, c số nguyên khác thỏa mãn điều kiện:

 

+ + = + +

 

 

2

2 2

1 1 1 1

a b c a b c

Chứng minh rằng: a3+b c chia hết cho 3+

(Đề thi HSG lớp TP Thanh Hóa 2016-2017)

Hướng dẫn giải

Từ giả thiết  + +  = + + ⇔  + + =

   

2

2 2

1 1 1 2 1 0

a b c a b c ab bc ca

a b c

abc

+ +

⇔ =

Vì a, b, c ≠0 nên a + b + c =

( ) ( )

⇒ + = −

⇒ + = −

⇒ + + + = −

⇒ + + =

3

3 3

3 3

a b c

a b c

a b 3ab(a b) c

a b c 3abc

Vậy a3+b c 33+ 3 với a, b, c ∈Z

Bài toán 2. Cho A=1.2.3 29, B=30.31.32 58

Chứng minh A + B chia hết cho 59

(35)

CH

IN

H

P

H

C

K

T

H

I H

C S

IN

H

GI

I C

P H

AI

Ta có:

(59 29)(59 28 59)( 27 59 1) ( ) 59 1.2.3 29 59 ( ) 59 59

= − − − − = − = − ∈ ⇒ + = 

B k k A k Z A B k

Vậy A + B chia hết cho 59

Bài toán 3. Cho số nguyên dương x, y, z Chứng minh rằng: ( ) (5 ) (5 )5

− + − + −

x y y z z x chia hết cho 5(x y y z z x− )( − )( − ) Hướng dẫn giải

Đặt a x y b y z= − , = − ⇒ − = − +z x (a b)

Do ta cần chứng minh: a b5+ 5− +(a b)5chia hết cho −5ab a b( + )

Ta có: a b5+ − +5 (a b)5 = −(5a b4 +10a b3 2+10a b2 3+5ab4)

( 3 2)

5 2

= − ab a b+ + a b+ ab

( )( ) ( )

( )( )

2

2

5

5

 

= −  + − + + + 

= − + + +

ab a b a ab b ab a b ab a b a ab b

Do tốn chứng minh

Bài tốn 4. Chứng minh với ba số tự nhiên a,b,c có số lẻ hai số

chẵn ta ln có ( ) (3 ) (3 ) (3 )3

c b a a c b c b a c b

a+ + − + − − + − − − + Chia hết cho 96

(Trích đề thi HSG lớp tỉnh Phú Thọ 2015)

Hướng dẫn giải

Đặt a+ − =b c z b; + − =c a x a; + − =c b y x+ + = + +y z a b c

Ta có (x y z+ + )3−x3−y z3− =3(x y)(y z)(x z) 3.2c.2a.2 b 24abc+ + + = =

Do số a, b, c có số chẵn nên abc chia hết cho 24abc chia hết cho 24.4 = 96

Vậy toán chứng minh

Dạng 3: Sử dụng phương pháp tách tổng

* Cởsởphương pháp: Để chứng minh A(x) chia hết cho p ta biết đổi A(x) thành tổng

số hạng chứng minh số hạng chia hết cho p

* Ví dụminh họa:

Bài tốn 1. Chứng minh m, n số nguyên ta có:

(36)

CH

UY

ÊN

Đ

S

H

C

Hướng dẫn giải

a) Ta có: n n( 2+11)=n3+11n n n= 3− +12n n=( −1) (n n+ +1 12) n

Dễ chứng minh: (n−1) (n n+1 6, 12 6) n (n Z∈ )

Do đó: n n( 2+11 6)

b) Ta có: ( 2− 2)= ( 2− −1) ( 2−1)= ( 2− −1) ( −1)

 

mn m n mn m n mn m mn n

Do: mn m( 2− =1) n m( −1) (m m+1 6,) mn n( − =1) m n( −1) (n n+1 6)

Do đó: mn m n( 2− 2)6

c) Ta có: n n( +1 2)( n+ =1) (n n+1)(n+ + − =2 n 1) (n n+1)(n+ +2) (n−1) (n n+1) Do: n n( +1)(n+2 6,) (n−1) (n n+1 6)

Do đó: n n( +1 2)( n+1 6)

Chú ý: Tách tổng phương pháp chứng minh chia hết mà lời giải dễ hiểu, ngắn gọn đẹp mắt nên thường trình bày tốn giải nhiều phương pháp, nhiên để áp dụng em cần linh hoạt việc tách

Ví dụ: câu a) ta thấy 12n chia hết ta tách riêng phần cịn lại phân đưa dạng tích, dựa vào tính chất chia hết tích số tự nhiên dễ dàng chứng chia

Câu b) nghĩ việc thêm bớt để tạo tổng hai tích số tự nhiên liên tiếp Tương tự câu c) dễ dàng tách 2n + = (n – 1) + (n + 2) để đưa tổng hai tích số tự nhiên tiếp

Bài tốn 2. Chứng minh rằng: n n5có chữ số tận giống với n số tự nhiên Hướng dẫn giải

Để chứng minh n n5có chữ số tận giống ta chứng minh (n n5− )10

Thật vậy: ( ) ( )( ) ( ) ( )

1 1  5

− = − = − + = −  − + 

n n n n n n n n n n

( )( ) ( ) ( )( ) ( )( ) ( ) ( )

1 1 1

− − + − = − − + + + − +

n n n n n n n n n n n n n

Nhận xét: (n−2)(n−1) (n n+1)(n+2)là tích năm số tự nhiên liên tiếp nên chia hết cho chia hết cho 10

Mặt khác (n−1) (n n+1)là tích số tự nhiên liên tiếp nên chia hết

( ) ( )

(37)

CH

IN

H

P

H

C

K

T

H

I H

C S

IN

H

GI

I C

P H

AI

Do (n n5− )10 tốn chứng minh.

Bài toán 3.a) Chứng minh + 15+

n n n số nguyên với n Z

b) Chứng minh

12 24+ +

n n n số nguyên với n số nguyên chẵn

Hướng dẫn giải

a) Ta có:

15 = −15 = − −5

n n n n n n

Do đó: 5

5 15 5

− −

+ + = + + − − = + +

n n n n n n n n n n n n n

Từ thí dụ ta dễ dàng chứng minh được: (n n5− ) (5, n n3− )3 tốn được

chứng minh

b) Do n số nguyên chẵn nên n = 2m (với m Z∈ )

Do đó: 3 3 ( 2)( 1)

12 24 6

+ +

+ +

+ + = + + = = m m m

n n n m m m m m m

Theo ý c) thí dụ ta có n n( +1 2)( n+1 6) tốn chứng minh

Bài toán 4. Chứng minh ax bx c Z x Z2+ + ∈ ∀ ∈, ,a a b+ ,c Z∈ Hướng dẫn giải

Ta có: 2 ( ) 2 ( 1) ( ) .

2

+ + = − + + + = x x + + +

ax bx c ax ax a b x c a a b x c

Dễ thấy: ( 1)

− ∈

x x

Z x (x – 1) hai số nguyên liên tiếp Do đó: ax bx c Z x Z2+ + ∈ ∀ ∈, ,a a b+ ,c Z∈

Bài toán 5. Cho số nguyên a ;a ; ;a1 2 n Đặt A a a a= 1+ 2+ + n = 3+ 3+ +

1 n

B a a a Chứng minh A chia hết cho B chia hết cho

Hướng dẫn giải

Trước hết ta chứng minh bổ đề: Với số ngun a ta ln có a a 63− 

Thật vậy, ta có a a3 − =(a a a 1− ) ( + ).

(38)

CH

UY

ÊN

Đ

S

H

C

Xét hiệu sau

( ) ( ) ( ) ( ) ( )

− = 3+ + + − + + + = 3− + − + + −

1 n n 1 2 n n

B A a a a a a a a a a a a a

Áp dụng bổ để ta ( 3− ) ( 3− ) ( − )

1 2 n n

a a 6; a a 6; ; a a

Do ta B A Suy A chia hết cho B chia hết cho − 

Dạng 4: Sử dụng đẳng thức

Cởsởphương pháp: Nếu a, b số nguyên thì:

a bnn chia hết cho a – b với n số tự nhiên a b

a bnn chia hết cho a + b với n số tự nhiên chẵn a≠ −b a bn+ n chia hết cho a + b với n số tự nhiên lẻ a≠ −b

(a b+ )n =ka b+ n với k số nguyên, n số tự nhiên

(a+1)n=ac+1 (a−1)n =ac+ −( )1 n , n số tự nhiên

* Ví dụminh họa:

Bài toán 1.Với n số tự nhiên chẵn.Chứng minh rằng:

a) 22 55

22 +55 b) 20n+16n−3 323.n− 

Hướng dẫn giải

a) Ta có: 22 55 ( )22 ( )55 ( )22 ( )55

22 55 21 56 7

= + = + + − = + + −

P BS BS

= BS + + BS – = BS nên 22 55

22 +55 chia dư

b) Ta có: 323 17.19= Ta biến đổi 20n +16n−3 1n− =(20 1n− +) (16n−3n) Ta có: (20n−1 : 20 1) ( − ⇒) (20n−1 19)

Mặt khác n số chẵn nên (16n−3n)(16 3+ )⇒(16n−3 19n)

Do (20n− +1) (16n−3 19n) ⇒(20n+16n −3 19 1n− ) ( )

Ta biến đổi 20n+16n−3 1n− =(20n −3n) (+ 16n−1n)

Ta có: (20n−3 : 20 3n) ( − )⇒(20 17n− )

Mặt khác n số chẵn nên (16n−1n)(16 1+ ⇒) (16n−3 17 2n) ( )

(39)

CH

IN

H

P

H

C

K

T

H

I H

C S

IN

H

GI

I C

P H

AI

Bài toán 2. Chứng minh với số tự nhiên n ta có:

n 2n n n 2n 2n n

a) 11 + +12 + 133 b) + +26.5 +8 + 59 c) 7.5 +12.6 19 Hướng dẫn giải

a)Ta có: 11n 2+ +122n 1+ =11 11 12.122 n + 2n =121.11 12.144n + n

(133 12 11 12.144) n n 133.11 12 144n ( n 11n)

= − + = + −

Do 133.11 133n 12 144( n−11n)(144 11 hay 12 144− ) ( n−11 133n)

Nên 133.11 12 144n+ ( n −11n)⇒11n 2+ +122n 1+ 133 (đpcm)

b)Ta có:

n n 2n n n 2n n n

5 + +26.5 +8 + =25.5 +26.5 +8.8 =51.5 +8.64 (59 8.64) n n 59.5 64n ( n 5n)

= − + = + −

Vì (64n−5n)(64 5− ⇒) (64n−5 59n)

Nên 59.5n+8 64( n−5 59n) ⇒5n 2+ +26.5n +82n 1+ 59(đpcm)

c) Ta có: 7.52n +12.6n =7.25n+(19 6− ) n =19.6 25 6n+ ( n− n)

Vì (25 6n− n)(25 6− ⇒) 7 25 19( n− n)

Nên 19.6n+7 25( n−6 19n) ⇒57.52n+12.6 19n (đpcm)

Bài toán 3. Chứng minh A 1993= 1997+1997199330 Hướng dẫn giải

Sử dụng tính chất (a b+ )n=ka b+ nvới k số nguyên, n số tự nhiên.

Ta có:

( ) ( )

( )

( )

1997 1993 1997 1993

1997 1993 1993

1993

A 1993 1997 1980 13 2010 13

1980c 13 2010d 13 1980c 2010d 13 13 30 66c 67d 952.13 30

= + = + + −

= + + −

= + +

= + + 

Bài toán 4. Chứng minh C 5= n( n+ −1 3) (n n+2 91 n N n) ( ∈ )

(Chuyên sư phạm Hà Nội 1997 – 1998)

(40)

CH

UY

ÊN

Đ

S

H

C

Sử dụng tính chất (a b+ )n=ka b+ n, (a+1)n=ac+1, (a−1)n =ac+ −( )1 nvới k số nguyên, n số tự nhiên

Ta có:

( ) ( ) ( )

( )

25 18 12

21 14

21 14

7

= + − −

= + + − + − +

= + + − − − −

= − − 

n n n n

n n n n

n n n n

C

c d e

c d e

Mặt khác:

( ) ( ) ( )

( ) ( )

( )

26 13 13

26 13 13

13 13

= − + − + − −

= + − + − − − − −

= − − 

n n n n

n n n n

C

f g h

f g h

Vì (13, 7) = nên C 7.13 91  =

Bài toán 5. Chứng minh rằng: 3 3

1 100

= + + + +

A chia hết choB = + + + +1 100

Hướng dẫn giải

Ta có B = (1 + 100) + (2 + 99) + …+ (50 + 51) = 101.50

Để chứng minh A chia hết cho B ta chứng minh A chia hết cho 50 101 Ta có:

( ) ( ) ( )

( )( ) ( )( ) ( )( )

( ) ( )

3 3 3

2 2 2

2 2 2

1 100 99 50 51

1 100 100 100 99 2.99 99 50 51 50 50.51 51

101 100 100 2.99 99 50 50.51 51 101

= + + + + + +

= + + + + + + + + + + + +

= + + + + + + + + + 

A

Ta lại có: ( 3) ( 3) ( 3)

1 99 98 50 100

= + + + + + +

A

Mỗi số hạng chia hết cho 50 nên A chia hết cho 50 (2)

Từ (1) (2) suy A chia hết cho 101 50 nên A chia hết cho B

Bài toán 6. Chứng minh với số nguyên n ta có: A n= +5 5+ 5+ + 5chia hết

choB= + + + +1 n

(Chuyên sư phạm Hà Nội 2001)

Hướng dẫn giải

(41)

CH

IN

H

P

H

C

K

T

H

I H

C S

IN

H

GI

I C

P H

AI

Lại có: 2A=(n 15+ +) (n 1− )5+25 + (n 2− )5+35+ + + n( 5)

   

   

Nhận thấy số hạng chia hết cho (n +1) nên 2A n( +1) ( )1

Lại có 2A 2n− =(n 1− )5+15 + (n 2− )5+25+

   

    chia hết cho n

Do 2n n5 nên 2A n ( )2

Do 2n n5 nên 2A n ( )2

Từ (1) (2) suy 2A chia hết cho n(n + 1) 2A 2B ⇒A B (đpcm)

Chú ý: Ta có cơng thức tổng qt: với n số nguyên dương k số tự nhiên lẻ thì:

( )

( ) ( )

)1

)1 2

+ + + + + +

+ + + +

 

k k k

k

k k

a n n

b k n n

Đây tập, bạn đọc tự chứng minh để củng cố kiến thức

Dạng 5: Sử dụng phương pháp xét số dư

* Cơ sởphương pháp: Để chứng minh A(n) chia hết cho p ta xét số n có dạng n = kp + r

với r∈{0;1; 2; ;p−1 }

* Ví dụminh họa:

Bài toán Chứng minh n 2n 7( 2+ ) chia hết cho với số nguyên dương n. Hướng dẫn giải

Xét trường hợp:

- Trường hợp 1: n = 3k ( + )=  ( ) + = ( + )

 

2

2

n 2n 3k 3k 3k 18k 3

- Trường hợp 2: n = 3k +

( ) ( ) ( )

( ) ( )( )

 

+ = +  + + 

 

= +  + + + = + + +

2

2

n 2n 3k 3k

3k 18k 12k 3k 6k 4k 3

- Trường hợp 3: n = 3k +

( ) ( ) ( )

( ) ( )( )

 

+ = +  + + 

 

 

= +  + + + = + + +

2

2

n 2n 3k 2 3k

3k 18k 24k 3k 6k 8k 3

Từ trường hợp suy n 2n 7( 2+ ) chia hết cho 3.

(42)

CH

UY

ÊN

Đ

S

H

C

Hướng dẫn giải

Trong số n (7n + 1) phải có số chẵn nên n 2n 7n 2( + )( + )

Mà (3, 2) = nên ta cần chứng minh n 2n 7n 3( + )( + )

Xét trường hợp:

- Trường hợp 1: n = 3k n 2n 7n 1( + )( + =) 3k 6k 21k 3( + )( + )

- Trường hợp 2: n = 3k + 2n 7+ =(6k 3+ ) ⇒n 2n 7n 3( + )( + )

- Trường hợp 3: n = 3k + 7n 1+ =(21k 15 3+ ) ⇒n 2n 7n 3( + )( + )

Từ trường hợp suy n(2n + 7)(7n + 1) chia hết cho

Bài toán Cho a, b, c số nguyên Chứng minh (a3+b c 93+ 3) một

trong ba số a, b, c chia hết cho

Hướng dẫn giải

Với a, b, c số nguyên ta có a 3q r ; b 3q= 1+ 1 = 2+r ;c 3q2 = 3+r3 với q ;q ;q1 2 3 số nguyên số dư r ;r ;r1 2 3∈ −{ 1;0;1}

Dễ thấy = = =

1

r r ;r r ;r r Từ ta

( ) ( ) ( )

= + = + = + = + = + = +

3 3

1 1 2 3 3

a 3q r 9k r ; b 3q r 9k r ;c 3q r 9k r

Khi ta 3+ 3+ = ( + + ) (+ + + ) 3

a b c k k k r r r

Mà theo giả thiết ta có (a3+b c 93+ 3) Do nên ta suy ( + + ) r r r Dễ thấy r r r1+ +2 3 ≤3, suy r r r 01+ + =2 3

Do r ;r ;r1 2 3∈ −{ 1;0;1} nên từ r r r 01+ + =2 3 suy r ;r ;r1 2 3 có số Điều có nghĩa ba số a, b, c có số chia hết cho

Bài toán Cho x, y, z số nguyên thỏa mãn (x y y z z x− )( − )( − )= + +x y z ( )* Chứng minh (x y z+ + )chia hết cho 27

(43)

CH

IN

H

P

H

C

K

T

H

I H

C S

IN

H

GI

I C

P H

AI

Ta xét trường hợp sau:

- Nếu số x, y, z chia cho có số dư khác (x – y), (y – z), (z – x) khơng chia hết cho (x – y)(y – z)(z – x) không chia hết cho Nhưng tổng số (x + y + z) chia hết cho điều trái với điều kiện (*) tốn, trường hợp xảy

-Nếu số x, y, z có số chia cho có số dư (x – y), (y – z), (z – x) có hiệu chia hết cho (x – y)(y – z)(z – x) chia hết cho Nhưng tổng số (x + y + z) không chia hết cho điều trái với điều kiện (*) bày tốn, trường hợp xảy

Vậy số x, y, z chia cho phải số dư, (x – y), (y – z), (z – x) chia hết tích (x – y)(y – z)(z – x) chia hết cho 27 Mặt khác theo giả thiết (*) ta có (x – y)(y – z)(z – x) = x + y + z nên (x + y + z) chia hết cho 27

Vậy toán chứng minh

Dạng 6: Sử dụng phương pháp phản chứng

* Cơ sởphương pháp: Để chứng minh A(x) không chia hết cho n ta giả sử A(x) chia hết cho n sau dùng lập luận để mâu thuẩn để điều giả sử sai

* Ví dụminh họa:

Bài toán Chứng minh n2+ −n 16 không chia hết cho 25 với số tự nhiên n Hướng dẫn giải

Giả sử n2 + −n 16 chia hết cho 25

Do n2+ −n 16 chia hết cho 25 nên chia hết cho

Ta có: n2+ −n 16=(n n 10+ )( − )−

Do n2+ −n 16 10 chia hết (n + 3)(n – 2) chia hết cho (1)

Mặt khác (n + 3) (n – 2) có hiệu nên chúng chia hết cho không chia hết cho 5, lại (1) nên (n + 3) (n – 2) chia hết cho suy ta có (n + 3)(n – 2) chia hết hết cho 25

Tức n2+ −n 16 chia cho 25 dư 15 mâu thuẫn với giả sử, toán chứng minh

Bài toán Chứng minh với số tự nhiên n,n3 chia hết cho n chia hết

cho

(44)

CH

UY

ÊN

Đ

S

H

C

Giả sử n không chia hết cho Khi n có dạng n = 3k +1 n = 3k + (với k số tự nhiên)

Nếu n = 3k + n3 =(3k 1+ )3 =27k3+27k2 +9k 1+ không chia hết cho 3 Nếu n = 3k + n3 =(3k 2+ )3 =27k3+54k2+36k 4+ khơng chia hết cho 3

Cả hai trường hợp mâu thuẫn suy n phải hết cho toán chứng minh

Bài toán Chứng minh số dương có tổng bình phương chia hết cho số phải chia hết cho

Hướng dẫn giải

Giả sử số ngun dương a, b có số khơng chia hết cho 3, chẳng hạn số a Khi a = 3k + a = 3k + với k số tự nhiên, ta cóa2 = +3l 1 số b chia hết cho

hoặc khơng chia hết cho a2 +b2 ln có dạng 3m + 3m + 2, nghĩa không chia

hết cho 3, mâu thuẫn

Vậy toán chứng minh

Dạng 7: Sử dụng phương pháp quy nạp

* Cơ sở phương pháp: Để kiểm tra mệnh đề với số tự nhiên n p≥ ta làm sau:

1) Kiểm tra mệnh đề với n = p.

2) Giả sử mệnh đề n = k (Giải thiết quy nạp) 3) Chứng minh mệnh đề với n = k + 1.

Nhận xét: Trong việc chứng minh phương pháp quy nạp bạn cần khai thác triệt để giả thiết quy nạp (là mệnh đề chia hết n = k), tức trình giải toán bước chứng minh n = k + bạn phải biến đổi xuất giả thiết quy nạp

* Ví dụminh họa:

Bài toán Chứng minh n 2n 7( 2+ ) chia hết cho với số nguyên dương n. Hướng dẫn giải

Với n = ta có: n 2n 7( + )=1 7( + )=9 3 , tốn với n=1

Giải sử toán đến n=k với k≥1,k N∈ tức là:

( ) ( ) ( *)

(45)

CH

IN

H

P

H

C

K

T

H

I H

C S

IN

H

GI

I C

P H

AI

( ) ( ) ( )( )

( ) ( )

( )

2 2

3 2

3

2

k k n 2n 4n

2n 4n 9n 2n 4n

2n 7n 6n 6n

3x 2n 2n 3y

 

+  + + = + + +

 

= + + + + +

= + + + +

= + + + = 

Do n 2n 7( + ) chia hết cho với

1

n= +k

Vậy toán chứng minh

Bài toán Chứng minh 4n +15n−1 chia hết cho với n N∈ *

Hướng dẫn giải

Với n = ta có: A 18=

chia hết cho 9, tốn với n = Giải sử toán đến n=kvới k≥1,k N∈ tức là:

( )

2 *

4k 15k hay 4k 15k 9x x N+ −  + − = ∈ ⇔4k =9x 15k 1− + ,

ta cần chứng minh toán với n = k + 1 Thật vậy:

( )

( )

k k

4 15 k 1 4.4 15k 14

4 9x 15k 15k 14 36x 45k 18

+ + + − = + +

= − + + +

= − + 

Do A 4n 15n 1= 2+ − chia hết cho với

1

n= +k Vậy toán chứng minh

Bài toán Chứng minh 52n+7 chia hết cho với số nguyên dương n Hướng dẫn giải

• Với n 1, ta có = 5 32 82 + =  (đúng) • Giả sử mệnh đề với , tức ta có 52n+7 8 .

• Ta cần chứng minh mệnh đề với n Thật vậy, ta có+

( +) ( )

+ = + = + +

2 n 2n 2n 2n

5 25.5 24.5

Để ý 52n+7 8 24.5 82n Do ta ( )+

+ 

2 n

5

Vậy theo nguyên lý quy nạp ta 52n+7 chia hết cho với số nguyên dương n

Bài toán Cho n số nguyên dương, Chứng minh rằng: 2 ( )

7.2 − −

= n + n

(46)

CH

UY

ÊN

Đ

S

H

C

Hướng dẫn giải

Xét với n = ta có: C =10 5 Vậy (1) với n =

Giả sử (1) với n = k (k≥1,k N∈ ), tức là: 2 ( )

7.2 − −

= k + k

k

C

Ta chứng minh (1) với n = k + 1, tức phải chứng minh:

( ) ( )

2 2 1

1 7.2

+ − + −

+ = k + k

k

C

Ta có: 2( 1) 2( 1) 2 2 2 2

1 7.2 7.2 3 4.7.2 9.3

+ − + − + − − − −

+ = k + k = k + k = k + k

k

C

( 2 1) 2

4 7.2 − − 5.3 − 5.3 −

= k + k + k = + k

k

C

Vậy theo nguyên lý quy nạp ta 2

7.2 − −

= n + n

C chia hết cho với số nguyên

dương n

Bài toán Chứng minh số tạo 3n chữ số giống chia hết cho3n

với ∈ *

n N

(Đề thi học sinh giỏi lớp toàn quốc năm 1978) Hướng dẫn giải

Với n = 1, ta có: aaa=111.a3, Vậy tốn với n =

Giả sử toán đến n = k (k≥1,k N∈ ), tức là: 

3

k

k

aa a

Ta chứng minh mệnh đề đến n = k +

Thật vậy:         

1 1 1

1

3 3 3 3 3

100 0100 01 100 0100 01

k k k k k k k k k

n

aa a aa a aa a aa a aa a do

+ − − − −

+  

= = ×  

 

 

Vậy toán chứng minh

Dạng 8: Sử dụng nguyên lý Dirichlet

* Cơ sởphương pháp: Đầu tiên ta phải nắm nguyên lý Dirichlet: “Nhốt m = kn + 1

con thỏ vào k (k < n) chuồng tồn chuồng có n + thỏ”

Áp dụng nguyên lý Dirichlet vào toán chia hết sau: “Trong m = kn + số có

n + số chia hết cho k có số dư”

* Ví dụminh họa:

Bài tốn Chứng minh số ngun tìm ba số có tổng chia hết cho

(47)

CH

IN

H

P

H

C

K

T

H

I H

C S

IN

H

GI

I C

P H

AI

Trường hợp 1: Nếu tồn loại số dư chia cho thì:

( )

1

2 2 3

3

3

3 3

3

= +

 = + ⇒ + + = + + +

 

= +



a k

a k a a a k k k

a k

Trường hợp 2: Chỉ tồn hai loại số dư, theo nguyên lýDirichlettrong số ngun ln tồn số dư chia cho suy tổng số chia hết cho

Trường hợp 3: Chỉ tồn tài du loại số dư chia hết cho suy số tùy số chia hết cho

Bài toán Cho số nguyên phân biệt a, b, c, d Chứng minh rằng:

( )( )( )( )( )( )12

= − − − − − − 

A a b a c a d b c b d c d

Hướng dẫn giải

Theo nguyên lýDirichlet số nguyên tùy ý tồn hai số nguyên tùy ý có số dư chia hết cho suy A3

Trường hợp 1: số số chẵn nên tồn hiệu chia hết cho suy A4

Trường hợp 2: số số lẻ nên tồn hiệu chia hết cho suy A4

Trường hợp 3: số chẵn hai số lẻ nên tồn hiệu chia hết cho suy A4

Trường hợp 4: số chẵn số lẻ , từ số chẵn cho ta hiệu chia hết cho suy A4

Trường hợp 5: số lẻ số lẻ, từ số lẻ cho ta hiệu chia hết cho suy A4

Do A chia hết cho mà (3, 4) = nên A chia hết cho 12

Bài toán Chứng minh 101 số ngun tìm hai số có chữ số tận giống

Hướng dẫn giải

Lấy 101 số nguyên chia cho 100 theo nguyên lý Dirichle có có số có số dư chia cho 100 Suy 101 số cho tồn số có chữ số tận giống

Bài toán Cho 2014 số tự nhiên x ,x ,x , ,x Chứng minh tồn số 1 2 3 2014 chia hết cho 2014 tổng số số chia hết cho 2014

Hướng dẫn giải

(48)

CH

UY

ÊN

Đ

S

H

C

Nếu tồn Si với i = 1, 2, 3, …., 2014 chia hết cho 2014 tốn chứng minh Nếu không tồn Si với i = 1, 2, 3, …., 2014 chia hết cho 2014 Đem 2014 số chia cho 2014 nhận 2014 số dư Giá trị số dư nhận thuộc vào tập hợp{1,2,3, ,2013 Vì 2014 số dư mà có 2013 giá trị nên theo nguyên lý Dirichlet có 2} số dư

Kí hiệu hai số S ,S có số dư chia cho 2014 m n {m,n N,1 n m 2014∈ ≤ < ≤ }

Thì hiệu: Sm−Sn =xn 1+ +xn 2+ + + xmchia hết cho 2014

Nhận xét:Ta tổng qt hóa tốn sau: Cho n số tự nhiên x ; x ; ; x1 2 n Chứng minh rằng n số có số chia hết cho n số số có tổng chia hết cho n.

Bài toán Chứng minh số tự nhiên có chữ số chọn hai số mà viết liền ta số có chữ số chia hết cho

Hướng dẫn giải

Lấy số cho chia số dư nhận giá trị 0, 1, 2, 3, …, Theo nguyên tắc Dirichlet có hai số số dư, giả sử abc def chia cho có số

dư Giả sử abc=7k+r def =7k+r Ta có:

( ) ( )

100 1000 7 1000 1001

abcdef = abc+def = k+r + l+ =r k+ +l r

Vậy tốn chứng minh

Bài tốn Có hay không số nguyên dương k để 29k số có chữ số tận

là 0001

Hướng dẫn giải

Ta cần chứng minh tồn số nguyên k cho

29k −1 10 Thật vậy, lấy 104 +1 số:

4

2 10

29, 29 , , 29 − chia cho 104, có hai số có hiệu chia hết cho 104, giả sử 29n

và 29m(n>m) Ta có 29m −29 10n hay 29m(29n m− −1 10 )

Vì ( 4)

29 ,10m =1 nên 29n m− −1 10 (đpcm) Dạng 9: Xét đồng dư

(49)

CH

IN

H

P

H

C

K

T

H

I H

C S

IN

H

GI

I C

P H

AI

Định nghĩa: Cho a, b số nguyên (n số nguyên dương) Ta nói a đồng dư với b

theo modunn kí hiệu a b≡ (mod n)nếu a b có số dư chia cho n.

Như vậy: a b≡ (mod n) (⇔ −a b n) .Ví dụ: 2019 mod 5≡ ( )

Một số tính chất bản:

1) Với số nguyên a ta có: a a≡ (mod n)

2) a b≡ (mod n)⇔ ≡b a(mod n)

3) a b≡ (mod n) b c≡ (mod n)⇒ ≡a c(mod n)

4) a b≡ (mod n) c d≡ (mod n) (⇒ ± ≡ ±a c) (b d)(mod n)

Hệ tính chất 4)

( ) ( ) ( )

( ) ( )

1 2

1 2

mod , mod , , mod

a mod

n n

n n

a b n a b n a b n

a a b b b n

≡ ≡ ≡

⇒ + + + ≡ + + +

5) a b≡ (mod n) c d≡ (mod n)⇒a c b d ≡ mod( n)

Hệ tính chất 5)

( ) ( ) ( )

( ) ( )

1 2

1 2

mod , mod , , mod

a mod

n n

n n

a b n a b n a b n

a a b b b n

≡ ≡ ≡

⇒ ≡

6) a b≡ (mod m)⇔anbn(mod m) ∀ ∈n N

7)Nếu a b≡ (mod m) d là ước chung a b cho (d, m) = 1

thìa b(modm) d d

8)Nếu a b≡ (mod m) d là ước chung a, b, m thìa b modm

d d d

 

≡  

 

9)Nếu a r≡ (mod m) 0≤ <r m,thì r số dư phép chia a cho m.

* Cơ sởphương pháp:Sử dụng định nghĩa tính chất đồng dư thức để giải toán chia hết

* Ví dụminh họa:

Bài tốn Chứng minh rằng:

7.5 n 12.6n

A= + chia hết cho 19

Hướng dẫn giải

(50)

CH

UY

ÊN

Đ

S

H

C

( ) ( ) ( )

≡ ⇒ ≡ ⇒ ≡

25 mod 19 25n mod 19n 7.25n 7.6 mod 19n

Do

( )

⇒7.25 12.6n+ n ≡7.6 12.6 mod 19n+ n

Mà 7.6 12.6n+ n=19.6 19n ⇒7.25 12.6 19n+ n ⇒ =A 7.52n+12.6 19n

Vậy toán chứng minh

Bài toán Chứng hai số: A=61000−1 và B=61001+1

Chứng minh A B bội số

Hướng dẫn giải

Ta có: 6≡ −1 mod 7( )⇒61000 ≡ −( ) (1 1000 mod 7)⇒61000 ≡1 mod 7( )⇒61000−1 7

Vậy A bội

Từ 61000 ≡1 mod 7( )⇒61001 ≡6 mod 7( )

Mà 6≡ −1 mod 7( )⇒61001 ≡ −1 mod 7( )⇒61001+1 7

Vậy B bội

Bài toán a) A = 22225555 + 55552222 chia hết cho

b) 1962 1964 1966

1961 1963 1965

B= + + + chia hết cho

Hướng dẫngiải

a) Xét số dư 22225555 chia cho

Ta có: 2222 ≡ (mod 7) (1)

⇒22224≡34 (mod 7) ⇒ 22224≡ 81 (mod 7)

Mà 81 ≡ (mod 7)

⇒22224≡ (mod 7) (2)

Nhân vế với vế (1) (2) ta 22225≡ 3.4 (mod 7) ⇒22225≡ (mod 7) ⇒22225555≡ 51111 (mod 7) (3)

(51)

CH

IN

H

P

H

C

K

T

H

I H

C S

IN

H

GI

I C

P H

AI

Cộng vế với vế (3) (4) ta có: A ≡ 21111 + 51111 (mod 7) (5)

Mặt khác: 21111 + 51111≡ (2 + 5) (mod 7)

≡0 (mod 7) (6)

Từ (5) (6) ta được: A ≡ (mod 7) Vậy: A = 22225555 + 55552222 chia hết cho

b) Ta có:

Ta có: 1961 ≡ (mod 7) => 19611962 ≡ (mod 7)

Tương tự: 1964 1964( ) ( )3 654( ) 654( ) ( )

1963 ≡3 mod ≡9 mod ≡9.27 mod ≡2 mod

1966 ( ) (1966 ) ( )3 655( ) 655( ) ( )

1965 ≡ −2 mod ≡2 mod ≡2.8 mod ≡2 mod

( ) ( )

1 2 mod mod

B

⇒ ≡ + + + ≡

Vậy: 1962 1964 1966

1961 1963 1965

B= + + + 

Bài tốn Tìm số dư phép chia: 1532 15− cho

Hướng dẫn giải

Ta có: 1532 mod9≡ ( )⇒15325 ≡2 mod 95( )

Mà 25 ≡5 mod9( )⇒15325 ≡5 mod 9( )⇒1532 mod 95− ≡ ( )

Vậy số dư phép chia 1532 15− cho

Dạng 10: Tìm điều kiện biến để chia hết

Bài tốn 1.

a) Tìm n nguyên để

2

= + − +

A n n n chia hết cho = 2− B n n

b) Tìm anguyên để a 2a 7a 73− 2+ − chia hết cho a 32 + Hướng dẫn giải

a) Chia A cho B ta có: ( )( )

2 3

+ − + = + − +

n n n n n n

Để A chia hết cho B phải chia hết cho ( )

1

− = −

n n n n chia hết cho n, ta có:

n -1 -2

n - -2 -3

n(n – 1) 2

(52)

CH

UY

ÊN

Đ

S

H

C

Vậy để giá trị biểu thức

2

= + − +

A n n n chia hết cho giá trị biểu thức B=n2−nthì n = -1

hoặc n =

b) Thực phép chia a 2a 7a 73− 2+ − cho a 32+ kết quả:

( )( ) ( )

3 2

a 2a 7a 7− + − = a +3 a 2− + 4a 1−

Để phép chia hết 4a 1− phải chia hết cho a 32+

( ) ( )

( )( ) ( )

( ) ( )

− +

⇒ − + + ∈ ⇒ + ∈

⇒ − +

  

2

2

2

4a a

4a 4a a (a 4a )

16a a

( )

( )

⇒ +

 + =  =

⇒ + = ⇒ = −

 

 2

49 a

a a

a 49 loai a

Thử lại ta a = a = - thỏa mãn

Bài tốn 2. Tìm số tự nhiên n để n2+(n 1+ ) (2 + n 2+ ) (2 + n 3+ )3chia hết cho 10 Hướng dẫn giải

Ta có: A n= 2+(n 1+ ) (2+ n 2+ ) (2+ n 3+ )3 =2 2n 6n 7( 2+ + )

( )

⇔ + + ⇔ + + ⇔ + +

⇔ + +

   

2 2

2

A 10 2n 6n 2n 6n n 3n

n 3n

Do n(n + 3) có tận hay n có tận

Vậy n có tận thỏa mãn yêu cầu tốn

Bài tốn 3. Tìm số ngun dương n để (n + 3)(n + 4) chia hết cho 3n

Hướng dẫn giải

Ta có: (n n 3n+ )( + ) ⇔n 7n 12 3n2 + +  ⇔n2+ +n 12 3n

( ) ( )  + +

 ⇒ 

+ + 

 

2

n n 12 n

n n 12

Từ (1) suy ra: 12 n ⇒ ∈n {1,2,3,4,6,12} ( )3

Từ (2) suy ra: ( + ) ⇒ = += ( ∈ ) 

 n 3k

n n k N

(53)

CH

IN

H

P

H

C

K

T

H

I H

C S

IN

H

GI

I C

P H

AI

Loại số có dạng n = 3k + (1) ta được:

n 12

3n 18 36

n2 + n + 12 18 24 54 168

Chỉ có n = n = n n 12 chia hết cho 3n đó: 2+ + (n n 3n+ )( + )

Vậy n = n =

Bài tốn 3. Tìm số nguyên dương x y lớn cho x + chia hết cho y y + chia hết cho x

Hướng dẫn giải

Giải sử ≤ x ≤ y

a) Xét y = x = 2, khơng thỏa mãn x + chia hết cho y

b) Xét y ≥ Đặt x + = ky (kN) (1) ky = x + ≤ y + ≤ y + y = 2y nên k ≤ 2.

Với k = 1, từ (1) có x + = y Thay vào: y+3x x+6xnên lại có x > nên

{2;3; }

x

x

y

Với k = 2, từ (1) có x + = 2y Thay vào: y+3x 2y+6x⇒ +x 9x⇒9x

do x > nên x∈{ }3;9

Khi x = y = 3, thử lại

Khi x = y = 6, loại trái với x ≤ y

Các cặp số (x, y) phải tìm (2; 5), (5; 2), (3; 6), (6; 3), (6; 9), (9; 6), (3; 3)

Dạng 11: Các tốn cấu tạo số liên quan đến tính chia hết số tự nhiên

* Cơ sởphương pháp: Số tự nhiên A a a a biểu diễn dạng tổng lũy = n n 1− 0

thừa sau: −

− −

= = n+ n 1+ +

n n n n

A a a a a 10 a 10 a Trong a ;a ; ;an n 1− 0 chữ số an khác Khi ta có dấu hiệu chia hết sau:

{ }

0

2 0; 2; 4; 6;8

A a a

•  ⇔  ⇔ ∈

( 1 )

3 n n

A a a aa

•  ⇔ + + + + 

1

4

A a a

(54)

CH

UY

ÊN

Đ

S

H

C

{ }

0

5 0;5

A a a

•  ⇔  ⇔ ∈

2

8

A a a a

•  ⇔ 

( 1 )

9 n n

A a a aa

•  ⇔ + + + + 

( ) ( )

11 11

A a a a a

•  ⇔ + + − + + 

1

25 25

A a a

•  ⇔ 

2

125 125

A a a a

•  ⇔ 

* Ví dụminh họa:

Bài tốn Tìm số tự nhiên có ba chữ số, chia hết cho 9, biết chữ số hàng chục trung bình cộng hai chữ số

Hướng dẫn giải

Gọi số phải tìm abc Do a+ +b c chia hết cho 9 2b= +a c nên 3b chia hết cho

9, suy b chia hết cho 3 Như b∈{0;3;6;9} Do abc5 nên c∈{ }0;5

Xét số ab0 với a=2b, ta số 630

Xét số ab5 với a=2b−5, ta số 135 675

Bài toán Tìm chữ số a b, cho:

a) a− =b 4 7 1a b chia hết cho

b) a− =b 6và 5a + b chia hết cho

Hướng dẫn giải

a) Số 3a b  ⇒ + + + +7 a b 3 ⇒13+ +a b3⇒ +a 3 chia cho dư 2( )1

Ta có а− =b nên:

4≤ ≤a 9

0≤ ≤b

Suy 4≤ + ≤a b 14 ( )2

Mặt khác ab số chẵn nên a+b số chẵn( )3 Từ ( )1 , ( )2 , ( )3 suy га: а+ ∈b {8;14 }

(55)

CH

IN

H

P

H

C

K

T

H

I H

C S

IN

H

GI

I C

P H

AI

b) 9a + b  ⇒512 10+ (a+b)9

( )

504 a b a b

⇒ + + + + +  ⇒ +a b chia cho 9 dư

Do a+ ≥ − =b a b nên a+ =b 10 Từ tìm được:a=8; b=2

Bài toán Chứng minh ab=2.cd abcd chia hết cho 67

Hướng dẫn giải

Ta có abcd =100⋅ab+cd =201⋅cd chia hết cho 67

Bài toán Cho số abc chia hết cho 27 Chứng minh bca chia hết cho 27

Hướng dẫn giải Ta có: abc27

0 27

1000 27

999 27

27.37 27 abc

a bc

a a bc

a bca

⇒ +

⇒ + +

⇒ +

  

Do 27.37a27 nên bca27

Bài toán Chứng minh ab+cd +eg chia hết cho 11 abcdeg chia hết cho 11 Hướng dẫn giải

( )

deg 10000.abc = ab+100×cd +eg =9999×ab+99×cd + ab+cd +eg chia hết cho 11

Bài tốn Tìm chữ số a, b cho 62ab427 chia hết cho 99

Hướng dẫn giải

Cách 1. Ta có 99 9.11= (9,11 1)= nên ta có 62ab427 chia hết cho 99 62ab427 chia hết cho chia hết cho 11

(56)

CH

UY

ÊN

Đ

S

H

C

Từ ta (a b 3+ + ∈) {9;18} nên suy (a b 3+ + ∈) { }6;15

• Ta có 62ab427 chia hết cho 11 (6 a 7+ + + ) (− b 11+ + ) hay (a b 11− + )

Từ ta (a b 2− + ∈) { }0;11 nên suy (a b− ) {∈ −2;9} Từ ta xét trường hợp sau

+ Trường hợp 1:  − = + =

a b

a b 6, trường hợp không tồn chữ số a, b thỏa mãn + Trường hợp 2:  − = + =

a b

a b 15, trường hợp không tồn chữ số a, b thỏa mãn + Trường hợp 3:  − = − ⇔ =

+ = =

 

a b a

a b b

+ Trường hợp 4:  − = − + =

a b

a b 15, trường hợp không tồn chữ số a, b thỏa mãn Vậy chữ số thỏa mãn yêu cầu toán a 2; b 4= =

Cách 2. Ta có 62ab427 62.100000 ab.1000 427 62630.99 ab.990 10.ab 57= + + = + + +

Suy 62ab427 chia hết cho 99 10.ab 57 chia hết cho 99 +

Từ ta 10.ab 57 99.k với k số tự nhiên + =

Dễ thấy 10.ab 57 có chữ số tận 7, + 99.k phải có chữ số tận nên ta k 3=

Từ suy 10.ab 57 99.3+ = ⇒ab 24 =

Vậy chữ số thỏa mãn yêu cầu toán a 2; b 4= =

Bài tốn Tìm chữ số a biết 20 20 20a a a chia hết cho

Hướng dẫn giải

20 20 20 20 20 1000 20 (20 1000 20 ).1000 20

n= a a a= a a + a= a + a + a

1001.20 1000a 20a

= +

Theo đề n chia hết cho 7, mà 1001 chia hết 20a chia hết cho

(57)

CH

IN

H

P

H

C

K

T

H

I H

C S

IN

H

GI

I C

P H

AI

Dạng 12: Cácbài chia hết sử dụng định lý Fermat

* Cơ sởphương pháp:

Với p số nguyên tố ta có: apa(mod p)

Đặc biệt, (a p, )=1thì

1 p

a − ≡ (mod p)

Chứng minh

Xét số a, 2a, …, (p−1)a Dễ thấy, khơng có số p−1 số chia hết

cho p khơng có hai số có số dư chia cho p Vậy chia p−1 số nói

cho p, ta nhận số dư 1, 2, …, p−1 Suy a 2( ) ( ) (a 3a ( p−1)a)≡1.2.3.(p−1)

(mod p) hay (1.2.3 (p−1 ))ap−1≡1.2.3 (p−1) (mod p)

Vì (1.2.3 (p−1 ,) p)=1 nên

1 p

a − = (mod p)

* Ví dụminh họa:

Bài tốn Cho *

; ,

aZ m nN Chứng minh a6n +a6m7khi a7

Hướng dẫn giải

Giả sử 6

7

n m

a +aa  7ta có ( )a, =1

Theo định lý Fermat: ( ) ( )

1 mod n mod

a ≡ ⇒aa6m ≡1 mod 7( )

( )

6

2 mod

n m

a a

⇒ + ≡ Vơ lí! Vậy a7

Ngược lại, a7 a6n +a6m7

Bài toán Chứng minh 24 34

3 n+ +2 n+ +5 11, với nN

Hướng dẫn giải Theo định lí Fermat, ta có 10 ( )

3 ≡1 mod11 210 ≡1 mod11 ( )

Ta tìm số dư phép chia

2 n+ 34n+1 cho 10, tức tìm chữ số tận

chúng

Ta có ( )

2 n+ =2.16n ≡2 mod 10 ⇒2 n+ =10k+2

( ) ( )

4

3 n+ =3.81n ≡3 mod 10 ⇒3 n+ =10l+3 k l, ∈N

Mà 10 ( )

3 k ≡1 mod11 210l ≡1 mod 11( )nên

( )

4

2 10 10 3

3 n+ +2 n+ + =5 k+ +2 l+ + ≡5 +2 + ≡5 mod 11 Vậy 24 34

(58)

CH

UY

ÊN

Đ

S

H

C

Bài tốn Một số có 6n chữ số chia hết cho Chứng minh chuyển chữ số tận lên đầu số số chia hết cho

Hướng dẫn giải

Gọi số ban đầu N = 10A + a, với a chữ số tận N A có 6n – chữ số Sau chuyển a lên đầu ta số

.10 n

M =a − +A

Ta chứng minh N−3M7

Thật vậy, ta có ( )

3 3.10 n

NM = Aa − −

Áp dụng định lý Fermat ta có:

( ) ( ) ( )

6 6

10 ≡1 mod ⇒10 n ≡1 mod ⇒3.10 n ≡ ≡3 10 mod

( )

6

3.10 n− mod

⇒ ≡

Vậy N −3M7, từ suy điều phải chứng minh

Dạng 13: Các toán chia hết liên quan đến đa thức

* Cơ sởphương pháp:

Định lýBơdu:

Phần dư phép chia đa thức f(x) cho nhị thức x - a giá trị đa thức x = a Tức là: f(x) = (x - a).g(x) + f(a)

Chứng minh : Gọi g(x) đa thức thương R số dư thì: f(x) =(x - a).g(x) + R

f(a) = (a - a).g(a) + R = R (đpcm)

* Ví dụminh họa:

Bài tốn Cho đa thức

( )

P x =ax +bx c+ Biết P x( ) chia cho x + dư 3,P x( )chia cho x dư

1 vàP x( )chia cho x – dư Tìm hệ số a, b, c

(Trích đềvào 10 Chuyên Nam Định năm 2015-2016)

Hướng dẫn giải

Vì P(x) chia cho x + dư nên P(x) – chia hết cho x +

⇒P(x) – = f(x).(x + 1)

Thay x = –1 vào đẳng thức ta có: P(–1) – = f(–1).( –1 + 1) =

(59)

CH

IN

H

P

H

C

K

T

H

I H

C S

IN

H

GI

I C

P H

AI

P(x) chia cho x – dư nên P(1) = (3)

Từ (1), (2), (3) ta có hệ phương trình:

2

2

2

.( 1) ( 1) 3

.0 1

5

.1

a b c a b c a

a b c c b

a b c c

a b c

 − + − + =  − + =  =

 + + = ⇔ = ⇔ =

  

 + + =  + + =  =

 

⇒P(x) = 3x2 + x + Thử lại ta thấy P(x) thỏa mãn đề Vậy P(x) = 3x2 + x + 1.

Bài toán 2. Tìm số thực a, b, cho đa thức 4x4 −11x3 −2ax2 +5bx 6− chia hết cho đa

thức x2 −2x 3.−

(Thi học sinh giỏi lớp 9, TP Hà Nội, năm học 2012 – 2013)

Hướng dẫn giải

Ta có x2 −2x x− = −2x 4+ − = (x 1− )2 −4

(x x 2)( ) (x x 1)( )

= − − − + = − +

Đặt thương q(x) ta có: 4x2 −11x3 −2ax2 +5bx 6− =(x x q x− )( + ) ( )

Chọn x=3 ta có: 4.34 −11a3 −2 3a +5 .3 6b − =0

⇒ 15b−18a= − ⇒21 5b−6a= −7 1( )

Chọnx= −1 , ta có: 4.( )−1 −11.( )−1 3−2a( )−1 +5b( )− − =1

⇒5b + 2a = (2)

Từ (1) (2) suy : 8a 16+ ⇒ =a Thay vào (2) suy ra: 5b 9+ = ⇒ =b

Bài tốn 3.Tìm đa thức f(x) biết: f(x) chia cho x + dư 1; f(x) chia cho x – dư 8; f(x) chia cho (x + 3)(x – 4) 3x cịn dư

Hướng dẫn giải

Theo định lý Bézout ta có f(3) 1;f(4) 8= =

Đặt dư f(x) chia cho (x x 4+ )( − )là ax + b

(60)

CH

UY

ÊN

Đ

S

H

C

• Với x =- ta có: 1= − +( 3)(− −3 3) ( ) ( )− +a 3− + b⇒ −b 3a 1= (1) • Với x = ta có: =(4 4 3.4+ )( − )( )+a.4 b+ ⇒ +b 4a 8=

(2)

Từ (1) (2) suy ra: 7a = ⇒ =a thay vào (2) ta b =

Từ ta được: f x( ) (= x x 3x x 4+ )( − ) + + Hay f x( ) 3= x3 −3x2 −35x +4

Bài toán 4.Chứng minh đa thức f x( ) (= x 3− )200 +(x 2− )100 −1chia hết cho đa thức

( )

g x = x −5x 6+

Hướng dẫn giải Ta có f 2( ) (= 3− )200+(2 2− )100 − =1 0nên f x( ) ( x 2− )

( ) ( )200 ( )100

f = 3− + 1− − =1 nên f(x) ⋮ (x - 3) Nên f(x) chia hết cho (x – 2)(x – 3) = x2 – 5x +

Bài toán 5.Cho đa thức P x( )=x x3− Q x( )=x81+x49+x25+x9+ +x 1.

a) Tìm số dư phép chia Q(x) cho P(x) b) Tìm x để Q x P x( ) ( )

Hướng dẫn giải

a) Ta có: P x( )=x x 1( 2− ); Q x( )=x x( 80− +1 x x) ( 48− +1 x x) ( 24 − +1 x x 5x 1) ( − +) +

Vì đa thức x80−1; x48−1; x 18 − chia hết cho x 12− nên phép chia Q(x) cho

P(x) dư 5x +

b) Để Q x P x( ) ( ) 5x x

+ = ⇔ = −

TỔNG KẾT CÁC PHƯƠNG PHÁP THƯỜNG ÁP DỤNG Để làm giải tốt toán chia hết, cần sử dụng linh hoạt phương pháp nêu trên, nhiều toán chia hết giải nhiều phương pháp, có tốn nhìn tương tự có phương pháp giải Để mô điều trích viết tác giả Nguyễn Đức Tấn tạp chí Tốn học tuổi trẻ:

(61)

CH

IN

H

P

H

C

K

T

H

I H

C S

IN

H

GI

I C

P H

AI

Cách 1:(Sử dụng phương pháp xét sốdư) Ta xét trường hợp sau:

- Trường hợp 1: n = 3k(k Z∈ ) n2+ + =n 3k k 1( + +) - Trường hợp 2: n = 3k + (k Z∈ )thì n2+ + =n 9k k 3( + +) - Trường hợp 3: n = 3k + (k Z∈ )thì n2+ + =n 3k( 2+5k 1+ )+

Từ trường hợp suy n n2+ +1 không chia hết cho với số nguyên n

Cách 2: (Sử dụng phương pháp tách tổng)

Ta có: n n2+ + =1 (n−1)(n+ +2 3)

Do (n + 2) – (n – 1) = nên (n + 2) (n – 1) đồng thời không đồng thời chia hết cho

Nếu (n+2 3;) ( n−1 3) ⇒(n−1)(n+2 9) nên (n−1)(n+2 3)+ không chia hết cho

Nếu (n + 2) (n – 1) đề không chia hết cho (n−1)(n+2 3)+ khơng chia hết cho

Vậy n n2 + +1 không chia hết cho với số nguyên n

Cách 3: (Sử dụng phương pháp phản chứng)

Giả sử (n n2+ +1 9.) Đặt n n2+ + =1 9m m Z( ∈ )⇒n n2+ + −1 9m=0 *( )

Phương trình (*) có ∆ =36m− =3 12( m−1)

Ta thấy ∆ số phương chia hết cho mà khơng chia hết (*) khơng có nghiệm Vô lý!

Vậy n n2 + +1 không chia hết cho với số nguyên n

Cách 4: Ta có: 4(n n2+ + =1) (2n+1)2+3

Nếu (2n+1 3) ⇒(2n+1 9)2 nên (2n+1)2+3sẽ không chia hết cho

Nếu (2n + 1) không chia hết cho (2n+1)2 khơng chia hết (2n+1)2+3sẽ khơng chia hết cho không chia hết cho

Vậy 4(n n2 + +1)

không chia hết n n2+ +1sẽ không chia hết cho với số nguyên n

Các bạn rèn luyện khảnăng sử dụng phương pháp chứng minh tốn chia hết thơng

qua toán tương tựsau: 1) Chứng minh: n2+11n+39

không chia hết cho 49 2) Chứng minh: n2+3n+5

không chia hết cho 49 3) Chứng minh: n2+5n+16

(62)

CH

UY

ÊN

Đ

S

H

C

Tuy nhiên với toán:

Chứng minh: 9n3+9n2+3n−16

không chia hết cho 343 với số nguyên n.

Ta dễ thấy với cách 1, 2, có lẽ phải bó tay, khai thác giải ý 343 7= ta có lời giải thật “dễthương” sau:

( )3

9n +9n +3n−16= 3n+1 −49

Nếu (3n+1 7) ⇒(3n+1 7)3 =343

nên (3n+1)3−49sẽ không chia hết cho 343

Nếu (3n + 1) không chia hết cho (3n+1)3−49 khơng chia hết (3n+1)3−49 không chia hết cho 343 7= 3

Vậy 9n3+9n2+3n−16sẽ không chia hết cho 343 với số nguyên n

Do để giỏi toán cần linh hoạt nắm vững phương pháp giải để vận dụng tốt ở các toán khác nhau!

C BÀI TẬP ÁP DỤNG

Câu Chứng minh a a 30 a5−  ( ∈)

Câu a) Đặt A n= +3n2 +5n 3.+ Chứng minh Achia hết cho với giá trị

nguyên dương n

b) Nếu achia 13 dư 2và b chia 13 dư a b2+ 2chia hết cho 13

Câu Chứng minh rằng: A=n n 73( 2− )2−36n 7

 

  với ∀ ∈n 

Câu Chứng minh n 28n3− chia hết cho 48 với nlà số nguyên chẵn

Câu Cho nlà số tự nhiên lẻ Chứng minh n n3− chia hết cho 24

Câu Chứng minh n 17n3+ chia hết cho 6 với n∈

Câu Chứng minh rằng: Q n= +(n 1+ ) (3+ n 9+ )3 với n∈*

Câu Chứng minh : 2019 2021

2021 +2019 chia hết cho 2020

Câu Chứng minh

a) 8 25+ 11chia hết cho 17

b) 1919 +6919chia hết cho 44

Câu 10 Chứng minh A n n 2= + + không chia hết cho 15 với số nguyên n.

(Đề thi HSG lớp huyện Thủy Nguyên 2018-2019)

Câu 11.Chứng minh với n N thì: ∈ n 6n 11n4+ 3+ 2+30n 24− chia hết cho 24

(Đề thi HSG lớp huyện Thanh Hà 2016-2017)

Câu 12 Cho a, b số nguyên thỏa mãn: 2a 3ab 2b2+ + 2 chia hết cho Chứng minh 2

(63)

CH

IN

H

P

H

C

K

T

H

I H

C S

IN

H

GI

I C

P H

AI

Câu 13. Cho n số tự nhiên không chia hết cho Chứng minh P=32n+ +3n chia hết cho 13 (Đề thi HSG lớp huyện Vũ Quang 2018-2019)

Câu 14. Cho biểu thức P a a a a= 1+ 2+ 3+ + 2019 với a ;a ;a ; ;a1 2 3 2019 số nguyên dương P chia hết cho 30 Chứng minh 5 5

1 2019

Q a= +a +a a+ + chia hết cho 30

(Đề thi HSG Thành Phố Hải Phòng 2018-2019)

Câu 15. Cho x số tự nhiên chẵn Chứng tỏ biểu thức M= x3 +x2 + x

24 12 có giá trị số nguyên

(Đề thi Chọn HSG lớp THCS Hiệp An 2018-2019)

Câu 16.Chứng minh với số tự nhiên n ta có:

7.5 n 12.6n

A= + chia hết cho 19

(Đề thi HSG lớp huyện Phù Ninh 2013-2014)

Câu17. Chứng minh với số tự nhiên nthì : A 5= n 2+ +26.5 8n+ 2n 1+ 59

Câu 18. Cho a ,a , ,a1 2 2016là số tự nhiên có tổng chia hết cho

Chứng minh rằng: 3

1 2016

A a a a= + + + chia hết cho

Câu 19 a)Chứng minh tổng hai số nguyên chia hết cho tổng lập phương chúng chia hết cho

b) Tìm số nguyên n để n 15+ chia hết cho n 13+

Câu 20. Cho số nguyên dương a, b, c thỏa mãn a2+b2 =c2 Chứng minh ab chia hết

cho a+ +b c

(Đềthi vào 10 Chuyên Lam Sơn năm 2019-2020)

Câu 21. Tìm số nguyên dương n bé để F = n3 + 4n2– 20n – 48 chia hết cho 125

(Đề thi HSG lớp huyện Hoằng Hóa 2015-2016)

Câu 22.Tìm tất cặp số nguyên dương a,b cho:

a+b chia hết cho a b2 −1

(đề thi học sinh giỏi lớp huyện Thanh Oai 2012-2013)

Câu 23. Cho số nguyên dương x, y, z thỏa mãn x y2 + =z2

Chứng minh A = xy chia hết cho 12

(Đề thi HSG lớp huyện Vĩnh Lộc 2016-2018)

Câu 24 Chứng minh số tự nhiên A 1.2.3 2017.2018 1 1

2 2017 2018

 

=  + + + + + 

 

chia hết cho 2019

(Đề thi HSG lớp huyện Hoài Nhon 2018-2019)

Câu 25 Tìm số dư phép chia đa thức (x x x x 2010+ )( + )( + )( + )+ cho đa thức x 10x 212+ +

(64)

CH

UY

ÊN

Đ

S

H

C

Câu 27. Cho đa thức f(x) x 3x= 3− 2+3x 4.− Với giá trị nguyên x giá trị đa

thức f(x)chia hết cho giá trị đa thức x2+2

Câu 28. Giả sử f(x) đa thức bậc với hệ số nguyên.Chứng minh rằng: Nếu f(x) 7với x

∀ ∈ Ζ hệ số f(x) 7

(Đề thi học sinh giỏi lớp trường Trần Mai Ninh năm 2012-2013)

Câu 29 Tìm số dư phép chia (x x x x 2033+ )( + )( + )( + )+

cho x 12x 302+ +

Câu 30 Tìm đa thức f(x) biết : f(x) chia cho x 2+ dư 10, f x( )chia cho x 2− dư 26, ( )

f x chia cho x 42 − được thương −5xvà dư

Câu 31. Cho đa thức P(x) = ax3 + bx2 + cx + d với a, b, c, d hệ số nguyên Chứng minh

rằng P(x) chia hết cho với giá trị nguyên x hệ số a, b, c, d chia hết cho

(đề thi học sinh giỏi lớp huyện Thạch Hà 2016-2017)

Câu 32. Cho p số nguyên tố lớn Chứng minh p20−1 chia hết cho 100

(Đề thi HSG lớp huyện Lục Nam 2018-2019)

Câu 33. Chứng minh với số tự nhiên n

3 11

n + n+ không chia hết cho 49

(Đề thi HSG lớp TP Hà Nội 2019-2020)

Câu 34 Cho N = k4 + k3– 16 k2– 2k +15, k số nguyên Tìm điều kiện k để số N chia

hết cho 16

(Đề thi HSG huyện Lê Ninh 2018-2019)

Câu 35. Cho hai số nguyên, số thứ chia cho dư 1, số thứ hai chia cho dư Hỏi tổng bình phương chúng có chia hết cho không ?

Câu 36. Chứng minh với số n nguyên dương thì: 5 5n( n+ −1 3) (n n+2 91n)

Câu 37. Chứng minh A = + + 32 + 33 + + 311 chia hết cho 40

Câu 38. Tìm đa thức f x( )biết: f x( ) chia cho x−2dư 5; f x( )chia cho x−3dư 7; f x( )chia cho (x−2)(x−3)được thương x2 −1và đa thức dư bậc với x

Câu 39. Cho số tự nhiên n>3. Chứng minh 2n =10a+b a b( , ∈,0< <b 10)thì tích

abchia hết cho

Câu 40. Cho a b, số nguyên dương thỏa mãn 2

p=a +b số nguyên tố

p− chia hết cho Giả sử số nguyên x y, thỏa mãn ax2−by2 chia hết cho p Chứng

minh hai số x y, chia hết cho p

(Đề thi HSG lớp TP Hải Phòng 2017-2018)

Câu 41. Cho ba số nguyên dương a b c, , thỏa mãn a3+b3+c3 chia hết cho 14 Chứng

(65)

CH

IN

H

P

H

C

K

T

H

I H

C S

IN

H

GI

I C

P H

AI

(Trích đềChun tốn Sư Phạm Hà Nội 2019-2020)

Câu 42.

a) Tìm tất số tự nhiên n cho 2n+1 chia hết cho

b) Cho n số tự nhiên n>3 Chứng minh 2n+1 không chia hết cho 2m−1 với

mọi số tự nhiên m cho 2< ≤m n

(Trích đề PhổThơng khiếu Hồ Chí Minh 2019-2020)

Câu 43. Chứng minh với số nguyên dương n, số 4

9.3 8.2 2019

= nn+

M chia

hết cho 20

(Trích đề Chuyên Quảng Nam 2019-2020)

Câu 44. Có số tự nhiên n không vượt 2019 thỏa mãn n3+2019 chia hết cho (Trích đềChuyên Nam Định 2018-2019)

Câu 45. Cho x y, số nguyên cho x2−2xyy xy2; −2y2−x chia hết cho

Chứng minh 2

2x +y +2x+ycũng chia hết cho

(Trích đề Chuyên KHTN Hà Nội 2018-2019)

Câu 46.Tìm tất số nguyên không âm a b c, , thỏa mãn ( ) (2 ) (2 )2

6

ab + bc + ca = abc a3 +b3 +c3+1 chia hết cho a+ + +b c

(Trích đềChuyên Nam Định 2016-2017)

Câu 47. Cho n sô tự nhiên chẵn, chứng minh số n n n

20 −3 +16 −1chia hết cho số 323

(Trích đềChun Bình Định 2018-2019) Câu 48. Cho a2+b2 bội số với a b số nguyên Chứng minh hai số

= +

A 2a b B 2b a= − hai số A' 2a b= − B' 2b a= + chia hết cho

Câu 49 Cho phương trình x3+2y3+4z3=9!(1)với

; ;

x y zlà ẩn 9! Là tích số nguyên

dương liên tiếp từ đến

a) Chứng minh có số nguyên x y z; ; thỏa mãn (1) x y z, , chia hết

cho

b) Chứng minh không tồn số nguyên x y z, , thỏa mãn (1)

(Trích đềChuyên Vĩnh Phúc 2018-2019)

Câu 50. Cho plà số nguyên tố lớn Chứng minh p2−1chia hết cho 24

(Trích đề Chuyên Bến Tre 2018-2019)

Câu 51. Cho số tự nhiên n≥2và số nguyên tố pthỏa mãn p−1chia hết cho nđồng thời

3

1

n − chia hết cho p Chứng minh n+plà số phương

(Trích đề Chuyên Phan Bội Châu 2018-2019)

Câu 52. Với n số tự nhiên chẵn, chứng minh rằng: (20n+16n− −3n 323)

(66)

CH

UY

ÊN

Đ

S

H

C

Câu 53. Đặt N = + +a1 a2 +a2018,

5 5

1 2018

M =a +a + +a (a a1; 2; a2018∈+) Chứng

mỉnh N chia hết cho 30 M chia hết cho 30

(Trích đề Chuyên Hải Dương 2018-2019)

Câu 54. Cho a, b,c số nguyên Chứng minh 2016 2017 2018

a +b +c chia hết cho 2018 2019 2020

a +b +c chia hết cho 6

(Trích đềChun Tun Quang 2018-2019)

Câu 55. Tìm dạng tổng quát số nguyên dương n biết: M = n.4n + 3n chia hết cho

(Trích đề Chuyên Hải Dương 2016-2017)

Câu 56. Chứng minh với số tự nhiên n thì

9 27

− +

n n khơng chia hết cho 81 (Trích đềChun Quảng Ngãi 2018-2019)

Câu 57. Cho m n, số nguyên thỏa mãn 4(m+n)2−mnchia hết cho 225 Chứng minh

rằng: mncũng chia hết cho 225

(Trích đềChuyên Lào Cai 2018-2019)

Câu 58. Cho n số nguyên dương tùy ý, với số nguyên dương k đặt 1k 2k k

k

S = + + +n Chứng minh S2019S1

(Chuyên toán Thanh Hóa 2018-2019)

Câu 59. Chứng minh p (p + 2) hai số nguyên tố lớn tổng chúng chia hết cho 12

(Trích đề Chun Hịa Bình 2015-2016)

Câu 60. Chứng minh số nguyên n lớn thoả mãn n2+ n2+16 số

nguyên tố n chia hết cho 5.

(Trích đề Chuyên Phú Thọ 2015-2016)

Câu 61. Chứng minh biểu thức 3( ) (2 )( 3 )

2

S =n n+ + n+ nn+ − n− chia hết cho 120,

với n số ngun

(Trích đềChun Bình Phước 2017-2018)

Câu 62. Cho ( 2015 2015 2015)

2

A= + + +n với n số nguyên dương Chứng minh A

chia hết cho n(n + 1)

(Trích đềChuyên Quảng Nam 2015-2016)

Câu 63. Cho biểu thức

2 16 15

Q=a + aaa+ Tìm tất giá trị nguyên a để Q

chia hết cho 16

(Trích đềChuyên Quảng Nam 2016-2017)

(67)

CH

IN

H

P

H

C

K

T

H

I H

C S

IN

H

GI

I C

P H

AI

Câu 65. Cho a, b, c ba số nguyên khác thỏa 1 1= +

a b c Chứng minh rằng: abc chia hết cho

(Trích đề thi HSG lớp tỉnh Đồng Nai 2019)

Câu 66. Chứng minh

2 n 4n 16

A= + + chia hết cho với số nguyên dương n

(Trích đề thi HSG lớp tỉnh Nghệ An Bảng A 2019)

Câu 67.Chứng minh A=4n +17 chia hết cho với số nguyên dương n

(Trích đề thi HSG lớp tỉnh Nghệ An Bảng B 2019)

Câu 68. Cho *

nN Chứng minh 2n + 3n + số phương n

chia hết cho 40

(Trích đề thi HSG lớp tỉnh Thanh Hóa 2019)

Câu 69.Chứng minh với số nguyên n chẵn thì: n3+20n 96+ chia hết cho 48 (Trích đề thi HSG lớp tỉnh Bình Phước 2019)

Câu 70. Cho p số nguyên tố thỏa mãn p a b= 3− 3 với

a,b hai số nguyên dương

phân biệt Chứng minh lấy 4p chia cho loại bỏ phần dư nhận số bình phương số nguyên lẻ

(Trích đề thi HSG lớp tỉnh Khánh Hòa 2018)

Câu 71.Cho đa thức f(x) x a x b 1.= 2− ( + ) + − Xác định a, b để f(x) chia hết cho (x – 1) và

và đa thức (x + 2)

Câu 72.1 Cho p số nguyên tố lớn Chứng minh p2016 – chia hết cho 60.

2 Cho x y z, , số dương khác đôi 3

x + y +z chia hết cho x y z2 2 Tìm

thương phép chia 3 2

:

x +y +z x y z

(Trích đề thi HSG lớp tỉnh Thanh Hóa 2017)

Câu 73. Cho hai số nguyên a b thỏa 24a b 2+ = Chứng minh có số a

hoặc b chia hết cho

(Trích đề thi HSG lớp tỉnh Quảng Nam 2017)

Câu 74.Cho p q số nguyên tố lớn thỏa mãn p q 2= + Tìm số dư chia p q+ cho 12

(Trích đề thi HSG lớp tỉnh Vĩnh Long 2016)

Câu 75.Cho nguyên a, b, c, d thỏa mãn điều kiện a3+b3 =2 c 8d( 2− 3).

Chứng minh a b c d+ + + chia hết cho

(Trích đề thi HSG lớp Thành Phố Hà Nội 2016)

Câu 76 Chứng minh với số nguyên n, số 3 15

Ann chia hết cho 18

(68)

CH

UY

ÊN

Đ

S

H

C

Câu 77. Biết a b; số nguyên dương thỏa mãn a2−ab b+ chia hết cho 9, chứng minh

rằng a b chia hết cho

(Trích đề thi HSG lớp Thành Phố Hà Nội 2019)

Câu 78 Chứng minh 3 3

1 n

a +a +a + +a chia hết cho 3, biết a a a1, 2, 3, ,an chữ

số 2018

2019

(Trích đề thi HSG lớp tỉnh Hải Dương 2019)

Câu 79 Cho n số tự nhiên lẻ Chứng minh: 46n +296.13n chia hết cho 1947

(Trích đề thi HSG lớp tỉnh Bà Rịa Vũng Tàu 2019)

Câu 80 Chứng minh

2n +3n +n chia hết cho 6với số nguyên n

(Trích đề thi HSG lớp tỉnh Lâm Đồng 2019)

Câu 81. Cho a b c, , số nguyên thỏa mãn a b+ = −c3 2018c Chứng minh 3

A=a + +b c chia hết cho

(Trích đề thi HSG lớp tỉnh Quảng Ngãi 2019)

Câu 82. Chứng minh số có dạng 20142014 2014 có số chia hết cho 2013

(Trích đềvào 10 Chuyên Lạng Sơnnăm 2013-2014)

Câu 83. Cho a b, hai số nguyên dương thỏa mãn a+20 b+13 chia hết cho 21

Tìm số dư phép chia A=4a +9b + +a b cho 21

(Trích đềvào 10 Chun Hải Phịng năm 2013-2014)

Câu 84 Cho biểu thức: ( 2020 2020 2020) ( 2016 2016 2016)

A= a +b +ca +b +c với a,b,c số

nguyên dương Chứng minh A chia hết cho 30

(Trích đềvào 10 Chuyên Tin Lam Sơn năm 2019-2020)

Câu 85. Cho hai số nguyên dương x y, với x>1 thỏa mãn điều kiện: 2x2− =1 y15

Chứng minh x chia hết cho 15

(Trích đềvào 10 Chun Tốn Lam Sơn năm 2019-2020)

Câu 86. Cho số 1; 2; 3; ; 100 Viết cách tùy ý 100 số nối hàng ngang ta số tự nhiên Hỏi số tự nhiên có chia hết cho 2016 hay khơng?

(Trích đềvào 10 Chuyên Toán Lam Sơn năm 2015-2016)

Câu 87. Tìm k để tồn số tự nhiên n cho (n2−k 4) với k∈{0;1; 2; 3}.

Câu 88. Cho n số dương Chứng minh rằng: (n n 2n+ )( + ) ( ) chia hết cho 2n.

Câu 89 Tìm a b, để P x( )=3x3+ax2+bx+9 chia hết cho Q x( )=x2−9

(Đề thi học sinh giỏi huyện Chương Mỹ - Hà Nội năm 2019-2020)

Câu 90 Chứng minh rằng: ( 2019 2020)

2019 +2021 2020

(69)

CH

IN

H

P

H

C

K

T

H

I H

C S

IN

H

GI

I C

P H

AI

Câu 91. Tìm tất số nguyên dương m, n cho

m+n chia hết cho

mn n+m

chia hết cho nm

(Đề thi học sinh giỏi tỉnh Bắc Ninh năm 2017-2018)

Câu 92 Chứng minh

2

nn +n chia hết cho 36 với n nguyên dương

(Trích đề thi học sinh giỏi tỉnh Bình Định năm 2017-2018)

Câu 93 Tìm số tự nhiên có dạng ab Biết ab2−ba2 số chia hết cho 3267

(Trích đề thi học sinh giỏi tỉnh Hải Dương năm 2017-2018)

Câu 94 Với a, b số nguyên Chứng minh 2

4a + 3ab 11b− chia hết cho

thì a4 −b4 chia hết cho

(Trích đề thi học sinh giỏi tỉnh Hải Dương năm 2011-2012)

Câu 95 Tìm cặp số nguyên dương ( )x y; cho x y2 + +x y chia hết cho

2

1

xy + +y

(Trích đề thi Chuyên Phan Bội Châu năm 2019-2020)

Câu 96. Tìm tất cặp số nguyên dương  x y; cho x2 2xy2.

(Trích đề thi Chuyên Phan Bội Châu năm 2016-2017)

Câu 97. Cho a, b số nguyên dương thỏa mãn 2

a +b ab

Tính giá trị biểu thức 2

a b

A

ab

+ =

(Trích đề thi Chuyên Phan Bội Châu năm 2015-2016)

Câu 98 Giả sử a, b, c số nguyên cho 2

c b

a + + chia hết cho Chứng minh

a, b, c đồng thời chia hết cho

(Trích đề thi Chuyên Vinh – NghệAn năm 2012-2013)

Câu 99. Chứng minh (53n+2 +22n+3)11, với số tự nhiên n

(Trích đề thi Chuyên Vinh – NghệAn năm 2007-2008)

Câu 100.Cho số nguyên dương x, y thỏa mãn x x chia hết cho 2 + + xy 1− Tính giá trị

của biểu thức = + + −

2

x x

A

xy

Câu 101.Cho S tập số nguyên dương n có dạng 2

3

= +

n x y , x, y số

nguyên Chứng minh rằng: a) Nếua b, ∈S abS

b) Nếu NS N số chẵn N chia hết cho ∈ N

S

(70)

CH

UY

ÊN

Đ

S

H

C

Câu 102 Tìm tất số nguyên tố p có dạng 2

p=a +b +c với a, b, c số nguyên

dương thỏa mãn 4

a +b +c chia hết cho p

(Trích đềthi Chuyên Sư phạm Hà Nội năm 2011-2012)

Câu 103. Chứng minh với số nguyên dương n, ta ln có

( )7 ( )7 ( )7

27n 10 10n 27 5n 10 27

 + +  + + +  + + + 

     

chia hết cho 42

(Trích đề thi Chuyên KHTN Hà Nội năm 2019-2020)

Câu 104. Với x, y số nguyên thỏa mãn đẳng thức 2

2

x  y  Chứng minh

2 40

xy

(Trích đề thi Chuyên KHTN Hà Nội năm 2016-2017)

Câu 105. Tìm số ngun  x y; khơng nhỏ cho xy1 chia hết cho x1y1

(Trích đề thi Chuyên KHTN Hà Nội năm 2015-2016)

Câu 106 Có số nguyên dương có chữ số abcde cho abc10d e  chia hết cho 101?

(Trích đề thi Chuyên KHTN Hà Nội năm 2014-2015)

Câu 107 Tìm hai chữ số cuối số : A41106572012

(Trích đề thi Chuyên KHTN Hà Nội năm 2012-2013)

Câu 108 Tìm chữ số tận số 13 2009

2009

13 + +

(Trích đề thi Chuyên KHTN Hà Nội năm 2009-2010)

Câu 109 Cho m n, hai số nguyên Chứng minh rằng: 7(m+n)2+2mn chia hết cho 225

mn chia hết cho 225

(Trích đề thi Chuyên TP HồChí Minh năm 2019-2020)

Câu 110 Cho m, n số thực dương thỏa mãn 5mn m 5n Chứng minh m n(Trích đề thi Chuyên TP Hồ Chí Minh năm 2016-2017)

Câu 111. Cho x, y hai số nguyên dương thỏa mãn x2 y2 10 chia hết cho xy

a) Chứng minh x y hai số lẻ nguyên tố b) Chứng minh k x2 y2 10

xy

 

 chia hết cho k 12

(Trích đềthi Chun tốn TP HồChí Minh năm 2016-2017)

Câu 112. Cho x, y số ngun khơng đồng thời Tìm giá trị nhỏ

2

5 11

F = x + xyy

(71)

CH

IN

H

P

H

C

K

T

H

I H

C S

IN

H

GI

I C

P H

AI

Câu 113. Cho a b, số nguyên, chứng minh rằng: P=a b7 3−a b3 chia hết cho 30

Câu 114. Cho đa thức

a 5a 5a 6a 240

5

P=a − + + − + Chứng minh a số nguyên

thì P chia hết cho 120

Câu 115. Cho a b, số nguyên dương a+1,b+2007 chia hết cho Chứng

minh rằng: P=4a + +a b chia hết cho

(Vòng 1, THPT Chuyên – Đại học Quốc gia Hà Nội, năm học 2007-2008)

Câu 116 Cho P=(a b b c c+ )( + )( +a)−abc, với a b c, , số nguyên Chứng minh

nếu a b c+ + chia hết cho P chia hết cho

(Vịng 2, THPT Chuyên – TP Hà Nội, năm học 2005-2006)

Câu 117. a) Chứng minh không tồn số nguyên x y z, , cho:

2 2

560 647

x +y +z =

b) Chứng minh không tồn số nguyên a b c d, , , thoản mãn:

3 3

660 064

a + + +b c d = + + + +a b c d Câu 118. Chứng minh với số nguyên a thì:

a)

3a 53

P=a + + không chia hết cho 49

b)

5a 185

Q=a + + khơng chia hết cho 169

Câu 119 Tìm số tự nhiên n 2 2

1 n

P= + + + + khơng chia hết cho

Câu 120. Tìm số nguyên a cho:

a)

124

P=a − +a chia hết cho 121

b)

7 14

Q=aa + a− chia hết cho a2+3 Bài 121

a) Tìm m để đa thức

( ) 21

A x =xx + x + + −x m chia hết cho đa thức

2

( )

B x =x − −x

b) Tìm a b để đa thức f x( )=2x3−3bx2+2x+ −a chia hết cho x−1 x+2

Bài 122 Tìm đa thức A x( ), biết A x( ) chia cho x−5 dư 7, A x( ) chia cho x+3 dư −1 A x( )

chia cho

2 15

xx− thương 2x3+1 dư Bài 123 Cho đa thức

1880 1840 1800 20 10

( ) , ( )

P n =n +n +n Q n =n +n +

Chứng minh với nZ P n( ) chia cho Q n( ) Bài 124: Cho a số nguyên dương Chứng minh rằng:

a) P=(a+4)(a+5)(a+ + +6) (2a+5)(2a+6) chia hết cho 2a+3 b) Q=(a+1)(a+2)(a+3) (3a−1)3a chia hết cho 3a

Bài 125: Tổng hai số tự nhiên chia hết cho tổng lập phương chúng chia hết cho

(72)

CH

UY

ÊN

Đ

S

H

C

Bài 126 Chứng minh

20

m + m chia hết cho 48 với số chẵn m

(Thi học sinh giỏi TP Hồ Chí Minh 1979 – 1980 vịng 2)

Bài 127 Tìm tất số tự nhiên mà gạch bỏ chữ số số giảm 71 lần

(Thi học sinh giỏi TP Hồ Chí Minh 1982 – 1983 vịng 2)

Bài 128 Tìm số có hai chữ số; biết số chia hết cho thêm số vào chữ số cộng vào số tạo thành số hai lần chữ số hàng trăm số lớn gấp lần số phải tìm

(Thi học sinh giỏi TP Hồ Chí Minh năm học 1983 – 1984 Vòng 2)

Bài 129 Chứng minh ( 2)

3

x +y  x y chia hết cho

(Thi học sinh giỏi TP Hồ Chí Minh 1984 – 1985 vòng 2)

Bài 130 Một số gồm chữ số giống chia cho số gồm chữ số giống thương 16 số dư số r Nếu số bị chia số chia bớt chữ số thương khơng đổi số dư giảm bớt 200 Tìm số

(Thi học sinh giỏi TP Hồ Chí Minh 1986 – 1987 vòng 2)

Bài 131

a) Tìm số có ba chữ số cho tỷ số số tổng chữ số có giá trị lớn b) Chứng minh rằng:

( ) (4 )4

4

A= + −a b ab− chia hết cho 16 với số nguyên a b

4n 60

B= + + n− chia hết cho 36 với số tự nhiên n

(Thi học sinh giỏi TP Hồ Chí Minh 1987 – 1988)

Bài 132

a) Chứng minh biểu thức ( )( )

2 n 2n

A= + + nn chia hết cho 30 với số tự nhiên n

b) Chứng tỏ n=1988 số tự nhiên cho tổng chữ số S n( )

bằng ( )

1988 26

S n =nn+

c) Chứng minh hai số: 1; ( 1)

n n

A= n+ B= + hai số nguyên tố với

số tự nhiên n

(Thi học sinh giỏi TP Hồ Chí Minh 1988 – 1989 vịng - vòng 2)

Bài 133

a) Cho hai số nguyên dương a b (ab) đề không chia hết cho Chứng minh rằng:

4

ab chia hết cho

b) Cho số a a a1, 2, 3, ,an mà giá trị −1 Chứng minh

1 2 3 n

a a +a a +a a + +a a = n chia hết cho

c) Tìm tất số tự nhiên n cho: n+S n( )+S S n( ( ))=60 Trong kí hiệu S n( )

(73)

CH

IN

H

P

H

C

K

T

H

I H

C S

IN

H

GI

I C

P H

AI

(Thi học sinh giỏi TP Hồ Chí Minh 1982 – 1983 vịng 2)

Bài 134 Cho số 1997 1993

1993 1997

M = +

a) Chứng minh rằng: M chia hết cho 15

b) Hỏi M tận chữ số nào? (có giải thích)

(Thi học sinh giỏi TP Hồ Chí Minh 1992 – 1993)

Bài 135. 1) Cho biết x y, , z số nguyên cho (xy)(yz)(zx)= + +x y z Chứng

minh ta có: x+ +y z bội số 27

2) Chứng minh với k nguyên dương a số nguyên tố lớn

1

k a

chia hết cho 240

(Thi học sinh giỏi TP Hồ Chí Minh 1995 – 1996)

Bài 136.

a) Tìm tất số nguyên dương n cho n.2n+3n chia hết cho

b) Tìm tất số nguyên dương n cho n.2n+3n chia hết cho 25

(Thi vào lớp 10 toán – tin P.T.N.K Đại học quốc gia TP Hồ Chí Minh)

Bài 137.

a) Chứng minh với n nguyên dương ta có:

( ) ( )

5n 5n 6n 3n 2n

A= + − + chia hết cho 91

b) Tìm tất cặp số nguyên tố p, q thỏa mãn phương trình sau:

2

2 2

5 p +1997=5 p +q

(Thi vào lớp 10 chuyên toán – tin ĐHSP Hà Nội 1997 - 1198)

Bài 138. Tìm tất số nguyên n P 1999n= 2+1997n 30+ chia hết cho 6n

Bài 139. a) Cho hai số tự nhiên a, b cho 1995

ab=1996 Hỏi a+b có chia hết cho 1995

hay không?

b) Cho hai số tự nhiên c, d cho cd=19911992 Hỏi c+d có chia hết cho 1992 hay

không?

(Thi vào 10 chuyên tốn Hà Nội – AMSTERDAM 1991)

Bài 140. Tìm số nguyên x, y thỏa mãn 3

x +y =1995

Bài 141.Cho *

n Chứng minh rằng: 2019 2019 2019

1

n

S     n chia hết cho

1 n

T     n

Bài 142.Cho m n, số nguyên dương, giả sử  

3

2

m n

A

n

 số nguyên lẻ Tìm giá trị bé A tìm m n, thỏa mãn giá trị Chứng minh cho câu trả lời

Bài 143.Tìm số nguyên dương a, b cho 1,

1

a b b a

a b

− +

(74)

CH

UY

ÊN

Đ

S

H

C

Bài 143. Cho số tự nhiên a b c d e, , , , biết:a b c d+ + + + =e 3a=4b=5 ,c d+ =e 13 Tìm số

lớn số a b c d e, , , , .

Bài 144.Tìm tất số nguyên dương m, n cho 2

( )

m+nmn vàn+m2(n2−m)

Bài 145 Tìm số nguyên dương x, y cho 2xy−1chia hết cho (x−1)(y−1)

Bài 146 Tìm số nguyên dương x y, cho 4x +6x 32 + chia hết cho 2xy l−

Bài 147 Tìm số nguyên dương x, y cho

2

x − chia hết cho xy+2

Bài 148 Tìm số tự nhiên x, y cho 2

3

x + xy+y lũy thừa

Bài 149 Cho x, ylà số nguyên x, y≠ −1 cho

4

1

1

x y

y x

− + −

+ + số nguyên Chứng

minh: 44

1

x y − chia hết y+1

Bài 150.Xác định tất cá số nguyên tố p, q cho 1

1

n

p q

p q

+ − −

=

− − với n>1,n

Bài 151.Cho a b, số nguyên p số nguyên tố lẻ Chứng minh

p ước

của 2

a +b a a( + b)2thì p4cũng ước a a( +b)

Bài 152 Cho a b, ∈ ab thỏa ab a( +b)chia hết cho a2+ab b+ Chứng minh rằng:

3

a b− > ab

Bài 153 Cho n số nguyên dương Tìm tổng tất số chẵn nằm

1

n − +n n2+ +n

Bài 154 Cho m, n số nguyên dương, giả sử ( )

2

m n A

n

+

= số nguyên lẻ, tìm giá trị bé có A tìm m, n thỏa mãn giá trị Chứng minh cho câu trả lời

Bài 155 Tìm tất số nguyên n>1 cho với ước số nguyên tố n6−1

một ước ( )( )

1

nn

Bài 156 Tìm n để 100 0100 01

n n

M =    chia hết cho 37

Bài 157 Tìm tất số có năm chữ số abcde cho 3abcde =ab

Bài 158 Tìm chữ số a, b, c với a≥1 cho abc=(a b+ ) c

Bài 159 Tìm số có chữ số abc biết abc= + +a! b! c!

Bài 160 Cho số tự nhiên a, b Chứng minh:

a, 2

a +b chia hết cho a, b chia hết cho

b, 2

a +b chia hết cho a, b chia hết cho

c, 4

a +b chia hết cho 15 a, b chia hết cho

Bài 161 Tìm tất số nguyên dương m, n cho m n+ 2(m2−n) n m+ 2(n2−m)

Bài 162 Xét phân số

5

n A

n

+ =

+ Hỏi có số tự nhiên n khoảng từ đến

2017 cho phân số A chưa tối giản

Bài 163 Cho a, b∈ cho a b

b a

+ +

+ ∈ Chứng minh ước chung lớn

(75)

CH

IN

H

P

H

C

K

T

H

I H

C S

IN

H

GI

I C

P H

AI

A.KiÕn thøc cÇn nhí

1.Định nghĩa số nguyên tố, hợp số.

1) Số nguyên tố số tự nhiên lớn 1, có ước số Ví dụ: 2, 3, 5, 7, 11, 13, 17, 19

2) Hợp số số tự nhiên lớn có nhiều ước Ví dụ: có ước số: ; nên hợp số

3) Các số só ngun tố khơng phải hợp số 4) Bất kỳ số tự nhiên lớn có ước số ngun tố

2 Một số tính chất.

● Có vơ hạn số nguyên tố

• Nếu số nguyên tố p chia hết cho số nguyên tố q p q =

• Nếu tích abc chia hết cho số ngun tố p thừa số tích abc chia hết cho số nguyên tố p

• Nếu a b khơng chia hết cho số ngun tố p tích ab khơng chia hết cho số nguyên tố p

●Nếu A hợp số A có ước ngun tố khơng vượt q A

Chứng minh n hợp số nên n=ab với a b, ∈,1< ≤ <a b n a ước nhỏ n

Thế

na Do an

3 Phân tích số thừa số ngun tố:

•Phân tích số tự nhiên lớn thừa số nguyên tố viết số dạng tích thừa số ngun tố

+ Dạng phân tích thừa số nguyên tố số ngun tố số

+ Mọi hợp số phân tích thừa số nguyên tố, phân tích khơng tính thứ tự thừa số

Chẳng hạn A a b c , a, b, c số nguyên tố = α β γ α β, , , γ ∈N* Khi số ước số A tính (α+1)(β+1 ) (γ +1)

Tổng ước số A tính α − β+ − γ+ −

− − −

+1 1

a b. c a b c

CH

Đ

(76)

CH

UY

ÊN

Đ

S

H

C

4 Số nguyên tố nhau.

Hai số a b nguyên tố ( )a,b =1 Các số a, b, c nguyên tố (a,b,c)=1

Các số a, b, c đôi nguyên tố ( ) ( ) ( )a,b = b,c = c,a =1

5 Cách nhận biết số nguyên tố.

Cách 1

Chia số cho số nguyên tố từ nhỏ đến lớn: 2; 3; 5;

-Nếu có phép chia hết số khơng số ngun tố

-Nếu thực phép chia lúc thương số nhỏ số chia mà phép chia có số dư số số ngun tố

Cách 2

- Một số có hai ước số lớn số khơng phải số ngun tố

-Nếu A hợp số A có ước nguyên tố không vượt A

-Với quy tắt khoảng thời gian ngắn, với dấu hiệu chia hết ta nhanh chóng trả lời số có hai chữ số ngun tố hay khơng

B. MỘT SỐ DẠNG TOÁN SỐ NGUYÊN TỐ, HỢP SỐ

Dạng 1: Chứng minh số số nguyên tố hay hợp số

Bài toán 1.Nếu p p28 số nguyên tố p22 số nguyên tố Hướng dẫn giải

Xét p3k1 (k nguyên) p28 3 , hợp số Xét p3k2 p28 3 , hợp số

Vậy p3k, mà p số nguyên tố nên p3

Khi

2 11

p   , số nguyên tố

Bài toán Chứng minh

4

+

n số nguyên tố n=1

Hướng dẫn giải Ta có: 4 4 2 2 ( 2 )2 ( )2

4 4 2

+ = + + − = + −

n n n n n n

( )( ) ( )2 ( )2

2 2  1   1

= n + − n n + + n = n− +   n+ + 

Nếu n>1 hai thừa số lớn Như n4+4 số nguyên tố n=1

Bài toán 3. Chứng minh với số tự nhiên n>1thì

1

(77)

CH

IN

H

P

H

C

K

T

H

I H

C S

IN

H

GI

I C

P H

AI

Hướng dẫn giải Ta có: ( )( )

1 1

n +n + = n + +n n − +n

n>1 nên

1

n + + >n suy n5+n4+1 hợp số

Bài toán 4. Chứng minh 2n −1 số nguyên tố (n>2) 2n+1 hợp số Hướng dẫn giải

Trong ba số nguyên 2n−1; ; 2n n+1 có số chia hết cho Mặt khác, 2n không chia hết cho 3, hai số 2n −1; 2n +1 phải có số chia hết cho 3, nghĩa hai số phải có hợp số Để cho 2n−1 số nguyên tố (n>2) nên chắn

chắn 2n+1 hợp số

Bài toán 5. Cho p 8p−1 số nguyên tố Chứng minh 8p+1 hợp số

Hướng dẫn giải Vì 8p−1 số nguyên tố nên p≠2

Nếu p=3 8p+ =1 25 hợp số

Nếu p>3 8p(8p−1 8)( p+1)3 Vì p 8p−1 số nguyên tố lớn nên 8p+1 chia hết cho hay 8p+1 hợp số

Bài toán 6. Chứng minh với số nguyên dương n, chọn 2020 2019

1

n +n +

số nguyên dương liên tiếp mà tất hợp số Hướng dẫn giải Xét ( 2020 2019 )

1 ! 2

A = n +n + + 

( )

( ) ( )

2020 2019

2020 2019

2020 2019 2020 2019 2020 2019

2 ! 3

2 ! 2

n n

A n n

A + + n n n n n n

= + + +

= + + + + + + +

Dãy 1 2 2020 2019

1 , , ,

n n

A A A + + hợp số liên tiếp

Dạng 2: Chứng minh sốbài tốn có liên quan đến tính chất sốnguyên tố

Bài toán Chứng minh nếup p+2 hai số nguyên tố lớn tổng

chúng chia hết cho 12

(78)

CH

UY

ÊN

Đ

S

H

C

Ta có : p+(p+2) (=2 p+1)

• p số nguyên tố lớn nên p số nguyên tố lẻ, suy :

( )

1 2

p+  ⇒ p+  (1)

p p, +1,p+2 ba số nguyên liên tiếp nên có số chia hết cho 3, mà p p +

không chia hết :

( )

1 3

p+  ⇒ p+  (2)

Từ (1) (2) suy : 2(p+1 12.) (đpcm)

Bài toán 2.Chứng minh ước nguyên tố 2014! 1− lớn 2014

Hướng dẫn giải

Gọi p ước nguyên tố 2014! 1−

Giả sử p≤2014⇒1.2.3 2014 p⇒2014! p

Mà (2014! 1− ) p nên 1 p Điều mâu thuẫn dẫn đến p>2014

Bài toán 3. Cho số p= +bc a q, =ab+c r, = +ca b số nguyên tố (a b c, , ∈N*)

Chứng minh ba số p, q, r có hai số Hướng dẫn giải

Trong số a b c, , có hai số tính chẵn lẻ

Giả sử hai số tính chẵn lẻ a b

Suy p = bc+ a số nguyên tố chẵn nên p =

Suy a = b = Khi q = c + r = c + nên q = r

Vậy ba số p q r, , có hai số

Bài toán 4.Cho số tự nhiên n≥2và số nguyên tố p thỏa mãn p−1 chia hết cho n đồng

thời

1

n − chia hết cho p Chứng minh n+ p số phương

Hướng dẫn giải Ta có: ( ) ( )

1

n − = nn + +np; (p−1)n⇒ − ≥ ⇒ ≥ +p n p n

p≥ + ⇒n (n−1) khơng chia hết cho p

Do đó: ( )( ) ( )

1 1

nn + +npn + +np

(79)

CH

IN

H

P

H

C

K

T

H

I H

C S

IN

H

GI

I C

P H

AI

Suy ( ) ( )

1 1

n + +nkn+ ⇒kn+ ≤n + +n ( )

2

1

kn n n k n

⇔ ≤ + ⇔ ≤ +

Ta có: ( ) ( ) ( ) ( ) ( )

1 1 1

k n + + −n n kn+  kn+ ⇒  kn+k kn+

Do k ≥1nên (k−1)n+ >k

Suy (k−1)n+ ≥k kn+ ⇒ ≥ +1 k n ( )2

Từ (1) (2) suy ra:

1 1

k = + ⇒ =n p kn+ =n + +n

( )2

2 1

n p n n n

⇒ + = + + = +

Vậy n+ plà số phương

Dạng 3: Tìm số nguyên tố thỏa mãn điều kiện đó

Đối với dạng tốn tìm số ngun tố thỏa mãn điều kiện cho trước, thường sử dụng tính chất phép chia số nguyên sau để giải:

* Trong n số nguyên liên tiếp có số chia hết cho n * Mọi số nguyên tố lớn có dạng 4n±1

* Mọi số nguyên tố lớn có dạng 3n±1

* Mọi số nguyên tố lớn có dạng 6n±1

Chứng minh:

● Xét m số nguyên tố lớn

Mỗi số tự nhiên chia cho có số dư 0, 1, 2, số tự nhiên viết dạng 4n – 1; 4n ; 4n + 1; 4n +

Do m số nguyên tố lớn nên chia hết m khơng có dạng 4n 4n +

Vậy số nguyên tố lớn có dạng: 4n±1 Khơng phải số có dạng 4n±1 số nguyên tố Chẳng hạn 4 - = 15 không số nguyên tố

● Xét m số nguyên tố lớn

+) Ta thấy số nguyên tố lớn phải có dạng 3n±1 có dạng 3k

sẽ chia hết số nguyên tố

Khơng phải số có dạng 3n±1 số nguyên tố

Chẳng hạn + = 16 không số nguyên tố

+) Mỗi số tự nhiên chia cho có số dư 0, 1, 2, 3, 4, số tự nhiên viết dạng 6n – 1; 6n ; 6n + 1; 6n + ; 6n +

Do m số nguyên tố lớn nên chia hết m khơng có dạng

6n 6n; 6n + 2; 6n +

(80)

CH

UY

ÊN

Đ

S

H

C

Chẳng hạn + = 25 khơng số ngun tố

* Ví dụ minh họa:

Bài tốn 1. Tìm tất số nguyên tố p cho p + p + số nguyên tố

Hướng dẫn giải

Với p = p + = p + = số nguyên tố Với p = p + = p + = số nguyên tố

Với p > mà p số nguyên tố nên p có dạng p = 3k + p = 3k + Nếu p=3k+1 p+ =2 3k+ =3 3( k+1 3) không số nguyên tố Nếu p = 3k + thìp+ =4 3k+ =6 3( k+2 3) không số nguyên tố; Vậy với p=3 p + p + số ngun tố

Bài tốn 2. Tìm tất số nguyên tố p cho p + 2; p + 6; p + 8; p + 14 số nguyên tố Hướng dẫn giải

Trường hợp 1: p = 5k mà p nguyên tố nên p = 5, đó:

p + = 7; p + = 11; p + = 13; p + 14 = 19 số nguyên tố nên p = thỏa mãn toán

Trường hợp 2: p = 5k + 1, đó: p + 14 = 5k + 15 = 5(k + 3) có ước 1, (p + 14) nên p + 14 không số nguyên tố

Vậy với p = 5k + khơng có tồn p ngun tố thỏa mãn toán

Trường hợp 3: p = 5k + 2, đó: p + = 5k + 10 = 5(k + 2) có ước 1, (p + 10) nên p + 10 không số nguyên tố

Vậy với p = 5k + khơng có tồn p nguyên tố thỏa mãn toán

Trường hợp 4: p = 5k + 3, đó: p + = 5k + = 5(k + 1) có ước 1, (p + 2) nên p + không số nguyên tố

Vậy với p = 5k + khơng có tồn p nguyên tố thỏa mãn toán

Trường hợp 5: p = 5k + 4, đó: p + = 5k + 10 = 5(k + 2) có ước 1, (p + 6) nên p + không số nguyên tố

Vậy với p = 5k + khơng có tồn p ngun tố thỏa mãn tốn Do p = số cần tìm

Bài tốn 3. Tìm số tự nhiên n cho

3

1

n

(81)

CH

IN

H

P

H

C

K

T

H

I H

C S

IN

H

GI

I C

P H

AI

3

1

n   n   nchia cho dư (vì n chia cho dư n3 chia

hết cho dư 2) Đặt n3k1(kN) Ta có

3 3

3 2

1 (3 1) 27 27

3 (3 1)

9 9

n k k k k

k k k k k k

    

       

Để

n

số nguyên tố, phải có k1 Khi n4

3

1 64

9

n  

  , số nguyên tố

Đáp số: n4

Bài tốn 4. Tìm số nguyên tố p cho 43p1 lập phương số tự nhiên

Hướng dẫn giải Đặt

43p 1 n (nN) 43p (n 1)(n2 n 1)

Số 43p có bốn ước nguyên dương 1, 43, , 43p p nên có ba trường hợp:

a) 2 1 43

n

n n p

  



   

 Khi n2

2

43p2   2 7, loại

b) 2 43

n

n n p

  



   

 Khi n44

2

44 44 1981

p     , loại

c) 43

n n

n p

    

  

 Khi n n(  1) 42 n 6,p5 (là số nguyên tố)

Đáp số: p5

Bài toán 5. Tìm tất số nguyên tố p để pvừa tổng vừa hiệu hai số nguyên tố

Hướng dẫn giải

Giả sử tồn số nguyên tố p thỏa mãn điều kiện đề

Khi p số nguyên tố lẻ p= p1+p2 = p3−p4 với p p p p1, 2, 3, số nguyên tố

p số nguyên tố lẻ nên p p1, 2 khơng tính chẵn lẻ Nhưng có số nguyên tố giả sử p2 =2

Lại p số nguyên tố lẻ nên p p3, khơng thể tính chẵn lẻ Cũng có số

nguyên tố Do p3 > p4 nên p4 =2

Từ p= p1+ =2 p3−2 ta suy p p p, 1, ba số nguyên tố lẻ liên tiếp

Chỉ có ba số 3; 5; thỏa mãn p= = + = −5

Dạng 4: Nhận biết số nguyên tố, phân bố nguyên tốtrong tập hợp số tự nhiên

(82)

CH

UY

ÊN

Đ

S

H

C

thứ hai có 145 số ngun tố, nghìn thứ ba có 127 số nguyên tố, … Như xa theo dãy số tự nhiên, số nguyên tố thưa dần

*Vídụ minh họa:

Bài tốn 1.Nếu p≥5 2p+1 số nguyên tố 4p+1 số nguyên tố hợp số?

Hướng dẫn giải

Xét ba số tự nhiên liên tiếp: , 4p p+1, 4p+2 Để ý ba số chắn có số chia hết cho

p≥5 số nguyên tố nên p có dạng 3k+1 3k+2

+) Nếu p=3k+1 2p+ =1 6k+ =3 2( k+1)3, mâu thuẫn với giả thiết

+) Nếu p=3k+2 4p+ =1 3( k+2)+ =1 12k+ =9 4( k+3)3 hay 4p+1 hợp số

Bài tốn 2. Tìm số tự nhiên k để dãy :k+1,k+2,k+3, ,k+10 chứa nhiều số nguyên tố

Hướng dẫn giải

• Với k = 0ta có dãy 1, 2, 3, , 10 chứa số nguyên tố 2, 3, 5,

• Với k = 1ta có dãy 2, 3, 4, , 11 chứa số nguyên tố 2, 3, 5, 7, 11

• Với k = 2ta có dãy 3, 4, 5, , 12 chứa số nguyên tố 3, 5, 7, 11

• Với k≥3 dãy k+1,k+2, ,k+10 chứa số lẻ liên tiếp, số lẻ lớn nên có số chia hết cho 3, mà số chẵn dãy hiển nhiên không số nguyên tố Vậy dãy có số ngun tố

Tóm lại với k=1thì dãy k+1,k+2,k+3, ,k+10 chứa nhiều số nguyên tố

Bài toán Chứng minh 30 số tự nhiên liên tiếp lớn 5, có 22 hợp số

Hướng dẫn giải

Trong 30 số tự nhiên liên tiếp cho, có 15 số chẵn, chúng lớn nên hợp số Ta tìm 15 hợp số

Chia 15 số lẻ cịn lại thành nhóm, nhóm gồm ba số lẻ liên tiếp Trong ba số lẻ liên tiếp, tồn số chia hết cho 3, số lớn nên hợp số Có nhóm nên ta tìm thêm hợp số

Trong 30 số tự nhiên liên tiếp, tồn số chia cho 30 dư 5, số chia cho 30 dư 25, giả sử a30m5 b30n25 Các số a b hợp số (vì chia hết cho lớn 5), đồng thời không trùng với hợp số tìm (vì a b không chia hết

cho 2, không chia hết cho 3) Ta tìm thêm hợp số

Vậy có 15  5 22 (hợp số)

(83)

CH

IN

H

P

H

C

K

T

H

I H

C S

IN

H

GI

I C

P H

AI

Hướng dẫn giải

Gọi A=2.3.4 1001

Ta có: A1 = + =A 2.3.4 1001+2 2

2

1000

3 2.3.4 1001 3

1001 2.3.4 1001 1001

A A

A A

= + = +

= + =

Dãy A A1, 2, A1000 gồm 1000 hợp số liên tiếp

Vậy tồn 1000 số tự nhiên liên tiếp hợp số

Bài toán 5. (Tổng quát số 4)

Chứng minh tìm dãy số gồm n số tự nhiên liên tiếp (n > 1) khơng có số số ngun tố ?

Hướng dẫn giải Ta chọn dãy số sau:

( )

1 !

a = n+ + a12, a1 >2 nên a1 hợp số

( )

2 !

a = n+ + a23, a2 >3 nên a2 hợp số

( !) ( 1) n

a = n+ + n+ an(n+1 ,) an > +n nên an hợp số

Dãy a a a1, 2, 2, ,an gồm có n số tự nhiên liên tiếp khơng có số số ngun tố

Nhận xét: Một vấn đề đặt ra: Có khoảng lớn số tự nhiên liên tiếp hợp số Vậy đến lúc khơng cịn số ngun tố khơng? Có số nguyên tố cuối không? Từ kỉ III trước Cơng ngun, nhà tốn học cổ Hi

Lạp Ơ – clit (Euclde) chứng minh rằng: Tập số ngun tố vơ hạn

Bài tốn 6. Chứng minh khơng thể có hữu hạn số ngun tố

Hướng dẫn giải

Giả sử có hữu hạn số nguyên tố p p1, 2, ,pn pn số lớn số nguyên tố

Xét số A= p p1 pn+1 A chia cho số nguyên tố pi (1≤ ≤i n) dư (1) Mặt khác A hợp số (vì lơn số ngun tố lớn pn) A phải chia hết cho số nguyên tố đó, tức A chia hết cho số pi (1≤ ≤i n) (2), mâu thuẫn với (1)

(84)

CH

UY

ÊN

Đ

S

H

C

Dạng 5:Chứng minh có vơ sốsốngun tốdạng ax+b (với xN và ( )a b, =1)

Đây dạng tốn tương đối khó, thường giải phương pháp phản chứng.Với dạng tốn này, trình độ THCS em giải tập dạng đơn giản 3x−1 4x+3 Việc chứng tập dạng phức tạp hơn, em gặp nhiều khó khăn khơng thể dễ dàng chứng minh Chẳng hạn chứng minh vơ số số ngun tố có dạng 4a + 1; 6a + phức tạp nhiều

*Ví dụ minh họa:

Bài toán 1. Chứng minh có vơ số số ngun tố dạng 3k−1

Hướng dẫn giải

Nhận xét: Mọi số tự nhiên khơng nhỏ có dạng: ;3k k+1 3k−1 Những số có dạng 3k (với k >1) hợp số, số nguyên tố phải

có dạng 3k+1 3k−1 Xét số có dạng 3k+1: số (3k1+1) số (3k2 +1) Vì với k k1, 2∈ thì(3k1+1)(3k2 + =1) 9k k1 +3k1+3k2 + =1 3(3k k1 + +k1 k2) 1+ =3k3+1,

do tích số nguyên có dạng 3k+1 số có dạng 3k+1

Nhận xét: Mỗi số có dạng 3k−1 có ước ngun tố có dạng Thật vậy, rõ ràng n có ước dạng với thân n ước n Gọi p ước nhỏ ước Nếu p số nguyên tố nhận xét chứng minh Nếu p hợp số p phân tích thành tích thừa số nguyên tố lẻ (do p lẻ) Các thừa số khơng thể có dạng 3k+1 (vì theo chứng minh p có dạng 3k+1) Vậy thừa số nguyên tố có dạng 3k−1 Do ước p ước n nên n có ước nguyên tố dạng3k−1

Bây giờta chứng minh có vơ sốcác số ngun tố có dạng 3k−1 Giả sử có hữu hạn số nguyên tố có dạng 3k−1 p p1, 2, ,pn

Xét số N =3p p1 pn −1 N có dạng 3k−1

Theo nhận xét N có ước nguyên tố có dạng 3k−1 Nhưng từ cách xác định N N khơng chia hết cho số nguyên tố có dạng 3k −1 Điều mâu thuẫn chứng tỏ giả sử sai Vậy có vơ số số ngun tố có dạng 3k−1

Bài tốn 2. Chứng minh rằn tồn vơ số số nguyên tố có dạng 4k+3

Hướng dẫn giải

Nhận xét: Mọi số tự nhiên khơng nhỏ số ngun tố phải có dạng

(85)

CH

IN

H

P

H

C

K

T

H

I H

C S

IN

H

GI

I C

P H

AI

thì(4k1+1)(4k2+ =1) 16k k1 2+4k1+4k2+ =1 4(4k k1 2+ +k1 k2) 1+ =4k3+1, tích

những số ngun có dạng 4k+1 số có dạng 4k+1

Nhận xét: Mỗi số có dạng 4k+3 có ước nguyên tố có dạng Thật vậy, rõ ràng n có ước dạng với thân n ước n Gọi p ước nhỏ ước Nếu p số nguyên tố nhận xét chứng minh Nếu p hợp số p phân tích thành tích thừa số nguyên tố lẻ (do p lẻ) Các thừa số khơng thể có dạng 4k+1 (vì theo chứng minh p có dạng 4k+1) Vậy thừa số ngun tố có dạng 4k+3 Do ước p ước n nên n có ước nguyên tố dạng4k+3

Bây giờta chứng minh có vơ sốcác số nguyên tố có dạng 4k+3 Giả sử có hữu hạn số nguyên tố có dạng 4k+3 p p1, 2, ,pn

Xét số N =4p p1 pn −1 N có dạng 4k+3 Theo nhận xét N có ước ngun tố có dạng 4k+3 Nhưng từ cách xác định N N không chia hết cho số nguyên tố có dạng 4k+3 Điều mâu thuẫn chứng tỏ giả sử sai Vậy có vơ số số nguyên tố có dạng 4k+3

Dạng 6: Sửdụng nguyên lý Dirichlet toán sốnguyên tố

Bài toán 1. Cho p>5 số nguyên tố Chứng minh tồn số có dạng 111 11

chia hết cho p

Hướng dẫn giải Ta xét dãy số: 1,11,111, ,111

p

 

Nếu dãy khơng có số chia hết cho p ta cho tương ứng số với số dư

của phép chia Tập hợp số dư có 1, 2, 3,,p−1 gồm p−1 phần tử (vì khơng thể có tập này)

Nhưng cóp số dạng nên theo nguyên lý Dirichlet tồn hai số có

số dư Giả sử số là: 111 m

 111 n

 với m>n

Khi 1≤ < ≤n m p.Như vậy: 111 111 111 1000 111 1.10n

m n m nn m n

− = =

    

   

Tích chia hết cho p (p,10)=1 suy 111 m n

 chia hết cho p nằm dãy

trên Mà 1≤ − ≤m n p mâu thuẫn với giả thiết khơng có số dãy chia hết cho p

Bài toán 2. Chứng minh 12 số nguyên tố phân biệt chọn số ký

hiệu p1, p2, p3, p4, p5, p6 cho (p1−p2)(p4− p3)(p5+p6)1800

(86)

CH

UY

ÊN

Đ

S

H

C

Vì ba số nguyên tố 2, 3, nên 12 số nguyên tố phân biệt cho ln có số lớn Vì số nguyên tố lớn nên: số chia cho có số dư Theo nguyên lý Dirichlet phải tồn số chia cho có số dư Mà số lại không chia hết cho 5, số có số mà ta giả sử p p1, 2 cho (p1−p2)5 Ngoài hiển nhiên

ta có (p1−p2)3 dẫn đến (p1−p2)15

Xét số cịn lại theo nguyên lý Dirichlet tồn số có số dư chia hết cho Đem số chia cho cho hai khả xảy ra:

Nếu có số (chẳng hạn p p3, 4)mà (p3−p4)5 Rõ ràng (p3−p4)2 (p3−p4)3

Vì (5;3; 2)=1 nên ta có (p3−p4)30 Lấy hai số p p5, (ngồi p p p p1, 2, 3, 4)

đã chọn p p5, lẻ (do số nguyên tố khác 2) nên (p5+p6)2 Từ suy (p1−p2)(p4− p3)(p5+p6)30.30.2=1800

Nếu số chia cho có số dư khác 1; 2;3; Giả sử (p5−1 5) ,

(p6−4 5) (p5+ p5−5 5) hay (p5+p6)5

Với số cịn lại p p3, 4 rõ ràng (p3−p4)3 (theo cách chọn số trên)

Do p p p p3; 4; 5; lẻ nên (p5+p6) (2, p3−p4)2 Từ suy (p5+ p6)10 (p3−p4)6

Do (p1−p2)(p4−p3)(p5+ p6)30.10.6=1800

Vậy tồn p1, p2, p3, p4, p5, p6 số nguyên tố phân biên cho (p1−p2)(p4−p3)(p5+ p6)1800

Dạng 7: Áp dụng định lý Fermat

Cho p số nguyên tố a số nguyên cho (a p, )=1 Khi đó:

1 p

a − ≡ (mod p)

Chứng minh

Xét số a, 2a, …, (p−1)a Dễ thấy, khơng có số p−1 số chia hết

cho p hai số có số dư chia cho p Vậy chia p−1 số nói

cho p, ta nhận số dư 1, 2, …, p−1 Suy a 2( ) ( ) (a 3a ( p−1)a)≡1.2.3.(p−1) (mod p) hay (1.2.3 (p−1 ))ap−1≡1.2.3 (p−1) (mod p)

Vì (1.2.3 (p−1 ,) p)=1 nên

1 p

a − = (mod p)

*Ví dụ minh họa:

Bài tốn 1. Tìm số nguyên tố p cho 2p +1 chia hết cho p

(87)

CH

IN

H

P

H

C

K

T

H

I H

C S

IN

H

GI

I C

P H

AI

Giả sử p số nguyên tố thỏa mãn 2p +1p

Theo Định lí Fermat: 2p mod( ) 2p (2p 1) (2p 2)

p p p p

≡ ⇒ −  ⇒ = + − −  ⇒ =

Với p=3 ta có 2p+ =1 3

Bài tốn 2. Cho p số nguyên tố lớn Chứng minh có vơ số số tự nhiên thỏa

.2n

n − chia hết cho p

Hướng dẫn giải

Ta có 2p−1≡1(mod p), ta tìm n m p= ( −1) cho n.2n≡1 (mod p) Ta có: n.2n =m p( −1).2m p( 1)− ≡m p( −1)(mod p) ⇒n.2n ≡ − ≡m 1(modp)

⇒ =m kp−1(k∈*).

Vậy, với n kp=( −1)(p−1) (k∈*) n.2 12− p

Bài toán 3. Cho p số nguyên tố, chứng ming số 1p− chỉ có ước nguyên tố có dạng

+ 2pk

Hướng dẫn giải

Gọi q ước nguyên tố 1p − q lẻ, nên theo Định lí Fermat:

−1−  ⇒ − −1− = ( , 1)− −  ⇒ − 

2q 1q (2 1,2p q 1) 2p q 1q q p, (q−1, ) 1p = 1q, vơ lí Mặt khác, q−1 chẵn suy q−1 2 p q⇒ =2pk+1

C. BÀI TẬP ÁP DỤNG

Bài 1. Tìm số nguyên tố p cho số sau số nguyên tố:

a) p + p + 10 b) p + 10 p + 20

Bài 2. Chứng minh n n2 + số nguyên tố

2

n + số nguyên tố

Bài 3. Chứng minh a, a + k, a + 2k ( a k, ∈N* ) số nguyên tố lớn k

chia hết cho

Bài 4. Chứng minh p số nguyên tố lớn (p - 1)(p + 1) chia hết cho 24

Bài 5. Một số nguyên tố p chia cho 42 có dư hợp số r Tìm r

Bài 6. Một số nguyên tố p chia cho 30 có số dư r Tìm r biết r khơng số nguyên

tố

Bài 7. Chứng minh số 11 1211 1 

n n

hợp số với n≥1

Bài 8. Tìm n số cho 10101 0101 (n chữ số n + chữ số xen kẽ nhau) số nguyên tố

(88)

CH

UY

ÊN

Đ

S

H

C

a) A = 11 1(2001 chữ số 1); b) B = 11 (2000 chữ số 1);

c) C = 1010101; d) D = 1112111;

e) E = 1! + 2! + 3! + +100!; g) G = - 28;

h) H= 311141111

Bài 10. Cho nN*,chứng minh số sau hợp số:

a) 22 n 3;

A= + +

b) B =

2

2 n+ +7;

c) C = 26

2 n+ +13

Bài 11. Cho p số nguyên tố lớn 5, chứng minh p4 ≡1 (mod 240)

Bài 12. Chứng minh dãy an =10n+3 có vơ số hợp số

Bài 13. Chứng minh với số ngun tố p có vơ số dạng 2nn chia hết cho p

Bài 14. Tìm nN* để

1

nn + −n số nguyên tố

Bài 15. Tìm số x y, ∈N* cho x4+4y4 số nguyên tố

Bài 16. Tìm tất số ngun tố p có dạng ( 1)( 2)

6

n n+ n+ + (

1

n≥ ) Bài 17. Cho nN*, chứng minh A=n4+4n hợp số với n >

Bài 18. Giải phương trình nghiệm nguyên

4(ax x)( − + − =b) b a y (1)

trong a, b số nguyên cho trước a > b

Bài 19. Cho tập hợp A gồm 16 số nguyên dương Hãy tìm số nguyên dương k

nhỏ có tính chất: Trong tập gồm k phần tử Ađều tồn hai số phân biệt

a, b cho 2

a +b số nguyên tố

(Trích đề thi học sinh giỏi lớp tỉnh Bắc Ninh 2017-2018)

Bài 20. Chứng minh a a, +m a, +2m số nguyên tố lớn m chia hết

cho

Bài 21. Cho tập A={6;12;18; 24} Tìm số nguyên tố p cho p cộng với phần tử

A nguyên tố

Bài 22. Tìm số nguyên tố p cho p4+2 số nguyên tố

(Vòng 2, THPT Chuyên – Đại học Quốc gia Hà Nội, năm học 2007 – 2008)

Bài 23.Cho biểu thức 4

4;

A=x + B=x + +x Tìm số tự nhiên x để A B

các số nguyên tố

Bài 24. Giả sử phương trình

1

x +ax+ + =b có hai nghiệm nguyên dương Chứng minh

rằng 2

(89)

CH

IN

H

P

H

C

K

T

H

I H

C S

IN

H

GI

I C

P H

AI

Bài 25. Giải phương trình

0

xmx+ =n biết phương trình có hai nghiệm nguyên dương

phân biệt m, n số nguyên tố

Bài 26.(Trích đềvào 10 Chuyên Vinh năm học 2013-2014)

Giả sử n số nguyên tố lớn Chứng minh 2013

n +

số nguyên dương

Bài 27.(Trích đề thi học sinh giỏilớp Ninh Bình năm học 2018-2019)

Tìm tất ba số nguyên tố (p;q; r) cho pqr= + + +p q r 160

Bài 28.(Trích đề thi học sinh giỏi lớp Bắc Ninh năm học 2018-2019)

Tìm số nguyên tố p thỏa mãn p 4p 93− + số phương

Bài 29.(Trích đề thi học sinh giỏi lớp Phú Yên năm học 2018-2019)

Tìm hai số nguyên tố p, q cho

8q+ =1 p

Bài 30.(Trích đề thi học sinh giỏi lớp Thái Bình năm học 2018-2019)

Tìm tất số nguyên dương (x y z; ; )sao cho 2019

2019

+ +

x y

y z số hữu tỉ

2+ +

x y z số nguyên tố

Bài 31 (Trích đề thi học sinh giỏi lớp Quảng Nam năm học 2018-2019)

Cho số nguyên tố p p( >3) p hai số nguyên dươnga b, cho p2 +a2 =b2

Bài 32 (Trích đề thi học sinh giỏi lớp Thanh Hóa năm học 2017-2018)

Cho a b, số nguyên dương thỏa mãn p a= 2+b2 số nguyên tố p 5− chia

hết cho Giả sử x y, số nguyên thỏa mãn ax2−by2 chia hết cho p Chứng

minh hai số x, y chia hết cho p

Bài 33 (Trích đề thi học sinh giỏi lớp Thanh Hóa năm học 2016-2017)

Cho p số nguyên tố lớn Chứng minh p2016– chia hết cho 60 Bài 34 (Trích đề thi học sinh giỏi lớp Nghệ An năm học 2016-2017)

Tìm tất số nguyên tố khác m, n, p, q thỏa mãn

1 1 1 1.

m n p q mnpq+ + + + =

Bài 35 (Trích đề thi học sinh giỏi lớp TP Hà Nộinăm học2014-2015) Cho n nguyên dương Chứng minh 3

2 + −

= n + n +

A

hợp số

Bài 36 (Trích đề thi học sinh giỏi lớp Vĩnh Long năm học 2015-2016)

Cho p q số nguyên tố lớn thỏa mãn p q 2= + Tìm số dư chia p q+ cho 12

Bài 37 (Thi vào lớp 10 chuyên Lê Hồng Phong năm 1981)

Chứng minh p

2

(90)

CH

UY

ÊN

Đ

S

H

C

Bài 38.(Trích đề thi học sinh giỏi lớp Nghệ An năm học 2014-2015)

Tìm số tự nhiên n cho số 2015 viết thành tổng n hợp số viết thành tổng n+1hợp số

Bài 39.(Trích đề thi học sinh giỏi lớp Thanh Hóa năm học 2014-2015)

Tìm tất số nguyên tố p, q cho tồn số tự nhiên m thỏa mãn: pq m 1.2 p q m

+ =

+ +

Bài 40.(Trích đề thi học sinh giỏi lớp Hải Dương năm học 2014-2015)

Tìm số nguyên tố p cho số 2p 1; 2p2− 2+3; 3p2+4 số nguyên tố

Bài 41.(Trích đề thi học sinh giỏi lớp Cẩm Thủynăm học 2011-2012)

Tìm số tự nhiên n để A n= 2012+n2002+1 số nguyên tố Bài 42.(Trích đề thi học sinh giỏi lớp Tiền Hảinăm học 2016-2017)

Tìm tất số nguyên dương a, b, c thỏa mãn: a b

b c

− số hữu tỉ

2 2

a +b +c số nguyên tố

Bài 43.(Trích đề thi học sinh giỏi lớp Gia Lộcnăm học 2015-2016)

Tìm số nguyên tố k để k 42+ k 162+ đồng thời số nguyên tố Bài 44.(Trích đề thi học sinh giỏi lớp Lục Namnăm học 2018-2019)

Cho p số nguyên tố lớn Chứng minh p20−1 chia hết cho 100

Bài 45 Giả sử a, b số tự nhiên cho

4

b a b p

a b − =

+ số nguyên tố Tìm giá trị lớn p

Bài 46.(Trích đề chọnhọc sinh giỏi lớp Amsterdamnăm học 2018-2019)

Tìm tất ba số nguyên dương (p;q; n), p, q số nguyên tố

thỏa mãn: p p q q 3( + ) (+ + ) (=n n 3+ )

Bài 47.(Trích đềvào 10Chun tốn Hải Phịng năm học 2019-2020)

Tìm số nguyên tố p, q thoả mãn đồng thời hai điều kiện sau: i)

p q+pchia hết cho p2+q

ii)

pq +qchia hết cho q2− p

Bài 48.(Trích đềvào 10Chun tốn Quảng Bình năm học 2019-2020)

Cho abc số nguyên tố Chứng minh phương trình

0

ax +bx+ =c khơng có

nghiệm hữu tỉ

Bài 49.(Trích đềthi HSG TP Hà Nội năm học 2013-2014)

Tìm số tự nhiên n để

25n − +n −12 số nguyên tố

(91)

CH

IN

H

P

H

C

K

T

H

I H

C S

IN

H

GI

I C

P H

AI

Cho dãy số tự nhiên 2; 6; 30; 210; xác định sau: số hạng thứ k tích k số nguyên tố đầu tiên(k=1; 2;3; ) Biết có hai số hạng dãy số có hiệu 30000 Tìm hai số hạng

Bài 51.(Vịng , THPT chun Đại học Vinh, năm học 2009 - 2010)

Chứng minh với số nguyên tố lẻ p không tồn số nguyên dương m n,

thỏa mãn : 12 12

p = m +n

Bài 52.(Trích đềvào 10Chuyên Quảng Nam năm học 2018-2019)

Tìm hai số nguyên tố p q, biết p+q p+4q số phương

Bài 53.(Trích đềvào 10Chuyên Hải Dương năm học 2018-2019)

Tìm tất số tự nhiên n k, để n8+42k+1là số nguyên tố

Bài 54.(Trích đềvào 10Chuyên Vĩnh Long năm học 2018-2019)

Tìm số tự nhiên x thỏa mãn biểu thức

14 49

= − + + +

P x x x số nguyên tố

Bài 55.(Trích đềvào 10Chuyên Phú Thọ năm học 2015-2016)

Chứng minh số nguyên n lớn thoả mãn n24 n216 số nguyên tố n chia hết cho

Bài 56.(Trích đềvào 10Chuyên Amsterdam năm học 2014-2015)

1) Cho số nguyên dương n thỏa mãn n 10 hai số nguyên tố Chứng minh

(n −1) 40

2) Tìm tất số nguyên tố p số nguyên dương x,y thỏa mãn 2 ( 2)

1 ( 2)

p x x

p y y

− = +

 − = +

Bài 57 (Trích đềvào 10Chuyên TP Hồ Chí Minh năm học 2014-2015)

Cho số nguyên dương a, b, c cho 1

a+ =b c

a) Chứng minh a + b số nguyên tố

b) Chứng minh c > a + c b + c đồng thời số nguyên tố

Bài 58. (Trích đềvào 10Chuyên Thái Bình năm học 2014-2015)

Cho a, b, c, d số nguyên dương thỏa mãn: 2 2

a +ab+b =c +cd +d Chứng

minh a + b + c + d hợp số

Bài 59. (Trích đềHSG lớp Gia Viễn năm học 2014-2015)

Tìm số tự nhiên n để plà số nguyên tố biết: p=n3 −n2 + −n

Bài 60. (Trích đềHSG lớp Thanh Chương năm học 2012-2013)

Chứng minh ∀ ∈n *thì

2

(92)

CH

UY

ÊN

Đ

S

H

C

Cho a b c, , số nguyên khác 0, ac cho

2

2

a b a

c

b c

+ =

+ Chứng minh

rằng 2

a +b +c số nguyên tố

Bài 62. (Trích đềHSG lớp Trực Ninhnăm học 2017-2018)

Cho pvà 2p+1là số nguyên tố lớn Chứng minh 4p+1là hợp số

Bài 63. Cho số nguyên tố p>3 Biết có số tự nhiên n cho cách viết thập

phân số n

p có 20 chữ số Chứng minh 20 chữ số có chữ

số giống

Bài 64. (Trích đềvào 10 Chun Vinh năm học 2015-2016) Tìm số nguyên tố p q, thỏa mãn p q+ = 2(p q− )2 Bài 65. (Trích đềHSG lớp Hoằng Hóa 2018-2019)

Tìm tất số nguyên tố p q, cho 7p+q pq+11 số nguyên tố Bài 66. (Trích đềHSG lớp Sông Lô 2018-2019)

Biết abcd nguyên tố có bốn chữ số thỏa mãn ab cd; số nguyên tố

và = + −

b cd b c Hãy tìm abcd

Bài 67. (Trích đềChuyên Khoa học tự nhiên Hà Nội năm 2009-2010)

Tìm số nguyên dương n cho tất số

n + 1, n + 5, n + 7, n + 13, n + 17, n + 25, n + 37 nguyên tố

Bài 68. (Trích đềHSG lớp Gia Bình 2018-2019)

Giả sử p p2+2 số nguyên tố Chứng tỏ p3+ p2+1 số nguyên tố

Bài 69. (Trích đềHSG lớp Nghĩa Đàn 2018-2019)

Tìm hai số nguyên tố x y, thỏa mãn x2−y2 =45

Bài 70. (Trích đềHSG lớp Như Thanh 2018-2019)

1) Chứng minh hai số 2n+1 10n+7 hai số nguyên tố với số tự nhiên n

2) Tìm số x, y nguyên tố để

23

x + = y

Bài 71. (Trích đềHSG lớp Nơng Cống 2018-2019)

Tìm số nguyên tố ab a( > >b 0), biết ab ba− số phương

Bài 72. Tìm tất số nguyên tố p để 4p 12 + 6p 12+ số nguyên tố Bài 73. Giả sử p q số nguyên tố thỏa mãn đẳng thức p p  1 q q 1.

a) Chứng minh tồn số nguyên dương k cho p 1 kq q, 2 1 kp.

b) Tìm tất số nguyên tố p, q thỏa mãn đẳng thức p p  1 q q 1.

(93)

CH

IN

H

P

H

C

K

T

H

I H

C S

IN

H

GI

I C

P H

AI

Bài 74. Cho p, q, r, s số nguyên tố lớn Chứng minh p q r s chia 2− 2+ −2

hết cho 24

Bài 75. Tìm tất cặp số nguyên tố ( )p;q cho p 2q2 − =1.

Bài 76. Chứng minh với số nguyên tố p p3+p 1−

2 khơng phải tích hai

số tự nhiên liên tiếp

Bài 77. Tìm số nguyên tố p, q, r thỏa mãn p qq+ p =r

Bài 78. Tìm số nguyên tố p,q,r thỏa mãn điều kiện sau:

≤ < < ≤ 2− 2− ≤2

5 p q r; 49 2p r ; 2q r 193

Bài 79. Tìm tất ba số nguyên tố a,b,c đôi khác thoả mãn điều kiện

( )

< + + <

20abc 30 ab bc ca 21abc

Bài 80. Tìm số nguyến tố p, q số nguyên x thỏa mãn x px 3q 5+ + =

Bài 81.Tìm số nguyên tố p để p 1+

2 +

2

p

2 số phương

Bài 82. Chứng minh tồn số nguyên dương x thỏa mãn (x 2x 1+ )( + )

2012 số phương x hợp số

Bài 83. Tìm tất số nguyên tố p cho − −

2

p p

2 lập phương số tự

nhiên

Bài 84. Cho bảy số nguyên tố khác a,b,c,a b c,a b c,a c b,b c a + + + − + − + −

hai ba số a, b, c có tổng 800 Gọi d hiệu số lớn số nhỏ bảy số nguyên tố Hỏi giá trị lớn d nhận

Bài 85. Cho p số nguyên tố Tìm tất số nguyên k cho k2−pk số nguyên

dương

Bài 86. Tìm số nguyên dương n lớn cho số 2016 viết thành

+ + + +

1 n

a a a a số a ;a ;a ; ;a1 2 3 n hợp số Kết thay đổi thay số 2016 số 2017

Bài 87. Tìm tất số nguyên tố p, q, r thỏa mãn phương trình (p q r 3+ )( + )( + )=4pqr

Bài 88 Cho số tự nhiên n 2≥ , xét số a ;a ; ;a1 2 n số nguyên tố phân biệt

1 n

p ; p ; ; p thỏa mãn điều kiện p a a1 1− 2 =p a a2 2− 3 = = p an n−a1 Chứng minh

= = =

1 n

a a a

(94)

CH

UY

ÊN

Đ

S

H

C

Bài 90.Tìm tất số nguyên tố p cho với số ngun tố p ln tồn số

nguyên dương n, x, y thỏa mãn pn =x3+y3

Bài 91 Tìm tất số nguyên dương n cho phần nguyên n 8n 13+ 2+

3n số nguyên tố

Bài 92 Cho p số nguyên tố cho

2

x py A

xy +

= số tự nhiên Khi A= +p

Bài 93 Tìm tất số nguyên tố p q thỏa mãn p q3− =(p q+ )2.

Bài 94. Cho a, b số nguyên p số nguyên tố lẻ Chứng minh p4 ước

của a2+b 2 a a b( + )2 p4 ước a a b( + ).

Bài 95. Tìm số ngun khơng âm a,b cho a2−b 5a 3b số nguyên tố 2− + +

Bài 96.Cho đa thức f x( )= ax3+bx cx d2+ + với a số nguyên dương Biết ( ) ( )=

f – f 2012 Chứng minh f – f 2( ) ( ) hợp số

Bài 97. Cho đa thức bậc ba f(x) với hệ số x3 số nguyên dương biết

− =

f(5) f(3) 2010 Chứng minh f(7) f(1)− hợp số

Bài 98. Tìm tất ba số nguyên dương (m; p;q) cho p, q số nguyên tố

+ =

m

2 p q

Bài 99.Tìm sáu số nguyên tố p ; p ; p ; p ; p ; p1 2 3 4 5 6 thỏa mãn 2+ 2+ 2+ 3+ =

1

p p p p p p

Bài 100 Cho số nguyên tố p dạng 4k Tồn hay không số nguyên a thỏa điều +

kiện (a p2+ )

Bài 101.(Trích đề thi HSG thành phố Hà Nội năm 2019-2020)

Tìm tất ba số nguyên dương (x y p, , ) với p số nguyên tố thỏa mãn

( )

2 2

6

x + p y = x+ p

Bài 102 Cho a b c, , số nguyên dương Chứng minh a b+ +2 ab c+

số nguyên tố

(Tuyển sinh vào lớp 10 chuyên TP Hà Nội, 2017).

Bài 103

a) Chứng minh với số nguyên tố khác khác có dạng: 6m+1 6m−1

b) Chứng minh có vơ số số ngun tố có dạng 6m−1

(Thi học sinh giỏi quốc gia1991 – 1992)

Bài 104 Tìm số nguyên tố x y z, , thỏa y

x + =z

Bài 105 Chứng minh 2+ +n (n nN*) số nguyên tố n=3k với kN

Bài 106 Cho a b c d, , , ∈N* thỏa mãn ab=cd Chứng minh rằng: n n n n

(95)

CH

IN

H

P

H

C

K

T

H

I H

C S

IN

H

GI

I C

P H

AI

Bài 107 Tìm tất số nguyên tố p dạng ( 1)

2

n n+

− (n≥1)

Bài 108. Tìm tất số có hai chữ số ab cho ab

ab số nguyên tố

Bài 109. a) Cho 2k +1 số nguyên tố (gọi nguyên tố Fermat) Chứng minh k =0

hoặc k =2 n

b) Cho 2k −1 số nguyên tố (gọi số nguyên tố Mersenne) Chứng minh k số

nguyên tố

Bài 110 (Thi học sinh giỏi TP Hồ Chí Minh 1995 – 1996)

1) Cho biết x y, , z số nguyên cho (xy)(yz)(zx)= + +x y z Chứng minh

rằng ta có: x+ +y z bội số 27

2) Chứng minh với k nguyên dương a số nguyên tố lớn

1

k

a − chia hết

cho 240

Bài 111. Chứng minh rằng: (p – 1)! chia hết cho p p hợp số, không chia hết cho p

nếu p số nguyên tố

Bài 112 Chứng minh rằng: ước nguyên tố 1994! – lớn 1994

Bài 113 Chứng minh rằng: n > n n! có số ngun tố (từ suy có

vơ số số ngun tố)

Bài 114 Giả sử p số nguyên tố lẻ

8 p

m= − Chứng minh m hợp số lẻ không

chia hết cho

3m− ≡1 (mod m)

Bài 115. Chứng minh dãy số 2003 23+ k với k=1, 2, chứa vô hạn số lũy thừa

cùng số nguyên tố

Bài 116. Tìm bảy số nguyên tố cho tích chúng tổng lũy thừa bậc sáu

bảy số

Bài 117. Tìm tất số nguyên tố p có dạng 2

p=a +b +c với a, b, c số nguyên

dương thỏa mãn 4

a +b +c chia hết cho p

(Trích đềtốn 10 chun sư phạm Hà Nội năm 2011-2012)

Bài 118 Tìm tất cặp số nguyên dương (x y, ) cho

2

x y

x y

+

− số nguyên dương

là ước số 1995

Bài 119. Một xí nghiệp điện tử ngày giao cho cửa hàng số máy tivi

Số máy số có ba chữ số mà tăng chữ số đầu lên n lần, giảm chữ số thứ hai thứ ba n lần số lớn gấp n lần số máy giao Tìm n số máy tivi giao

(96)

CH

UY

ÊN

Đ

S

H

C

Bài 121. Cho n số nguyên dương Chứng minh tồn n số tự nhiên liên tiếp

sao cho chúng hợp số

Bài 122. Cho số nguyên dương n thỏa mãn 2n−1 số nguyên tố Chứng minh n

số nguyên tố

Bài 123. Tìm số nguyên tố p để 2p +p2 số nguyên tố

Bài 124. Cho p q, số nguyên tố phương trình x2− px+ =q có nghiệm nguyên

dương Tìm p q

Bài 125 Cho p q r, , số nguyên tố n số tự nhiên thỏa n n

p +q =r Chứng

minh n=1

Bài 126. Cho p số nguyên tố dạng 4k+3 Chứng minh x2+y2 chia hết cho p

x y chia hết cho p

Bài 127. Tìm số tự nhiên m n, cho x=33m2+ −6n 61+4 số nguyên tố

Bài 128. Tìm tất số tự nhiên a,b,c cho 3

3

a +b + −c abc số nguyên tố

Bài 129.(Trích đề thi HSG quận Thanh Xuân năm 2019-2020)

Chứng minh rằng, p 8p2+1 hai số nguyên tố lẻ 8p2+2p+1 số nguyên tố

Bài 130 Tìm số nguyên tố a b c, , số nguyên dương k cho 2 2

16

a +b + c = k +

Bài 131 Tìm số nguyên tố p q cho p2|q3+1 q2|p6−1

Bài 132 Ta gọi p, qlà hai số nguyên tố liên tiếp, p q khơng có số ngun tố

nào khác Tìm ba số nguyên tố liên tiếp p, q, r cho p q2+ 2+r2 số nguyên tố

Bài 133 Cho số 512525

A= −

− Chứng minh A hợp số

Bài 134 Chopp+2 số nguyên tố (p>3 ) Chứng minh rằngp+1 6

Bài 135 Cho p p+4là số nguyên tố (p>3) Chứng minh p+8 hợp số

Bài 136 (Chuyên Vũng Tàu 2016-2017)

Tìm cặp số nguyên tố (p q, ) thỏa mãn 2

5

pq =

Bài 137. Chứng minh 12 số nguyên tố phân biệt chọn số ký

hiệu p1, p2, p3, p4, p5, p6 cho (p1−p2)(p4− p3)(p5+p6)1800

Bài 138.(Đềthi HSG Toán TP.HCM năm học 2004 – 2005)

Tìm tất số nguyên dương n cho phần nguyên

3

8

3

n n

n

+ + là số nguyên tố

Bài 139. Cho p q, hai số nguyên tố cho p> >q p− =q Chứng minh rằng:

(p+q)12

Bài 140. Tìm số nguyên tố p cho p+10 p+14 số nguyên tố

Bài 141. Cho p số nguyên tố lớn Chứng minh

(97)

CH

IN

H

P

H

C

K

T

H

I H

C S

IN

H

GI

I C

P H

AI

(Đề tuyển sinh Chuyên Toán Amsterdam 2017).

Bài 142 Tìm ba số nguyên tố p, q, r cho pq+qp =r

Bài 143 a) Chứng minh số dư phép chia số nguyên tố cho 30 là số nguyên tố Khi chia cho 60 kết sao?

b) Chứng minh tổng n lũy thừa bậc số nguyên tố lớn số nguyên tố ( ,30) 1n =

Bài 144 Tìm tất ba số nguyên tố a, b, c cho abc ab bc ca< + +

Bài 145. Cho dãy số nguyên dương a a1, , ,2 an xác định sau: a1=2, an ước

nguyên tố lớn a a a1 2 n−1+1 với n≥2 Chứng minh ak ≠5 với k

Bài 146.Tìm tất số nguyên tố pđể 2p+p2cũng số nguyên tố

Bài 147.Tìm n∈* để: n2003+n2002+1 số nguyên tố

Bài 148 a) Tìm số nguyên tố pđể 2p+1 lập phương số tự nhiên

b) Tìm số nguyên tố pđể 13p+1 lập phương số tự nhiên

Bài 149.Cho a b c d, , , ∈Nthỏa mãn a > b > c > d ac+bd =(b+ + −d a c)(b+ − +d a c)

Chứng minh ab+cd hợp số

Bài 150 Cho số nguyên dương a b c d, , , thỏa mãn: a2 +b2 =c2 +d2 Chứng minh a+ + +b c d hợp số

(Trích đềthi HSG lớp NghệAn 2014-2015)

Bài 151 Chứng minh 1 2+ +n 4 (n n∈*) số nguyên tố n=3k với k∈

Bài 152 Tìm số tự nhiên n cho số 2015 viết thành tổng n hợp số

không thể viết thành tổng n + hợp số

(Trích đềthi HSG lớp NghệAn 2014-2015)

Bài 153 Tìm tất số nguyên tố p dạng ( +1) ( 1).− ≥

2

n n n

Bài 154 Chứng minh số 22

A + 31

n+

= hợp số với số tự nhiên n

(Trích đềthi HSG lớp Nghệ An 2017-2018)

Bài 155. Cho n∈*, chứng minh 10

2

2 n+ +19 234n+1 +324n+1 +5 hợp số

Bài 156 Chứng minh không tồn số nguyên dương m n p, , với p nguyên tố

thỏa mãn: m2019+n2019 = p2018

(Trích đềthi HSG lớp TP Hà Nội 2017-2018)

Bài 157. Tìm số tự nhiên n cho n+3 số nguyên tố A=2n+7 lập phương

một số tự nhiên

Bài 158 Chứng minh b số nguyên tố lớn số

3 2009

A= n+ + b

hợp số, với số tự nhiên n

(98)

CH

UY

ÊN

Đ

S

H

C

A.KiÕn thøc cÇn nhí 1 Định nghĩa số phương.

Số phương số bình phương số nguyên (tức n số phương thì: = ( ∈ )

n k k Z )

2 Một số tính chất cần nhớ

1- Số phương chỉcó thể có chữ số tận 0, 1, 4, 5, 6, 9; có chữ tận 2, 3, 7,

2- Khi phân tích thừa số nguyên tố, số phương chứa thừa số nguyên tố với số mũ chẵn

3- Số phương có hai dạng 4n 4n + Khơng có số phương có dạng 4n + 4n + (n ∈ N)

4- Số phương có hai dạng 3n 3n + Không có số phương có dạng 3n + ( n ∈ N )

5- Số phương tận 1, chữ số hàng chục chữ số chẵn Số phương tận chữ số hàng chục

Số phương tận chữ số hàng chục chữ số lẻ 6- Số phương chia hết cho chia hết cho

Số phương chia hết cho chia hết cho Số phương chia hết cho chia hết cho 25 Số phương chia hết cho chia hết cho 16 Mọi số phương chia cho 5, cho dư 1, 0,

8 Giữa hai số phương liên tiếp khơng có số phương

9 Nếu hai số nguyên liên tiếp có tích số phương hai số số 10 Số ước số phương số lẻ Ngược lại, số có số ước số lẻ số số phương

11.Nếun2< k < (n + 1)2( n ∈ Z) k khơng số phương.

CH

Đ

4 CÁC BÀI TOÁN V

(99)

CH

IN

H

P

H

C

K

T

H

I H

C S

IN

H

GI

I C

P H

AI

12 Nếu hai số tự nhiên a b nguyên tố có tích số phương số a, b số phương

13 Nếu a sốchính phương, a chia hết cho sốnguyên tố p a chia hết cho p2

14 Nếu tích hai số a b sốchính phương số a b có dạng

2

;  

a mp b mq

B CÁC DẠNG TOÁN THƯỜNG GẶP

Dạng 1: Chứng minh số số phương, tổng nhiều số

phương

* Cơ sở phương pháp:

Để chứng minh số n số số phương ta thường dựa vào định nghĩa, tức chứng minh: = ( ∈ )

n k k Z

* Ví dụ minh họa:

Bài tốn 1.Cho n sốtựnhiên Chứng minh rằng: An n 1n2n 3 số phương

Hướng dẫn giải

Ta có:  2  2   2 2  2   2 2

3 3 3

            

A n n n n n n n n n n

n nên n23n 1  Vậy A sốchính phương

Bài toán 2.Cho: B1.2.32.3.4  k k 1k2với k sốtựnhiên.Chứng minh

rằng 4B + sốchính phương

Hướng dẫn giải

Ta thấy biểu thức B tổng biểu thức nghĩ đến việc phải thu gọn biểu thức B trước

Ta có:

 1 2  1 2  3  1  1 2 3  1  1 2

4

n nn  n nn  n  n  n nnn  n n nn 

Áp dụng:

 

1

1.2.3 1.2.3.4 0.1.2.3

(100)

CH

UY

ÊN

Đ

S

H

C

 

 

1

2.3.4 2.3.4.5 1.2.3.4

1

3.4.5 3.4.5.6 2.3.4.5

   

           

1 2 1

4

k kk  k kkk  k k kk 

Cộng theo vếcác đẳng thức ta được:

      

   

1

1.2.3 2.3.4 2

4

4 1

B k k k k k k k

B k k k k

                

Theo ví dụ ta có:  2 2

4B 1 k 3k1

k  nên k23k 1  Vậy 4B1 sốchính phương

Bài toán 3.Chứng minh rằng:  

2

11 44 n n

C   với n sốtựnhiên.Chứng minh

Clà sốchính phương

Hướng dẫn giải Ta có: 11 100 0   11 44

n n

n n

C   

Đặt 11 1

n

a= 99 9

n

a= Do 99 10 n n

a

+ = = +

 

 

2

2

1

.10

9

33 n

n

C a a a a a a

C a a a

C

             

 

Vậy Clà số phương Nhận xét:

Khi biến đổi số có nhiều chữ số giống thành số phương ta nên đặt 11 1

n a

= 99 10 n n

a

+ = = + .

Bài toán 4.Cho 

2016

11

a= , 

2015

10

b= Chứng minh ab+1 số tự nhiên

Hướng dẫn giải

(101)

CH IN H P H Ụ C K Ỳ T H I H Ọ C S IN H GI Ỏ I C Ấ P H AI

Ta có:   

2015 2016 2016

10 10 9

b= = − + = + = a+ .

⇒ ab + = a(9a + 6) + = 9a2+ 6a + = (3a + 1)2

ab+1= (3a+1)2 =3a+1∈N

Vậy ab+1 số tự nhiên

Cách 2:

Ta có:  2016 2016

2016

10

11 , 10

9

a= = − b= +

( ) ( 2016)2 2016

2016

2016 10 4.10

10

1 10

9

ab − + − +

⇒ + = + + = 2016 10  +  =    

( 2016 )

10

3

ab +

⇒ + =

Mà ( 2016 )

10 +2 3 Do đó, ab+1 số tự nhiên Vậy ab+1 số tự nhiên

Bài toán 5.Cho số tự nhiên agồm 60 chữ số 1, số tự nhiên bgồm 30 chữ số Chứng minh

a - blà số phương

Hướng dẫn giải

Cách 1:

Ta có:  60

60

10 11

9

a= = − , 

30

30

10 22 2

9

b= = −

60 30 60 30

10 2(10 1) 10 2.10

9 9

a b − − − +

⇒ − = − =  2 30 30 10 33 3  −    =  =      . Cách 2:   30 30 22 2.11

b= = ,    

60 30 30 30

11 11 1.00 11

a= = +  30 

30 30

11 1.10 11

= +

Đặt 

30

11

c=  30

30

9c 99 10

⇒ + = + =

Khi đó: ( )

9

a=c c+ + =c c + c b=2c

( )2  2

30

9 2 33

a b c c c c  

⇒ − = + − = =  

 

(102)

CH

UY

ÊN

Đ

S

H

C

Bài toán 6.Cho n∈ cho

2

1

n

là tích hai sốtựnhiên liên tiếp Chứng minh

n tổng hai sốchính phương liên tiếp

Hướng dẫn giải Giảsửta có:

3

n

=a a( +1)

Từđó có 2

3

n = a + a+ ⇒ 4n2− =1 12a2+12a+3

⇒ ( )( ) ( )2

2n−1 2n+ =1 2a+1

Vì 2n+1; 2n−1là hai sốlẻliên tiếp nên ta có trường hợp:

Trường hợp 1: 32 2

n p

n q

 − = 

+ =



Khi 2

3

q = p + ( Vơ lí ) Vậy trường hợp không xảy

Trường hợp 2:

2

2

2

n p

n q

 − = 

+ =



Từđó p sốlẻnên p=2k+1

Từđó ( )2

2n= 2k+1 +1 ⇒ n=k2+(k+1)2 (đpcm)

Bài toán 7.Cho k sốnguyên dương a3k23k1 a) Chứng minh 2a a2 tổng ba sốchính phương

b) Chứng minh a ước sốnguyên duong b b tổng gồm ba sốchính phương n

b tổng bà sốchính phương

Hướng dẫn giải

a) Ta có  2  2

2a6k 6k 2 2k1  k1 k

và 2 4 3 2  2  2 2  2 2 2 2 2 2

1

9 18 15

              

a k k k k k k k k k k a a a

b) Vì b a nên đặt bca

b tổng ba sốchính phương nên đặt bb12 b22 b32 Khi 2 2 2 2

1

(103)

CH

IN

H

P

H

C

K

T

H

I H

C S

IN

H

GI

I C

P H

AI

Đểkết thúc việc chứng minh, ta tiến hành sau: cho n2p1 ta được:

  2 

2 2

1

   

p p

b b b b b cho n2p2 ta   2 2 2

1

  

n p

b b b a a a

Dạng 2: Chứng minh số không số phương

* Cơ sở phương pháp:

Để chứng minh n khơng số phương, tùy vào tốn ta sử dụng cách sau:

1) Chứngminh n viết dạng bình phương số nguyên 2) Chứng minh k2< n < (k + 1)2với k số nguyên

3) Chứng minh n có tận 2; 3; 7; 4) Chứng minh n có dạng 4k + 2; 4k + 5) Chứng minh n có dạng 3k +

6) Chứng minh n chia hết cho số nguyêntố p mà không chia hết cho p2

* Ví dụ minh họa:

Bài tốn 1. Một sốtựnhiên có tổng chữsốbằng 2018 có thểlà sốchính phương khơng ? sao?

Hướng dẫn giải Gọi số tự nhiên có tổng chữ số 2018 n

Ta có : 2018 = 3m + nên số tự nhiên n chia dư 2, số n có dạng 3k + với k số tự nhiên Mặt khác số phương trình khơng có dạng 3k + suy số tự nhiên n khơng số phương

Bài tốn 2.Chứng minh số

2 2

Annnn n ∈ N n >

không phải số phương

Hướng dẫn giải Ta có:

   

     

 

4 2

2 2

2

2

2 2 2

1

1

A n n n n n n n n n

n n n n n n

A n n n

                  

     Mặt khác:

 

 

2

2 2

4 2

1 2

2 2 1

n n n n n n n

n n n n n A n A n

       

(104)

CH

UY

ÊN

Đ

S

H

C

 2 2

1

A n n

   

Do  2 2  2 2

1

nn  A n  n

Ta có (n2+ n) (n2+ n + 1) hai số tự nhiên liên tiếp nên A số

phương

Bài tốn 3.Cho 33

1 2

A= + + + + + Hỏi A có số phương khơng? Vì sao?

Hướng dẫn giải

Ta có ( 5) ( 30 31 32 33)

1 2 2 2 2

A= + + + + + + + + + +

( ) ( )

2 30

3 2 2

= + + + + + + + + +

( )

29 29

3 2.30 30 3.10

= + + + = + + +

Ta thấy A có chữ số tận

Mà số phương khơng có chữ số tận Do đó, A khơng số phương Vậy A khơng số phương

Bài tốn 4.Chứng minh 4 4

2012 n 2013 n 2014 n 2015 n

A= + + + số

phương với số nguyên dương n

(Đềthi vào lớp 10 chuyên trường ĐHSP TP Hồ Chí Minh 2015 - 2016)

Hướng dẫn giải Ta có:

4

2012 n4; 2014 n4, ∀ ∈n N*

( )

4 4

2013 n =2013 n− + =1 2013 n− +1 chia cho dư

( )4

4

2015 n =2015 n− −1 n +1 chia cho dư

Do đó, 4 4

2012 n 2013n 2014 n 2015 n

A= + + + chia cho dư

Ta có: A2, A khơng chia hết cho 22, mà số nguyên tố Suy A khơng số phương

Vậy A khơng số phương

Bài tốn 5.Cho 2 n , Chứngminh An6 n4 2n32n2 khơng thểlà sốchính

(105)

CH

IN

H

P

H

C

K

T

H

I H

C S

IN

H

GI

I C

P H

AI

Hướng dẫn giải

Ta có 2 

2 2

       

A n n n n n n n n

   

2 2

1

 

n n n   n 

    

2

1

 

n n nn  n 

 2 

2

1 2

n nnn

Với 2 n , ta có n22n 2 n22n  1 n 12

Và 2  

2 2

     

n n n n n Do n12n22n 2 n2 Như

2  

n n khơng phải sốchính phương nên A khơng phải sốchính phương

Bài tốn 6.Chứng minh tổng bình phương haisốlẻbất kì khơng phải số phương

Hướng dẫn giải Giảsử: a2m1, b2n1, với m n, 

Ta có: 2   2 2  2 

2 4

           

a b m n m m n n k với k

Khơng có sốchính phương có dạng 4k2 a2b2 khơng phải sốchính phương  Dạng 3: Điều kiện để số số phương.

* Cơ sở phương pháp: Chúng ta thường sử dụng phương pháp sau: - Phương pháp 1: Sửdụng định nghĩa

- Phương pháp 2: Sửdụng tính chẵn, lẻ

- Phương pháp 3: Sửdụng tính chất chia hết chia có dư - Phương pháp 4: Sửdụng tính chất

*Ví dụ minh họa:

Bài tốn 1.Tìm số ngun n cho n n 3 số phương

Hướng dẫn giải

Để An n 3 số phương n n  3 k2 với k số tự nhiên, đó:

2

2

3

4 12

n n k

n n k

(106)

CH UY ÊN Đ Ề S Ố H Ọ C        2 2

4 12 9

2

2 2

n n k

n k

n k n k

        

      Ta có 2n2k 3 2n2k3

Và 99.13.3        1   9 3

Trường hợp 1: 2

2 1

n k n k n

A

n k n k k

                                   

Trường hợp 2: 2 3 0

2 3 0

n k n k n

A

n k n k k

                                  

Trường hợp 3: 2 4

2

n k n k n

A

n k n k k

                                       

Trường hợp 4: 2 3 3

2 3

n k n k n

A

n k n k k

                                       

Vậy n  4; 3; 0;1 ta có A sốchính phương

Bài tốn 2.Tìm sốngun n cho n+1955 n+2014 sốchính phương

Hướng dẫn giải

Giảsử

1955

n+ =a ; n+2014=b2 với a, b∈ a<b

Khi 2 ( )( ) 29

59 59

59 30

b a a

b a b a b a

b a b

− = =

 

− = ⇔ − + = ⇔ ⇔

+ = =

 

Dễdàng suy n= −1114

Bài tốn 3.Tìm số ngun dương n để biểu thức sau số phương:

2

) )

a An  n b Bn  n

Hướng dẫn giải a) Với n = A = n2 – n + = khơng số phương

Với n = A = n2 – n + = số phương

Với n > A = n2 – n + khơng số phương

 2 2   2   2

1 2

n  n n n   n n

(107)

CH

IN

H

P

H

C

K

T

H

I H

C S

IN

H

GI

I C

P H

AI

b) Ta có:    

1

n  n nn n

Với n = 5k n chia hết cho

Với n5k1thì n21chia hết cho

Với n5k2thì n2 1chia hết cho Do

nn chia hết cho

Nên

2

n  n chia cho dư nên

2

n  n có chữsốtận nên

5

2

Bn  n không sốchính phương

Vậy khơng có giá trịnào n thỏa đểB sốchính phương

Bài tốn 4.Tìm sốngun dương n nhỏnhất cho số n+1, 2n+1, 5n+1 sốchính phương

Hướng dẫn giải

Nếu n=3k+1 (k∈) n+ =1 3k+2, khơng sốchính phương

Nếu n=3k+2 2n+ =1 6k+5, cho cho dư nên khơng sốchính phương Vậy n3 2n+1 sốchính phương lẻ nên chia cho dư Suy 2n8⇒n4⇒ +n lẻ Do n+1 sốchính phương lẻnên n+1 chia cho dư 1, suy n8

n chia hết cho số nguyên tốcùng nên n24 Với n=24

1 25

n+ = = ,

2n+ =1 49=7 , 5n+ =1 121 11=

Giá trịnhỏnhất n phải tìm 24

Bài tốn 5.Tìm số tự nhiên n ≥ cho tổng 1! + 2! + 3! + … + n! số phương

(Đề thi HSG lớp - Phòng giáo dục đào tạo Phúc Yên - Vĩnh Phúc)

Hướng dẫn giải Với n = 1! = = 12là số phương

Với n = 1! + 2! = khơng số phương

Với n = 1! + 2! + 3! = + 1.2 + 1.2.3 = = 32là số phương

Với n ≥ ta có 1! + 2! + 3! + 4! = + 1.2 + 1.2.3 + 1.2.3.4 = 33 5!; 6!; …; n!đều tận 1! + 2! + 3! + … n! có tận cùngbởi chữ số nên khơng phải số phương Vậy có số tự nhiên n thoả mãn đề n = 1; n =

Bài tốn 6. Tìm số ngun dương n cho ( )( )

3 14

(108)

CH

UY

ÊN

Đ

S

H

C

phương

(Đề thi chọn HSG Tốn tỉnh Thái Bình)

Hướng dẫn giải

Ta có: ( )( )

4n +14n+ =7 n+3 4n+2 +1 n số nguyên dương nên n+3 4n2+14n+7 nguyên tố Vì vậy, để A số phương

4n +14n+7 n + phải số phương

Do nZ+ nên ta có ( )2 ( )2

2n+3 ≤4n +14n+ <7 2n+4

( )2

4n 14n 2n

⇒ + + = + ⇒ =n Khi n + = số phương Thử lại, với n=1, ta có A=102

Vậy số ngun dương cần tìm n=1

Bài tốn 7.Tìm 3≤ ∈a  cho a a( −1 ) (a a− =1) (a−2) (aa a−1 )

Hướng dẫn giải

Ta có ( ) ( ) ( ) ( ) ( ) (2 ) ( )

1 1

a aa a− = aaa a− ⇔a a− = aaa a− (*) Vì VT(*) sốchính phương nên VP(*) sốchính phương

Vì sốchính phương chỉcó chữsốtận thuộc tập hợp {0;1; 4;5; 6;9} nên a có chữsốtận thuộc tập hợp {1; 2;5; 6; 7; 0}

Do a chữsốnên a≤9 Kết hợp với 3≤ ∈a  nên a∈{5; 6; } Thửlần lượt giá trịta thu a=7 thỏa mãn 762 =5776 Bài tốn 8.Tìm sốtựnhiên n cho 2n+9 sốchính phương

Hướng dẫn giải

Giảsử

2n+ =9 m , m∈ ⇔ (m−3)(m+ =3) nm− < +3 m nên ,

3 a

b

m m

 − = 

+ =

 với a, b∈ a<b

Ta có 2b−2a = ⇔6 2a(2b a− − =1)

(109)

CH

IN

H

P

H

C

K

T

H

I H

C S

IN

H

GI

I C

P H

AI

Dạng 4: Tìm số phương.

* Cơ sở phương pháp: Dựa vào định nghĩa số phương

Ak , với k số

nguyên u cầu tốn để tìm số phương thỏa tốn *Ví dụ minh họa:

Bài tốn 1.Tìm số phương abcd biết ab cd− =1

Hướng dẫn giải

Giả sử ( )

100 100

n =abcd = ab+cd = +cd +cd =101cd+100, nZ

( )( )

2

101.cd n 100 n 10 n 10

⇒ = − = − +

n<100 101 số nguyên tố nên n+10 101= 91

n

⇒ =

Thử lại:

91 8281

abcd = = có 82 81 1− = Vậy abcd =8281

Bài tốn 2. Cho A số phương gồm chữ số Nếu ta thêm vào chữ số A đơn vị ta số phương B Hãy tìm số A B

Hướng dẫn giải

Gọi

A=abcd =k

Theo đề ta có:

2

1111

A abcd k

B abcd m

 = =

 

= + =



(với k m, ∈N*và 31< <k m<100,

, , , 1,

a b c d = )

2

1111

m k

⇒ − = ⇔ (m - k)(m + k) = 1111 (*)

Nhận xét thấy tích (m – k)(m + k) > nên m – k m + k số nguyên dương Và m – k < m + k < 200 nên (*) viết (m – k) (m + k) = 11.101

Do đó: 11 56 2025

101 45 3136

m k m A

m k k B

      

  

  

  

      

  

  

Vậy A = 2025, B = 3136

(110)

CH

UY

ÊN

Đ

S

H

C

căn bậc hai số có tổng chữ số số phương

Hướng dẫn giải

Gọi số phải tìm abcd với a; b; c; d số tự nhiên

và ≤ a ≤ 9; ≤ b, c, d ≤ Ta có abcd phương ⇒ d ∈{0,1,4,5,6,9}

Vì d số nguyên tố ⇒ d =

Đặt

10000

abcdk  ⇒ 32 ≤ k < 100, kN

Do k số có hai chữ số mà k2có tận ⇒ k tận 5

Tổng chữ số k số phương ⇒ k = 45 (vì k tận có chữ số)

abcd2025

Vậy số phải tìm là: 2025 C BÀI TẬP ÁP DỤNG

Bài 1: Cho a b c; ; số nguyên thỏa mãn điều kiện ab+bc+ca=1

Chứng minh 2

(a +1)(b +1)(c +1) số phương

Bài 2: Tìm số ngun dương n cho (2 1) 26

n n

là số phương

(Đề TS lớp 10 THPT Chuyên Lam Sơn- Thanh Hóa 2012-2013)

Bài 3: Tìm tất số nguyên n cho

A=n +n +n có giá trị số phương

(Đề TS lớp 10 THPT Chuyên Phan Bội Châu-Nghệ An 2010-2011 )

Bài 4: Chứng minh số nguyên x, y thìbiểu thức

( )( )( )( )

2

A= x+ y x+ y x+ y x+ y +y có giá trị số phương

Bài 5: Chứng minh sốsau sốchính phương:

a)  

2

224 99 9100

n n

A b)  

1

11 155

n n B

Bài 6: Chứng minh tổng bình phương sốliên tiếp khơng thểlà sốchính phương Bài 7: Cho dãy số 49; 4489; 444889; 44448889;

(111)

CH

IN

H

P

H

C

K

T

H

I H

C S

IN

H

GI

I C

P H

AI

Bài 8: Chứng minh p tích n sốngun tốđầu tiên p−1 p+1

khơng thểlà sốchính phương

Bài 9: Có hay khơng số tự nhiên n để 2010 + n2là số phương

Bài 10: Chứng minh tổng bình phương sốtựnhiên liên tiếp khơng thểlà sốchính phương

Bài 11: Chứng minh n số tự nhiên cho n + 2n + số phương n bội số 24

Bài 12: Tìm số phương có chữ số biết chữ số đầu giống nhau, chữ số cuối giống

Bài 13: Tìm số lẻ liên tiếp mà tổng bình phương số có chữ số giống Bài 14: Cho sốnguyên dương n số A =

2

444 n

 (A gồm 2n chữ số 4); B = 888 n

 (B

gồm n chữ số 8) Chứng minh A + 2B + số phương

(Đề vào chuyên toán Hà Nam năm 2013-2014)

Bài 15: Giảsử N =1.3.5.7 2007

Chứng minh sốngun liên tiếp 2N−1, ,N 2N+1khơng có sốnào số phương

Bài 16: Với số nguyên dương n, ký hiệu Sn tổng n số nguyên tố

S1 2,S2  2 3,S3   2 5,  Chứng minh dãy số S S S1, 2, 3, không tồn hai số hạng liên tiếp số phương

(Đề vào chun tốn sư phạm Hà Nội năm 2013-2014)

Bài 17: Cho p số nguyên tố Tìm p để tổng ước nguyên dương

p số phương

(Đề vào chuyên Hưng Yên năm 2013-2014)

Bài 18: Tìm tất cảsốtựnhiên n cho

14 256

nn mộtsốchính phương

(Đề thi HSG lớp Thanh Oai năm 2012-2013)

Bài 19: Cho sốnguyên a, b, c ≠ thoảmãn: 1 1

a b c abc+ + =

Chứng minh rằng: (1 a b c+ 2)( + 2)( + 2) là sốchính phương

(Đề thi HSG lớp trường Trần Mai Ninh năm 2012-2013)

Bài 20: Tìm sốtựnhiên n cho

6

An  n sốchính phương

(Đề thi HSG lớp huyện Vĩnh Lộc năm 2018-2019)

Bài 21: Tìm sốtựnhiên gồm bốn chữsố abcd biết sốchính phương, chia

hết cho d sốnguyên tố

(Đề thi HSG lớp quận Ngô Quyền năm 2018-2019)

Bài 22: (Đề thi HSG lớp huyện Cẩm Giang năm 2018-2019)

Cho 98

2 2

(112)

CH

UY

ÊN

Đ

S

H

C

Bài 23: Tìm x nguyên dương để 4x 14x 9x 63+ 2+ − là sốchính phương

(Đề thi HSG lớp TP Bắc Giang năm 2017-2018)

Bài 24: Tìm sốtựnhiên n cho n 172+ là sốchính phương?

(Đề thi HSG lớp huyện Kim Thành năm 2012-2013)

Bài 25: Tìm sốngun dương n cho 2n+3n+4n sốchính phương

(Đề thi HSG lớp huyện Vũ Quang năm 2018-2019)

Bài 26: Tìm tất cảcác sốnguyên n cho n 20142 + là sốchính phương

(Đề thi HSG lớp Trường Thanh Văn năm 2017-2018)

Bài 27: Tìm sốnguyên x cho x 3x x 23− 2+ + sốchính phương

(Đề thi HSG lớp huyện Lục Nam năm 2018-2019)

Bài 28: Tìm sốtựnhiên Abiết ba mệnh đềsau có hai mệnh đềđúng mệnh đềsai:

a) A 51+ sốchính phương

b) Chữsốtận bên phải Alà số c) A 38− sốchính phương

(Đề thi HSG lớp huyện Đan Phượng năm 2018-2019)

Bài 29: Tìm sốhữu tỉ n thỏa mãn tổng sau sốchính phương: n2+ +n 503 Giảsửtồn sốhữu tỉ n sốnguyên dương m để 2

503

n + +n =m

(Đề thi HSG lớp 9huyện Vũ Quang năm 2018-2019)

Bài 30: Tìm sốtựnhiên n cho n50 n50 sốchính phương

(Đề thi HSG lớp huyện Thăng Bình năm 2018-2019)

Bài 31: Tìm sốtựnhiên n cho: n24 n65 hai sốchính phương

(Đề thi HSG lớp huyện Phù Ninh năm 2018-2019)

Bài 32: Chứng minh rằng:     2

4

Bx xy x y z x z y z sốchính phương

với x,y, z sốnguyên

(Đề thi HSG lớp huyện Tiền Hải năm 2017-2018)

Bài 33: Tìm *

n∈ cho: n4+n3+1 sốchính phương

(Đề thi HSG lớp huyện Thanh Oai năm 2012-2013)

Bài 34:Tìm tất cảcác cặp sốtựnhiên ( )x y; cho x( +y2 −3x 2y 1+ )−

( 2 )

5 x +y +4x 2y 3+ + sốchính phương

(Đề vào 10 Chuyên Nam Định năm 2019-2020)

Bài 35: Chứng minh số M=(n 1+ )4 +n4 +1 chia hết cho sốchính phương khác

với số n nguyên dương

(Đề vào 10 Chuyên Bình Thuận năm 2019-2020)

Bài 36: Cho n số nguyên dương thỏa mãn 12n2 1 là số nguyên Chứng minh

2 12n  1 2 số phương

(Đề vào 10 Chuyên Bắc Ninh năm 2019-2020)

Bài 37: Cho a, b, c số nguyên dương nguyên nguyên tố thỏa mãn

1 1

(113)

CH

IN

H

P

H

C

K

T

H

I H

C S

IN

H

GI

I C

P H

AI

(Đề vào 10 Chuyên Thái Nguyên năm 2016-2017)

Bài 38: Chứng minh a b số tự nhiên lẻ 2

a +b khơng phải số phương

(Đề vào 10 Chun Hịa Bình năm 2016-2017)

Bài 39: Tìm tất số nguyên dương n cho

3n

n + số phương

(Đề vào 10 Chuyên Quốc Học Huế năm 2017-2018)

Bài 40: Chứng minh sốtựnhiên abc sốnguyên tốthì

4

bac khơng số phương

(Đề vào 10 Chuyên Bình Định năm 2017-2018)

Bài 41: Tìm số nguyên m cho

12

m + số phương

(Đề vào 10 Chuyên Phú Thọ năm 2017-2018)

Bài 42: Tìm tất cảcác cặp (x; y) nguyên dương cho

8

+

x y

8

+

y x sốchính

phương

(Đề vào 10 Chuyên Toán Hải Dương năm 2017-2018)

Bài 43: Cho biểu thức ( )2

3

A= m+n + m+n với m, n số nguyên dương Chứng minh A số phương

1

n + chia hết cho m

(Đề vào 10 Chuyên TP Hồ Chí Minh năm 2017-2018)

Bài 44: Cho p số nguyên tố Tìm tất số nguyên n để

4 p

A=n + n − số phương

(Đề vào 10 Chuyên Bà Rịa Vũng Tàu năm 2017-2018)

Bài 45: Cho hai sốnguyên dương m, n thỏa mãn m n+ +1là ước nguyên tốcủa

( 2)

2 m +n −1 Chứng minh m n sốchính phương

(Đề vào 10 Chuyên Nghệ An năm 2018-2019)

Bài 46: Tìm giá trịnguyên xđể ( )3

1 2

M =x + x+ − xxlà sốchính phương

(Đề vào 10 Chuyên Hưng Yên năm 2018-2019)

Bài 47: Cho sốtựnhiên n≥2và sốnguyên tố pthỏa mãn p−1chiahết cho nđồng thời

3

1

n − chia hết cho p Chứng minh n+plà sốchính phương

(Đề vào 10 Chuyên Đại học Vinh Nghệ An năm 2018-2019)

Bài 48: Tìm hai số nguyên tố p q, biết p+q p+4q số phương

(Đề vào 10 Chuyên Quảng Nam năm 2018-2019)

Bài 49: Chứng minh hiệu lập phương 2sốnguyên liên tiếp bình phương sốtựnhiên n n tổng sốchính phương liên tiếp

(Đề vào 10 Chuyên Bắc Ninh năm 2018-2019)

Bài 50: Chứng minh không tồn số tự nhiên n để 2018+n2 số phương

(Đề vào 10 Chuyên Bắc Giang năm 2018-2019)

Bài 51: Cho 2

4

A=m nmn với m n, sốnguyên dương Khi n=2 tìm m đểA sốchính phương Khi n≥5chứng minhrằng Akhơng thểlà sốchính phương

(Đề vào 10 Chuyên Bà Rịa Vũng Tàu năm 2018-2019)

(114)

CH

UY

ÊN

Đ

S

H

C

Bài 53:Tìm số tự nhiên x để biểu thức x2+2x+20 có giá trị số phương

Bài 54.Tìm sốngun x cho Ax x( 1)(x7)(x8) sốchính phương

Bài 55.Cho  

2

11 88

n n

A= − + Chứng minh A số phương

Bài 56.Tìm tất số tự nhiên x,y để 2x 5y số phương Bài 57.Tìm nN để 28+211+2n số phương

Bài 58.Tìm sốtựnhiên n có chữsốbiết 2n+1 1n+ sốchính phương

Bài 59.Cho số:

2

1

11 11 11 11 ;

66 66

m m

m

A B C

+

 =    =    = 

  

Chứng minh rằng: A B C+ + +8 sốchính phương

Bài 60.Tìm tất cảcác sốnguyên n cho n42n32n2 n sốchính phương

(Đềthi vào lớp 10 chuyên, trường ĐHKHTN – ĐHQG Hà Nội năm 1992)

Bài 61.Tìm tất cảcác sốngun khơng âm n cho có sốnguyên a, b thỏa mãn

n = +a b n3 =a2+b2

(Romanian MO 2004)

Bài 62.Hãy tìm hai sốchính phương phần biệt a a a a1 b b b b1 biết

1 2 3 4

a − =b a − =b a − =b ab

Bài 63.Có tồn hay không 2013 sốnguyên dương a1, a2, , a2013 cho số

2

1 2,

a +a a12+a22+a32,

2 2

1 2013

a +a + +a sốchính phương?

Bài 64.Thay dấu * chữ số cho số sau số tự nhiên

6

4 ****

A=

Bài 65 Với n∈ , đặt ( )( )

10n 10n 10 10n

n

A = + − + + + + + + Chứng minh An

là sốchính phương

Bài 66 Giảsửrằng 2n+1 3n+1 sốchính phương Chứng minh 5n+3 hợp số

Bài 67 Có hay khơng số x y, phân biệt thuộc khoảng (988;1994) choxy+x xy+y sốchính phương ?

( Thi học sinh giỏi toán lớp 9, TP.HCM năm 1994)

(115)

CH

IN

H

P

H

C

K

T

H

I H

C S

IN

H

GI

I C

P H

AI

Bài 69 Cho dãy số , a2 =144, a3 =1444,

4

1444 44

n

n chu so a = 

Tìm tất số tự nhiên n cho an số phương

Bài 70 Chứng minh có vơ số ba số tự nhiên (a b c, , )sao cho a b c, , nguyên tố số 2 2 2

n=a b +b c +c a số phương

Bài 71 Tìm số nguyên m n đa thức

( ) 29 4,

p x =x +mx + x +nx+ x∈

số phương Bài 72

1 Tìm số tự nhiên a nhỏ nhất, a≠0sao cho a chia hết cho 1000a số phương

2 Tìm số tự nhiên b nhỏ cho số (b−1) khơng chia hết cho 9, b chia hết cho tích

của bốn số nguyên tố liên tiếp 2002.b số phương

Bài 73 Cho a b số tự nhiên, a2b2 số phương khơng?

Bài 74 Tìm sốtựnhiên k =ab có hai chữsốsao cho k+ab=(a b+ )2

Bài 75 Tìm tất cảcác sốnguyên n để 2 2

2017

An  n n sốchính phương

(Tạp chí Tốn & học tuổi trẻsố468)

Bài 76 Tìm sốnguyên dương n để 37 43

n n

 bình phương sốhữu tỷdương tùy ý

(HSG Nam Định 2015 -2016)

Bài 77 Tìm sốtựnhiên có dạng abc thỏa mãn: abc=n2−1 cba=(n−2)2 với n∈, n >2

(HSG Sóc Trăng 2015 - 2016)

Bài 78.Tìm sốtựnhiên n cho n+12 n−11đềulà sốchính phương

(HSG Sóc Trăng 2016 - 2017)

Bài 79 Tìm tất cảcác sốtựnhiên n cho

14 256

− −

n n sốchính phương

(HSG Quảng Nam 2014 - 2015)

Bài 80 Cho n sốtựnhiên có chữsố Tìm n biết n + 2n sốchính phương

(HSG Trà Vinh 2016 - 2017)

Bài 81 Cho n số tự nhiên Hãy tìm tất số nguyên tố p cho số ( ) 195

2 10

1010 2010 10

= + + +

A n n p có thểviết dạng hiệu sốchính phương

(HSG Lâm Đồng 2016 - 2017).

Bài 82.Tìm nghiệm nguyên dương x để 3x+171là sốchính phương

(HSG Lai Châu 2015 - 2016)

Bài 83.Tìm tất cảcác sốtựnhiên x cho 5x+12x sốchính phương

(116)

CH

UY

ÊN

Đ

S

H

C

Bài 84. Tìm tất số nguyên n cho A số phương với

4

4 22 37 12 12

= + + + −

A n n n n

(Chuyên Yên Bái 2016 - 2017).

Bài 85.Tìm sốnguyên kđể

8 23 26 10

− + − +

k k k k sốchính phương

(Chuyên Hải Dương 2015 - 2016).

Bài 86.Tìm sốtựnhiên n (n > 1) bé cho 12+22+ + ⋅⋅⋅ +32 n2

n sốchính phương

(Tạp chí tốn học tuổi trẻsố362).

Bài 87: Tìm tất số tự nhiên n cho hai số 9n+16 16n+9 số

phương

Bài 88:Lấy sốtự nhiên có chữ số chia cho số có chữsố viết theo thứtự ngược lại thương dư 15 Nếu lấy số trừ số tổng bình phương chữsốtạo thành sốđó Tìm sốtựnhiên

Bài 89 Viết số1, 2, 3, …, 2007 thành dãy theo thứtựtùy ý sốA Hỏi số 2007

2008 2009

A+ + có phải sốchính phương hay khơng? Vì sao?

(Tạp chí tốn học tuổi trẻsố377)

Bài 90 Cho sốhữu tỉx, y thỏa mãn 5 2

2x

x +y = y Chứng minh 1−xy bình phương sốhữu tỉ

Bài 91.Cho m n, hai sốnguyên dương lẻsao cho n21 chia hết cho [m2 1 n2] Chứng

minh 2

[m  1 n ] sốchính phương

Bài 92 Chứng minh ba sốchính phương tuỳý tồn hai sốmà hiệu chúng chia hết cho

Bài 93 Chứng minh

1999 2017

nn (nN) sốchính phương

(HSG Tỉnh Quảng Ngãi 2017 – 2018)

Bài 94 Giảsử n sốnguyên dương thoảmãn điều kiện

3

n  n sốnguyên tố Chứng minh n chia dư 1và 7n26n2017 khơng phải sốchính phương

(Chun Tỉnh Quảng Ngãi 2017-2018)

Bài 95.Cho x y, sốnguyên thoảmãn 2x2 x 3y2y

Chứng minh xy; 2x2y1và 3x3y1 sốchính phương

(HSG Tỉnh Thanh Hố 2015-2016)

Bài 96 Cho biểu thức 2

2(1 2017 )

A    Hỏi A có bình phương số ngun hay khơng?

(Tốn học tuổi thơsố120)

Bài 97.Cho a b sốtựnhiên thoảmãn 2

(117)

CH

IN

H

P

H

C

K

T

H

I H

C S

IN

H

GI

I C

P H

AI

Chứng minh ab sốchính phương

(Tốn học tuổi thơ số120)

Bài 98 Cho x y z, , số nguyên tốcùng thoảmãn (xz y)(  z) z2 Chứng

minh tích

2017 xyz sốchính phương

(Tốn học tuổi thơ số120)

Bài 99:Xác định sốđiện thoại THCS thành phốThủ Dầu Một, biết sốđó dạng 82 xx yy với x yyx sốchính phương

(HSG Bình Dương 2016 – 2017)

Bài 100:Tìm tất cảcác sốtựnhiên nsao cho C 2019= n+2020 sốchính phương

(HSG Quảng Bình 2018 – 2019)

Bài 101:Tìm sốnguyên tố p thỏa mãn p 4p 93− + là sốchính phương.

(HSG Bắc Ninh 2018 – 2019)

Bài 102: Cho B=1.2.3 2.3.4 3.4.5 + + + +n n.( −1 ) (n−2) với n∈* Chứng minh B khơng sốchính phương

(HSG Bắc Ninh 2018 – 2019)

Bài 103: Cho số nguyên tố p p 3và hai số nguyên dương a b, cho p2 a2 b2 Chứng minh a chia hết cho 12 2p a 1 số phương

(HSG Quảng Nam 2018 – 2019)

Bài 104: Từ625 sốtựnhiên liên tiếp 1; 2; 3; …; 625 chọn 311 sốsao cho khơng có hai số có tổng 625 Chứng minh 311 sốđược chọn, bao giờcũng có sốchính phương

(HSG Hưng Yên2017 – 2018)

Bài 105: Tìm sốtựnhiên n cho n2+2n+ n2+2n 18 9+ + sốchính phương

(HSG Hải Dương2016 – 2017)

Bài 106: Tìm sốcó chữsố ab a( ≠b) cho số n=ab ba− sốchính phương

(HSG Hưng Yên2015 – 2016)

Bài 107:Cho = 

2017 sè

a 111 1 = 

2016 sè

b 000 5 Chứng minh số M ab 1= + sốchính phương

(HSG Đăk Lăk2015 – 2016)

Bài 108:Chứng minh với sốnguyên n ≥ số: 2.6.10 (4 2) ( 5)( 6) (2 ) n

n a

n n n

− = +

+ +

sốchính phương

(118)

CH

UY

ÊN

Đ

S

H

C

Bài 109: Tìm a b, để f x( )=x4+2x3−x2+x a( − + +4) b viết thành bình phương đa thức

(HSG huyện Chương Mỹ 2019 – 2010)

Bài 110:Xác định số điện thoại THCS X thành phố Thủ Dầu Một, biết số dạng 82xxyy với xxyy số phương

(HSG tỉnh Bình Dương2016 – 2017)

Bài 111:Cho hai sốtự nhiên a, bthỏa mãn 2

2a + =a 3b +b Chứng minh 2a + 3b +

sốchính phương

(HSG tỉnh Hải Dương2016 – 2017)

Bài 112:Cho n sốnguyên dương m là ước nguyên dương 2n2 Chứng minh

n2+ mkhơng sốchính phương

(HSG tỉnh Hải Dương2016 – 2017)

Bài 113:Tìm tất cảcác sốnguyên dương n để 13

2 2 2n

A= + + sốchính phương

(HSG tỉnh Hải Dương 2009 – 2010)

Bài 114 Cho a, b hai sốnguyên dương, đặt ( )2 2 ( )2 2

2 ,

= + − = + −

A a b a B a b b

Chứng minh A B không đồng thời sốchính phương

(Vào 10 Chuyên Sư Phạm Hà Nội 2018 – 2019)

Bài 115. Cho số nguyêna,b thỏa mãn 2

1 2( )

a +b + = ab+ +a b Chứng minh a b hai

số phương liên tiếp

(Vào 10 Chuyên Sư Phạm Hà Nội 2015 – 2016)

Bài 116.Cho hai sốhữu tỉa, bthỏa mãn đẳng thức 3 2

2 2

+ + + + + =

a b ab a b a b Chứng minh 1 – ab là bình phương sốhữu tỉ

(Vào 10 Chuyên Sư Phạm Hà Nội 2011 – 2012)

Bài 117. Giảsửmn là sốnguyên dương với n > Đặt 2

4

= − +

S m n m n

Chứng minh rằng:

1) Nếu m > n thì ( 2 )2 2 2 4

2

− < <

mn n S m n

2) Nếu Slà sốchính phương m = n.

(Vào 10 Chuyên Sư Phạm Hà Nội 2010 – 2011)

Bài 118.Cho x, y số nguyên lớn cho 2

4x y −7x+7y số phương

Chứng minh rằng: x= y

(Vào 10 Chuyên Khoa học tựnhiên 2014 – 2015)

Bài 119 Cho biểu thức ( )2

3

A= m+n + m+n với m, n số nguyên dương Chứng minh A số phương

1

n + chia hết cho m

(Vào 10 Chuyên TP HồChí Minh 2017 – 2018)

Bài số 120 Chứng minh rằng: Nếu abc sốnguyên tốthì

4

(119)

CH

IN

H

P

H

C

K

T

H

I H

C S

IN

H

GI

I C

P H

AI

Bài 121 Tìm sốnguyên dương n nhỏnhất để ( 4)( 3)

3

n+ n+

là sốchính phương Bài 122 Tìm sốnguyên tố x y, cho: x2+3xy+y2 sốchính phương

Bài 123 Cho sốtựnhiên y>x thỏa mãn: (2y−1) (2 = 2yx)(6y+x) Chứng minh 2yx sốchính phương

Bài 124 Cho sốnguyên dương a b c, , thỏa mãn: (a b c, , )=1,ab=c a b( − ) Chứng minh: a b− sốchính phương

Bài 125 Cho x y, số nguyên dương chox2+y2−x chia hết cho xy Chứng minh: x

là sốchính phương

Bài 126 Cho số tự nhiên a b c, , thỏa mãn điều kiện a b− số nguyên tố

( )

2

3c =ab c a b+ + Chứng minh: 8c+1 sốchính phương

Bài 127 Giả sử n số tự nhiên lớn cho 8n+1 24n+1 số phương Chứng minh rằng: 8n+3 hợp số

Bài 128 Cho a b, hai số nguyên cho tồn hai số nguyên liên tiếp c d để

2

a b− =a c b d− Chứng minh a b− sốchính phương

Bài 129 Cho sốtự nhiên a b c, , cho a2+b2+c2 =(a b− ) (2 + b c− ) (2+ −c a)2 Chứng minh số ab bc ca, , ab bc ca+ + sốchính phương

Bài 130 Cho 2  2

1

33 55 44

n n n

A

= + Chứng minh Alà sốchính phương Bài 131 Tìm tất cảcác sốtựnhiên n để 4n+9 9n+10 sốchính phương Bài 132 Tìm tất cảcác sốtựnhiên n để 3n+144 sốchính phương

Bài 133 Tìm tất cảcác sốnguyên dương n để 3n+63 sốchính phương

Bài 134 Chứng minh thêm chữ số vào chữ số số 1681 đểthu sốchính phương

Bài 135 Tìm tất cảcác sốtựnhiên n để 2012 2015

2 +2 +2n sốchính phương Bài 136 Tìm tất cảcác cặp sốtựnhiên m n, cho 2m+3n sốchính phương

Bài 137 Tìm tất cảcác cặp sốnguyên dương (m n, ) để 5m n+25 sốchính phương

Bài 138 Tìm sốnguyên dương x y, cho x2+3y y2+3x sốchính phương

Bài 139

a) Chứng minh rằng: Nếu n sốtựnhiên cho 2n+1 3n+1 sốchính phương 40

n

b) Tìm tất cảcác sốtựnhiên ab để 2ab+1, 3ab+1 sốchính phương Bài 141

a) Chứng minh: n=1984 giá trịlớn n đểsố 31 1008

4 +4 +4n sốchính phương b) Tìm sốnguyên dương x y z, , để: 4x+4y+4z sốchính phương

Bài 142 Cho sốnguyên dương n d ước sốnguyên dương

3n Chứng minh:

n +d sốchính phương chỉkhi d =3n2

Bài 143 Cho m n, sốnguyên dương lẻsao cho n2−1 chia hết cho m2−n2+1 Chứng

minh rằng: 2

1

(120)

CH

UY

ÊN

Đ

S

H

C

A. KiÕn thøc cÇn nhí 1 Định nghĩa

Cho a b, số nguyên n sốnguyên dương Ta định nghĩa a đồng dư với b theo môđun n kí hiệu là: ab(modn), a b có sốdư chia cho n Chú ý: a) a≡b(mod m) đồng dư thức với a vế trái, b vế phải

b) a≡b(mod m) ⇔ a – b  m ⇔ ∃ ∈t Z cho a = b + mt

c) Nếu a b không đồng dư với theo môđun m ta ký hiệu : a ≡/ b (mod m)

d)Nếu a chia cho br ar(modb) 2 Tính chất

1 Tính chất phản xạ: a ≡ a (mod m)

2 Tính chất đối xứng : a ≡ b (mod m) ⇒ b ≡ a (mod m) Tính chất bắc cầu :

a ≡ b (mod m); b ≡ c (mod m) ⇒ a ≡ c (mod m) Cộng hay trừ vế đồng dư thức có mơđun : a ≡ b (mod m) ; c ≡ d (mod m) ⇒ a ± c ≡ b ± d (mod m)

Tổng quát : aibi (mod m), i = 1; 2; ; k ⇒ a1 ±a2 ± ± ak = ±b1 b2 ± ± bk (mod m) a) Nhân hai vế đồng dư thức với số nguyên :

a ≡ b (mod m) ⇒ ka ≡ kb (mod m) với k ∈Z

b) Nhân hai vếvà môđun đồng dư thức với sốnguyên dương: a ≡ b (mod m) ⇒ ka ≡ kb (mod km) với k ∈N*

6 Nhân vế nhiều đồng dư thức có môđun : a ≡ b (mod m) ; c ≡ d (mod m) ⇒ ac ≡ bd (mod m)

Tổng quát aibi (mod m), i = 1; 2; ; k ⇒ a a1 akb b1 bk (mod m) Nâng hai vế đồng dư thức lên lũy thừa : a ≡ b (mod m) ⇒ ak ≡ bk(mod m) (k ∈N*)

CH

Đ

5 ỨNG DỤNG ĐỒNG DƯ THỨC

(121)

CH

IN

H

P

H

C

K

T

H

I H

C S

IN

H

GI

I C

ẤP

H

AI

8 Nếu hai sốđồng dư với theo nhiều mơđun chúng đồng dư với theo môđun BCNN môđun ấy:

a ≡ b (mod mi), i = 1; 2; ; k ⇒ a ≡ b (mod [m m1; 2; ;mk]) Đặc biệt (m mi, j)=1 (i, j = 1; 2; ; k)

a ≡ b (mod mi) ⇒ a ≡ b (mod m m1 mk)

9 Nếu a ≡ b (mod m) tập hợp ước chung a m tập hợp ước chung b m

Đặc biệt : a ≡ b (mod m) ⇒ (a, m) = (b, m)

10 Chia hai vếvà môđun đồng dư cho ước dương chung chúng : a ≡ b (mod m) , k ∈ UC(a,b,m), k > ⇒ a b modm

k k k

 

≡  

  Đặc biệt : ac ≡ bc (mod m) ⇒ a ≡ b mod m

(c, m)

 

 

 

B CÁC DẠNG TOÁN THƯỜNG GẶP

Dạng 1: Sử dụng đồng dư thức toán chứng minh chia hết

* Cơ sở phương pháp: Khi sốdư phép chia a cho m a  m Như để chứng tỏa  m ta chứng minh a ≡ (mod m)

* Ví dụ minh họa:

Bài toán Chứng minh rằng: ( 5555 2222) 2222 +5555 7

Hướng dẫn giải

Ta có: 2222≡3 mod 7( ) hay 2222≡ −4 mod 7( )⇒22225555 ≡ −( ) (4 5555 mod 7)(*) Mặt khác ( ) 2222 2222( )

5555≡4 mod ⇒5555 ≡4 mod (**) Từ(*) (**)

( ) ( ) ( )

( ) ( )( )

5555

5555 222 2222

5555 222 2222 3333

2222 5555 4 mod

2222 5555 4 mod

 

⇒ + ≡ − + 

⇒ + ≡ − −

Ta lại có: 3333 ( )3 1111 1111

4 = =64 mà 64≡1 mod 7( )⇒43333≡1 mod 7( )

( ) ( ) ( )

3333 2222 3333

4 mod 4 mod

⇒ − ≡ ⇒ − − ≡

Do ( 5555 2222) ( )

2222 +5555 ≡0 mod hay (22225555+55552222)7 Bài toán Chứng minh rằng: ( )

7.5 n 12.6n 19

(122)

CH

UY

ÊN

Đ

S

H

C

Hướng dẫn giải

Ta có:

( )

( ) ( ) ( ) ( )

( )

2

5 25 7.25 12.6

25 mod19 25 mod19 7.6 12.6 mod19 19.6 mod19 mod19 19

n

n

n n n n

n n n n

A

A A

A A

= = ⇒ = +

≡ ⇒ ≡ ⇒ ≡ + ⇔ ≡

⇒ ≡ ⇒ 

Bài toán Chứng minh 122n+1 + 11n+2  133 ( n ∈ N)

Hướng dẫn giải

Cách 1:Ta có 122= 144 ≡ 11(mod 133) ; 112= 121 ≡ –12(mod 133) Do 122n+1= 12 ( )2 n

12 ≡ 12 11n (mod 133)

11n+2= 112 11n ≡ –12 11n(mod 133)

Do 122n+1 + 11n+2 ≡ 12 11n – 12 11n ≡ (mod 133) Vậy với n ∈ N 122n+1 + 11n+2  133

Cách 2: Ta có 122= 144 ≡ 11(mod 133) ⇒122n ≡ 11n(mod 133) (1) Mà 12 ≡ – 112(mod 133) (2) Nhân vế với vế (1) (2) ta có : 122n 12 ≡ 11n (– 112) (mod 133) ⇒ 122n+1 ≡ –11n+2(mod 133)

122n+1 + 11n+2 ≡ (mod 133) hay 122n+1 + 11n+2  133 Bài toán Chứng minh rằng: ( ) ( )

2

2 n

A= +  ∀ ∈n N

Hướng dẫn giải

Ta có ( )

2 = ≡8 mod Ta tìm sốdư

2 n chia cho (đây điểm mấu chốt tốn)

Vì ( ) ( ) ( )

4≡1 mod ⇒4n ≡1 mod ⇒2 n ≡1 mod hay n chia cho dư Giả sử: ( )

2 n =3k+1 kN

Khi ta có:

2 k 2.8k

A= + + = +

Vì 8k ≡1 mod 7( )⇒2.8k ≡2 mod 7( )⇒2.8k + ≡ +5 mod 7( )

( )

0 mod A

⇒ ≡

(123)

CH

IN

H

P

H

C

K

T

H

I H

C S

IN

H

GI

I C

ẤP

H

AI

Dạng 2: Sử dụng đồng dư thức tìm số dư

* Cơ sở phương pháp: Với hai sốnguyên a m, m > ln có cặp số ngun q, r cho a = mq + r, 0≤ <r m Để tìm sốdư r phép chia a cho m ta cần tìm r cho

a r(mod m) r m

≡   ≤ <

* Ví dụ minh họa:

Bài tốn 1.Tìm sốdư chia 2000

3 cho

Hướng dẫn giải

Ta có

( ) ( ) ( )

( ) ( ) ( )

3

2

333

6 1998

3 mod 3 mod mod mod

≡ ⇒ ≡ ≡

⇒ ≡ ⇔ ≡

Mặt khác ( ) 2000 1998 ( )

3 ≡2 mod ⇒3 ≡3 ≡1.2 mod ⇒32000: dư Nhận xét: Để tìm số dư chia n

a cho b>0, ta lấy lũy thừa với số mũ tăng dần a chia cho b để tìm sốdư Ta dừng lại để xem xét tìm sốdư có giá trị tuyệt đối nhỏ giá trịđặc biệt có liên quan đến tốn

Bài tốn Tìm sốdư phép chia 70 50

5 +7 cho 12

Hướng dẫn giải

Ta có

( ) ( ) ( ) ( )( )

( ) ( ) ( ) ( )( )

35

2 70

25

2 50

5 mod12 mod12 mod12 * mod12 mod12 mod12 **

≡ ⇒ ≡ ⇔ ≡

≡ ⇒ ≡ ⇔ ≡

Từ ( ) ( )* ; ** ⇒ 570 +750 cho 12 dư

Bài tốn Tìm sốdư số 2005 2005

3

A= + chia cho 11

Hướng dẫn giải

Ta có ( ) ( )5 401 ( ) 2005 ( )( )

3 =243 mod11≡ ⇒ ≡1 mod11 ⇒3 ≡1 mod11

Mặt khác ( ) ( )5 401 ( ) 2005 ( )( ) =1024≡1 mod11 ⇒ ≡1 mod11 ⇒4 ≡1 mod11 Từ ( ) ( )1 ; ⇒ sốdư số A=32005+42005 chia cho 11

(124)

CH

UY

ÊN

Đ

S

H

C

Hướng dẫn giải

a) Ta có 1532 = 9.170 + ≡ (mod 9) 15325 ≡ 25(mod 9)

⇒ 15325 – ≡ 25 – (mod 9) Vì 25 – = 31 ≡ (mod 9) Do 15325 – ≡ (mod 9) Vậy sốdư cần tìm

b) Ta có 2016 ≡ (mod 5) 20162018≡ 12018(mod 5)

suy 20162018+ ≡ 12018+ (mod 5) Vì + = ≡ (mod 5) Do 20162018+ ≡ (mod 5)

Vậy sốdư cần tìm

Dạng 3: Tìm điều kiện biến để chia hết

* Cơ sở phương pháp: Dựa vào tính chất đồng dư thức sốdư đểtìm điều kiện ẩn để biểu thức chia hết

* Ví dụ minh họa:

Bài tốn 1.Tìm số tự nhiên n cho: a (23n+4+32n+1)19 b (n.2n +1 3)

Hướng dẫn giải

a Ta có

2 n+ +3 n+ =16.8n+3.9n

Vì 16≡ −3 mod19( )⇒16.8n ≡ −3.8n(mod19)

( ) ( ) ( )

( ) ( )

16.8 3.9 19 3.9 mod19 mod19 mod19

0

n n n n

n n n n

n

⇒ + ⇔ − + ≡

⇔ − ≡ ⇔ ≡

⇒ =

vì trái lại 9n ≡8n(mod19)⇒ ≡9 mod19( ) vô lý Vậy n=0

b.Ta xét trường hợp sau Trường hợp

Nếu n=3k k( ∈N)⇒n.2 3n ⇒n.2n+1 3 ⇒ loại Trường hợp

Nếu ( ) ( ) 3 3

3 2n k k k 2k 2.8k

n= k+ kNn + = k+ + + = k + + + + = k + + +

( )

( ) ( ) ( )

.2 2.8

8 mod mod

n k

k k

n

⇒ + ⇔ +

≡ − ⇒ ≡ −

 

( ) ( )

2.8k k mod

⇒ +  ⇔ − + ≡

(125)

CH

IN

H

P

H

C

K

T

H

I H

C S

IN

H

GI

I C

ẤP

H

AI

Nếu

( ) ( )

( ) ( ) ( )

3

3 3

1

3 2 2

3 2.2

.2 1 mod

n k

k k k k

k n

n k k N n k

k k

n

+

+ + + +

+

= + ∈ ⇒ + = + +

= + + = + +

⇒ +  ⇔ − + ≡

⇒ k+1 lẻ k =2m m( ∈N)⇒ =n 6m+2(mN)

Vậy điều kiện cần tìm m≡1 mod 6( ) m≡2 mod 6( )

Bài tốn 2.Tìm số tự nhiên n có chữ sốsao cho chia n cho 131 dư 112 chia n cho 132 dư 98

Hướng dẫn giải

( ) ( )( )

( )

( ) ( )

( )( )

98 mod132 132 98

132 98 112 mod131

98 33 112 33 mod131 14 mod131

131 14

n n k k N

k k

k m m N

≡ ⇒ = + ∈

⇒ + ≡

⇒ + + = + ⇒ ≡

⇒ ≡ + ∈

Từ(1) (2) n=131.132m+1946⇒ =n 1946

Dạng 4: Tìm chữ số tận

* Cơ sở phương pháp:

Nếu ar(mod10 ;0) ≤ <r b r chữ số tận a Ta cần lưu ý số tính chất sau:

Tính chất 1

Nếu a có chữ số tận 0;1;5;6 an có chữ số tận a nghĩa

(mod10)

n

aa

Tính chất 2

Nếu a có chữ số tận 4;9 a2 có chữ số tận 6;1

Nghĩa là: Nếu ( ) ( ) ( )

4 mod10 mod10 k mod10

a≡ ⇒a ≡ ⇒a

Nếu ( ) ( ) ( )

9 mod10 mod10 k mod10

a≡ ⇒a ≡ ⇒a

Do để tìm chữ số tận n

a ta chia n cho Tính chất

Nếu a có chữ số tận 2;3;7;8 ta áp dụng kết quảsau:

( ) ( ) ( ) ( )

4 4

2 k ≡6 mod10 ;3 k ≡1 mod10 ;7 k ≡1 mod10 ;8 k ≡6 mod10 Do để tìm chữ số tận n

(126)

CH

UY

ÊN

Đ

S

H

C

* Ví dụ minh họa:

Bài toán 1. Cho số 2013 2012

A= tìm chữ số tận A

Hướng dẫn giải

Ta có 2013=4.503 1+

Vì ( ) ( )

2012≡2 mod10 ⇒2012 ≡6 mod10

( ) ( ) ( )

( ) ( )

503

4 2012

2013 2013

2012 mod10 2012 mod10 2012 6.2 mod10 2012 mod10

⇒ ≡ ⇔ ≡

⇒ ≡ ⇔ ≡

Vậy A có chữ số tận Bài toán 2. Cho

1986 1978

B= tìm chữ số tận B

Hướng dẫn giải

( ) ( )

( ) ( )

( )

4

8

4

1978 mod10 1978 mod10 1986 mod 1986

1978 k mod10

k k N

C

≡ ⇒ ≡

≡ ⇒ = ∈

⇒ = ≡

Vậy chữ số tận B

Dạng 5: Tìm hai chữ số tận

* Cơ sở phương pháp: Nếu ar(mod100 ;10) ≤ <r 100 r chữ số tận a Ta cần lưu ý số tính chất sau:

( ) ( ) ( )

( ) ( )

( ) ( )( )

20 20

6

2 76 mod100 ;3 01 mod100 ;6 mod100 01 mod100 ;5 25 mod100

76n 76 mod100 ; 25n 25 mod100 n

≡ ≡ ≡

≡ ≡

≡ ≡ ∀ ≥

Từđó ta có:

( ) ( )

( ) ( )

( ) ( )

( )

20

20

20

20

0 mod10 01 mod100 1;3;7;9 mod10 01 mod100 mod10 25 mod100 2; 4;6;8 76 mod100

k k k k

a a

a a

a a

a a

 ≡ ⇒ ≡

≡ ⇒ ≡

 

≡ ⇒ ≡

 ≡ ⇒ ≡

Do đểtìm hai chữ số tận n

a ta chia n cho 20 * Ví dụ minh họa:

Bài tốn 1. Cho số 2013 2012

A= tìm hai chữ số tận A

Hướng dẫn giải

(127)

CH IN H P H Ụ C K Ỳ T H I H Ọ C S IN H GI Ỏ I C ẤP H AI ( ) ( ) ( ) ( ) ( )( ) 20 100 20 2000

2013 20.100 13

2012 mod10 2012 76 mod100

2012 76 mod100 2012 76 mod100

= +

≡ ⇒ ≡

⇒ ≡ ⇔ ≡

Mặt khác ( ) ( ) ( )

( ) ( ) ( )( )

6 6

6 12 2013

2012 12 mod100 2012 12 mod100 2012 84 mod100

2012 56 mod100 2012 56 mod100 2012 72 mod100

≡ ⇒ ≡ ⇒ ≡

⇒ ≡ ⇒ ≡ ⇒ ≡

Từ(1) (2) 2013 2000 2013 ( ) 2013 ( )

2012 2012 2012 76.72 mod100 2012 72 mod100

⇒ = ≡ ⇔ ≡

Vậy A có hai chữ số tận là: 72

Bài toán 2.Tìm hai chữ số tận sốsau

a 79

9

A= b B=2992012 c C =197819868

Hướng dẫn giải

a Vì ( )

7 ≡01 mod100 nên ta tìm sốdư chia 979 cho Ta có ( ) ( ) ( ) ( ) ( ) ( ) 9 9 7 7

9 4

9 mod mod

7 k 7 k 7.01 mod100 07 mod100

k k N

A +

≡ ⇒ ≡ ⇒ = ∈

⇒ = = = ≡ ⇒ ≡

Vậy A có hai chữ số tận 07 b Vì 10 ( )

29 ≡01 mod100 ⇒nên ta tìm sốdư chia 92012 cho 10 Ta có :

( ) ( ) ( )

( ) ( ) ( )

2012 2012

10 10

9 mod10 mod10 10

29 k 29 29 k 29.01 mod100 29 mod100

k k N

B + B

≡ − ⇒ ≡ ⇒ = + ∈

⇒ = = ≡ ⇔ ≡

Vậy B có hai chữ số tận 29

c Vì ( ) 20 ( ) 20 ( )

6 mod10 76 mod100 m 76 mod100

C ≡ ⇒C ≡ ⇒C

Mặt khác

( ) ( )

( ) ( )

8

20 20 16 16

1986 mod 20 1986 16 mod 20

1978 k 1978 k.1978 1978 76 mod100

C +

≡ ⇒ ≡

⇒ = = ≡

Ta lại có :

( ) ( ) ( ) ( )

( )

( ) ( )

4

4 4

16

1978 22 mod100 1978 56 mod100 1978 56 mod100 1978 76 mod100

96.76 mod100 76 mod100

C C

≡ − ⇒ ≡ ⇒ ≡

⇒ ≡

⇒ ≡ ⇔ ≡

(128)

CH

UY

ÊN

Đ

S

H

C

Dạng 6: Sử dụng đồng dư thức toán số phương

* Cơ sở phương pháp:

Sốchính phương số có dạng 2( )

n nN

Ta chứng minh số tính chất sốchính phương đồng dư : Sốchính phương chia cho chỉcó hai sốdư

Thật ta xét trường hợp sau

Với ( ) 2( ) ( )

3 mod mod mod

n= k⇒ ≡nn ≡ ⇔ n ≡ sốdư

Với ( ) ( ) (2 ) ( )

3 1 mod mod mod

n= k± ⇒ ≡ ±nn ≡ ± ⇔n ≡ ⇒ sốdư

2 Sốchính phương chia cho chỉcó hai sốdư Chứng minh tương tự :

Với ( ) 2( ) ( )

4 mod mod mod

n= k⇒ ≡nn ≡ ⇒ ≡n ⇒ sốdư

Với ( ) 2 ( )2 2 ( )

4 1 mod mod mod

n= k± ⇒ ≡ ±nn ≡ ± ⇔n ≡ ⇒ sốdư

Với ( ) 2 ( ) ( )

4 2 mod 4 mod mod

n= k+ ⇒ ≡nn ≡ = ⇔n ≡ ⇒ sốdư

3 Sốchính phương chia cho chỉcó ba sốdư 0,1 Tương tựta xét trường hợp sau :

( ) ( )

( ) ( )

( ) ( ) ( )

( ) ( ) ( ) ( )

( ) ( ) ( )

2

2

2

2

2

2 2

8 mod mod

8 1 mod mod

8 2 mod mod

8 3 mod mod mod

8 4 mod mod mod

n k n n

n k n n

n k n n

n k n n n

n k n n n

= ⇒ ≡ ⇒ ≡

= ± ⇒ ≡ ± ⇒ ≡

= ± ⇒ ≡ ± ⇒ ≡ ± =

= ± ⇒ ≡ ± ⇒ ≡ ± ⇔ ≡

= + ⇒ ≡ ⇒ ≡ ⇔ ≡

Hoàn toàn tương tự ta có thểxét trường hợp sốdư sốchính phương chia cho 5,7,9

* Ví dụ minh họa:

Bài toán 1. Chứng minh số : A=19k +5k +1995k +1996k với k chẵn số phương

Hướng dẫn giải

Với k chẵn ta có

( ) ( ) ( )

( ) ( ) ( )

( ) ( )

5 19 mod 19 mod 1995 mod 1995 mod

1996 mod 19 1995 1996 mod

k

k k

k k

k k k k k

A

≡ − ⇒ ≡

≡ − ⇒ ≡

≡ ⇒ = + + + ≡

(129)

CH

IN

H

P

H

C

K

T

H

I H

C S

IN

H

GI

I C

ẤP

H

AI

Bài tốn 2.Tìm tất số tự nhiên x,y để 2x + 5y số phương

Hướng dẫn giải

Giả sử ( )

2x +5y =k kN

Nếu x=0 5y

k

+ = k chẵn

k

⇒ chia hết cho 5+ ychia dư Vậyx≠0 , từ 5+ y =k2 ⇒k lẻ k không chia hết cho Xét hai trường hợp

+) Với 2 ( )2

2x

k n

+ = = + (vì k lẻ nên k=2n+1,nN)

2x (n n 1) n

⇒ = + ⇒ = Khi x =3; y=0 (thỏa mãn)

Thử lại:

2x+5y =2 +5 =9 số phương

+) Với y≠0 k không chia hết cho ⇒k2 ≡ ±1(mod 5)

Từ

2x+5y =k ⇒2x≡ ±1(mod 5) ⇒xchẵn

Đặt x=2x1 (x1∈N), ta có 1

5y =(k+2 )(x k−2 )x 1 2 5 x y

x y

k k

 + =

 ⇒ 

− =

 với y1+y2 = y với y1> y2, y1, y2là số tự nhiên 1 2

2

2x+ (5y yy 1) 5y y

⇒ = − ⇒ = ⇒ =

1

y y

⇒ = Khi 2x1+1=5y−1

Nếu y= 2t(tN) 2x1+1=52t− =1 25t −1 3 , vơ lý Vậy y lẻ, 2x1+1=5y− =1 4(5y−1+5y−2+ + + 1) Nếu y>1 5y−1+5y−2+ + 1,lẻ (vô lý)

Nếu y= ⇒ =1 x1 1khi x=2;y=1

Thử lại

2x+5y =2 + =5 số phương

Vậy x=2;y=1 x = 3, y =

Bài toán 3. Giả sử 2n+1 3n+1 sốchính phương Chứng minh 5n+3

một hợp số

Hướng dẫn giải

Giả sử

2n+ =1 a 3n+ =1 b2 với a b, ∈*

Khi ( ) ( ) 2

(130)

CH

UY

ÊN

Đ

S

H

C

Do ( )

1 d2

amo nên a2 ≡1(mod 4) Suy n≡0 mod 2( ) b≡1 mod 2( ) Do 2a b− >1

và 2a b+ >1 Vậy 5n+3 hợp số

Bài tốn 3.Tìm nghiệm ngun dương xđể 3x+171 sốchính phương

(HSG Lai Châu 2015 - 2016)

Hướng dẫn giải

Ta có: 3x ≡1, 3(mod8); y2 ≡0,1, 4(mod8) Mà:

3x+171=y ⇒3x ≡1(mod8) Do đó: x có

dạng 2k (k∈)

Phương trình trở thành ( )2 2

3 171

= k + =

A y với k = 0, 1, phương trình vơ nghiệm nên

nếu phương trình có nghiệm nghiệm phải ≥3 Do theo nguyên lý kẹp ta

có: ( )2 ( )2

3 3

 +  ≥ >

 

ka k

Khi đó: ( )2

3

 

= + 

k

A ( )

2

3

 

= + 

k

A

Giải trường hợp ta k = ⇒ = ⇒ =x y 30 Vậy x =

Dạng 7: Sử dụng đồng dư thức toán số nguyên tố, hợp số

* Cơ sở phương pháp: Đối với nhiều toán số nguyên tố hợp số ngồi sử dụng tính chất số ngun tốchúng ta cịn phải vận dụng tính chất đồng dư thức định lý Fermat

* Ví dụ minh họa:

Bài tốn 1.Tìm tất số nguyên tố p cho p2+14 số nguyên tố

Hướng dẫn giải

Ta xét hai trường hợp sau Trường hợp

Với

3 14 23

p= ⇒ p + = số nguyên tố Trường hợp

Với ( ) ( )

3 mod 14 14

p≠ ⇒ p ≡ ⇒ p +  p + > ⇒ p2+14 số nguyên tố Vậy p=3

Bài toán 2. Chứng minh với số nguyên tố p tồn vô số số tự nhiên n cho 2nn p

(131)

CH

IN

H

P

H

C

K

T

H

I H

C S

IN

H

GI

I C

ẤP

H

AI

Ta xét hai trường hợp sau Trường hợp

Nếu p= ⇒2 2nn2(∀ =n ;k kN) Trường hợp

Nếu ( )

2 2p mod

p> ⇒ − ≡ p

Theo định lý Fermat ( 1) ( ) ( )( )

2 pk p k k modp k N

⇒ − − ≡ + ∀ ∈

Do với số tự nhiên n có dạng ( )( )( *)

1

n= phpkN

Ta có 2n− ≡ +n (hp− ≡1) (0 modp) tức 2nn pBài toán 3. Cho *

nN chứng minh rằng: 19.8n+17 hợp số

Hướng dẫn giải

Ta xét trường hợp sau Trường hợp

Nếu ( )2 ( )

2 19.8n 17 1 k mod 19.8n 17

n= k⇒ + ≡ − + = ≡ ⇒ + 

Mặt khác 19.8n+17> ⇒3 19.8n+17 hợp số Trường hợp

( )2 ( )

4

4 19.8n 17 19.8 k 17 19.8.64 k 17 6.8 k 52 mod13

n= k+ ⇒ + = + + = + ≡ − + ≡ ≡ Mà

19.8n+17 > ⇒3 19.8n +17 hợp số

Trường hợp

( ) ( )2 ( )

4

4 19.8 17 19.8 17 19.8.64 17 mod 19.8 17

k

n k k

n

n= k+ ⇒ + = + + = + + ≡ − − + + ≡ ≡

⇒ + 

Mà 19.8n+17> ⇒5 19.8n+17 hợp số

Bài toán 4. Cho p số nguyên tố lớn Chứng :(3p −2p −1 42) p

Hướng dẫn giải

Ta có 42p=2.3.7.9 đề chứng minh A=3p −2p −1chia hết cho 42p ta cần chỉra A chia hết cho 2,3,7

Thật

Ta có A≡1p − − =0 mod 2( )⇒ A2

p số nguyên tố lớn nên p số lẻ :

( )

2

2 3p k 1k 1.2 mod 3

(132)

CH

UY

ÊN

Đ

S

H

C

Mặt khác 2 2 ( )( )( )

3 k k 3.9k k 3.2k k 2k k mod

A= + − + − = − + − ≡ − + − = − − + −

Do p=2k+3 không chia hết cho 3⇒k3 k+1 3 Ta xét trường hợp sau:

Trường hợp

Nếu k=3h h( ∈N)⇒2k − =1 8h−1 7 Trường hợp

Tương tự

1 2k

k+  ⇒ + − 

Vậy trường hợp ta có A7

Theo định lý Fermat ta có A=3p−2p− =1 (3p− −3) (2p−2)p Từđó suy điều phải chứng minh

Dạng 8: Sử dụng đồng dư thức tốn giải phương trình nghiệm ngun

* Cơ sở phương pháp: Trong giải phương trình nghiệm ngun việc lựa chọn mơđun cách thích hợp giúp việc giải phương trình khó phức tạp trở nên đơn giản Đặc biệt tốn chứng minh phương trình nghiệm ngun vơ nghiệm * Ví dụ minh họa:

Bài tốn 1. Chứng minh phương trình sau khơng có nghiệm ngun:

a)x2 – y2= 1998 b) x2+ y2= 1999

Hướng dẫn giải

- Nhận xét: Sốchính phương chia cho có sốdư

a) Ta có: ( )

( ) ( )

2

2

2

x 0,1 mod

x y 0,1,3 mod

y 0,1 mod

≡  ⇒ − ≡

≡ 

Mà 1998 chia cho dư 2, nên phương trình khơng có nghiệm ngun

b) Ta có: ( )

( ) ( )

2

2

2

x 0,1 mod

x y 0,1,2 mod

y 0,1 mod

≡  ⇒ + ≡

≡ 

Mà 1999 chia cho dư 3, nên phương trình khơng có nghiệm ngun Bài tốn 2. Giải phương trình nghiệm nguyên: = 2− +

x 2y 8y (1)

Hướng dẫn giải

Ta có: (1) ⇔ = − 2−

x 2(y 2)

- Nhận xét: Sốchính phương chia cho có sốdư 0,

Ta có: ( )

(133)

CH

IN

H

P

H

C

K

T

H

I H

C S

IN

H

GI

I C

ẤP

H

AI

( ) ( ) ( ) ( )

( ) ( ) ( )

2

2

y 0,1,4 mod y 0,2 mod

2 y 3,5 mod

5 mod

− ≡ ⇒ − ≡  ⇒

− − ≡

− ≡ 

Suy phương trình khơng có nghiệm ngun

Bài tốn 3.Phương trình z2 =(x2 −1).(y2− +1) 2013 có nghiệm nguyên dương hay

khơng?

Hướng dẫn giải

Ta có:

( )( )

2

2

2

x 0,1, 4(mod 8) x 0,3,7(mod 8)

x y 0,1,5(mod 8) y 0,1, 4(mod 8) y 0,3,7(mod 8)

2013 5(mod 8)

 

≡ ⇒ − ≡  ⇒ −

− ≡ 

≡ ⇒ − ≡  

≡ 

(x2 1 y)( 1) 2013 5,6,2(mod 8)

⇒ − − + ≡

Mà z2 ≡0,1, 4(mod 8)

Suy phương trình khơng có nghiệm ngun

Dạng 9: Sử dụng định lý (ta thừa nhận không chứng minh)

* Cơ sở phương pháp:

1 Định lý Fermat bé Cho a sốnguyên dương p số nguyên tố Khi ta ln có p

aa (mod p) Đặc biệt (a, p) =1thì ap−1 ≡1 (mod p)

2 Định lý Wilson Với số nguyên tố p (p – 1)! ≡–1(mod p)

3 Định lý Euler Cho m sốnguyên dương a số nguyên tốcùng với m; (m)

ϕ số sốnguyên dương nhỏhơn m nguyên tố với m Khi (m)

aϕ ≡1 (mod m)

Chú ý: Nếu sốngun dương m có dạng phân tích thành thừa số nguyên tố: m = k

1 k

p p pα α α ϕ(m)=

1 k

1 1

m 1

p p p

    

− − −

    

    

* Ví dụ minh họa:

Bài toán Cho a b, ∈Z a b;( ), =1 Chứn minh : a3−2b3 không chia hết cho 19

Hướng dẫn giải

Ta chứng minh phản chứng sau: Giả sử ( 3)

2 19

(134)

CH

UY

ÊN

Đ

S

H

C

Mặt khác ( ) ( )3 6 18 18

2 64

ab =ab Nếu a b, không chia hết cho 19 theo định lý

Fermat (Định lý Fermat: ( ) ( )

mod mod

p p

aa pa − ≡ p Với a nguyên p nguyên tố).

( ) ( )

18 18 18 18

1 mod19 64 64 63 mod19

a b a b

⇒ ≡ ≡ ⇒ − ≡ − = − ≡ (Vô lý)

Nếu hai sốchia hết cho 19 từ ( 3) 19 19

19 a

a b

b

− ⇒ ⇒

 

 vơ lý ( )a b, =1

Vậy 3

ab khơng chia hết cho 19

Bài tốn Chứng minh với số tự nhiên n : 34 24

2 n+ +3 n+ +2007chia hết cho 22

Hướng dẫn giải

Theo Định lý Fermat bé ta có 210 ≡ 1(mod 11) ; 310 ≡ 1(mod 11) Ta có 34= 81 ≡ 1(mod 10) ⇒ 34n+1 = (34)n ≡ 3(mod 10)

⇒ 34n+1= 10k + , (k ∈ N)

Mặt khác 24= 16 ≡ (mod 5) ⇒24n ≡ 1(mod 5)

⇒ 24n+1 = 2.(24)n≡ (mod 10) ⇒ 24n+1= 10t + , (t ∈ N) Do n n

3 10k 10t

2 + +3 + +2007=2 + +3 + +2002 5+

3( )10 k 2( )10 t

2 22.91

= + + + ≡ 23+ 32+ + ≡ (mod 11)

Mà n n

3

2 + +3 + +2007 (vì 34 n

2 + số chẵn 324 n 1+ số lẻ 2007là số lẻ) Do (2 ; 11) = nên n n

3

2 + +3 + +2007  22

Bài toán Cho a a1; 2; ;a2016 2016 sốnguyên dương Chứng minh điều kiện cần

và đủđể 5 5

1 2016

a +a + + +a a 30 a1 +a2 + +a201630

Hướng dẫn giải

Theo định lý Fermat bé , 2; 3; số nguyên tốvà a sốnguyên dương ta có :

a2 ≡ a (mod 2) ⇒ a4= (a2)2 ≡ a2 ≡ a (mod 2) ⇒ a5 ≡ a (mod 2) a3 ≡ a (mod 3) ⇒ a5= a3 a2≡ a.a2≡ a3≡ a (mod 3)

a5 ≡ a (mod 5)

Theo tính chất hai sốđồng dư với theo nhiều mơđun chúng đồng dư với theo mô đun BCNN mơđun

Do a5 ≡ a (mod 2.3.5) hay a5 ≡ a (mod 30) ⇒ a5 – a ≡ (mod 30)

Nghĩa ( 5 5 )

1 2016

a +a +a + + a – (a1 +a2 + +a2016) ≡0 (mod 30) Vậy a1+a2 + +a201630 ⇔

5 5

1 2016

(135)

CH

IN

H

P

H

C

K

T

H

I H

C S

IN

H

GI

I C

ẤP

H

AI

Bài toán Chứng minh số tự nhiên thếnào có số k cho 1983k – chia hết cho 105

(Đề thi học sinh giỏi tốn cấp tồn quốc năm 1983)

Hướng dẫn giải

Vì 1983 khơng chia hết cho không chia hết cho mà 105= 25.55nên (1983; 105) = Áp dụng định lý Euler ta có :

( )105 ( )

1983ϕ ≡1 mod 10

Ta có ( )5 1

10 10 1 4.10

2

  

ϕ =  −  − =

   Nghĩa

4

4.10

1983 −1  10

Vậy k = 104

B. BÀI TẬP ÁP DỤNG Bài Chứng minh 42018 – 

Bài 2: Chứng minh với số nguyên

( 2 ) ( ) (2 )

1 ,

n

A= nn + −nn− ∀ ∈n Z n>

Bài Chứng minh rằng: (9n+1) không chia hết cho 100(∀ ∈n N) Bài 4. Cho sốa = a an n 1− a a1 (1 a≤ n ≤9 ; 0≤ ≤ai 9; i = 0; 1; ; n –1) Hãy xác định dấu hiệu chia hết :

a) Cho 3; b) Cho

Bài 5. Chứng minh rằng: ( 2004 ) ( )

2003

1924 1920 124 *

n

A= +  ∀ ∈n N

Bài a) Hãy tìm chữ số tận 10

9 b) Hãy tìm hai chữ số tận 1000

3 Bài 7. Tìm sốdư phép chia

a) 8! – cho 11 b) 20142015 + 20162015 + 2018 cho c) 250 + 4165cho d) 15 + 35 + 55 + + 975 + 995 cho Bài 8. Tìm sốdư phép chia :

a)15325 – cho ; b)22000cho 25;

c) 2016

2015

2014 cho 13

Bài 9. Tìm sốdư phép chia :

a)A = 352 – 353 + 354 – 358 + 3516 + 3532 cho 425

b) B = 10 102 103 1010

(136)

CH

UY

ÊN

Đ

S

H

C

Bài 10. a) Tìm chữ số tận 32

4

b) Tìm hai chữ số tận 3999. c) Tìm ba chữ số tận số 2512. Bài 11. Chứng minh :

a) 412015 – 7 ; b) 24n+1 –  15 (n ∈N); c) 376 – 276  13 ; d) 2015 –  341

Bài 12. Chứng minh 189079+ 19452015+ 20172018  Bài 13 a) Chứng minh 55552222+ 22225555+ 155541111 

b) Cho M = 11969 69220 220119 102

220 +119 +69 +(220 119 69)+ +

Chứng minh M  102

Bài 14 Chứng minh 52n-1 2n+1 + 22n-1 3n+1  38 ( n ∈N*) Bài 15 Cho sốa = a an n 1− a a1 (1 a≤ n ≤9 ; 0≤ ≤ai 9; i = 0; 1; ; n –1) Hãy xác định dấu hiệu chia hết :

a) Cho 9; b) Cho 25; c) Cho 11; d) Cho

Bài 16 Cho A = 10 n

2 + +19 với n ∈ N*.Chứng minh A hợp số Bài 17 Cho B = ( )13

12! + 20162015 Chứng minh B chia hết cho 13 Bài 18 Chứng minh với n ∈ N :

a) n

2 3n

2 + +3.2 7 ; b) 24 n 5n 1 2n

2 + +2.12 + +5.10 11

Bài 19. a) Với giá trị số tự nhiên n 3n+ 63 chia hết cho 72 b) Cho A = 20n + 16n – 3n – Tìm giá trị tự nhiên n đểA323 Bài 20 Tìm số nguyên tố p thỏa mãn 2p +1p

Bài 21 Tìm tất số nguyên tốp cho p2+ 20 số nguyên tố Bài 22 Cho p số nguyên tố Chứng minh số p p

abbap với số nguyên

dương a, b

Bài 23.a) Chứng minh tổng bình phương ba số nguyên phép chia cho khơng thểcó dư

b) Chứng minh phương trình 2

4x + y +9z =2015khơng có nghiệm ngun Bài 24.Tìm hai chữ số tận 2009

2010

2011

(137)

CH

IN

H

P

H

C

K

T

H

I H

C S

IN

H

GI

I C

ẤP

H

AI

Bài 25. Cho biểu thức A = (a2012+ b2012+ c2012) – (a2008+ b2008+ c2008) với a, b, c số nguyên dương Chứng minh A chia hết cho 30

(Đề thi chọn học sinh giỏi mơn tốn lớp TP Hà Nội năm học 2011 – 2012) Bài 26 Chứng minh không tồn bộba sốnguyên (x; y; z) thỏa mãn đẳng thức

4 4

7

x +y = z +

(Đề thi vào lớp 10 trường THPT chuyên KHTN Hà Nội năm học 2011 – 2012) Bài 27 Tìm hai chữ số cuối số 106 2012

41 57

A= +

(Đề thi vào lớp 10 trường THPT chuyên KHTN, ĐHQG Hà Nội năm học 2012 – 2013) Bài 28 Cho a, b hai sốnguyên dương thỏa mãn a + 20 b + 13 chia hết cho 21 Tìm sốdư phép chia A=4a +9b + +a b cho 21

(Đề thi tuyển sinh lớp 10 THPT chuyên Trần Phú Hải Phòng năm học 2013 – 2014) Bài 29 Cho n sốnguyên dương chứng minh 3

2 n n

A= + + − + hợp số

(Đề thi học sinh giỏi lớp TP Hà Nội năm học 2014 – 2015)

Bài 30. Chứng minh 4 4

2012 n 2013 n 2014 n 2015 n

A= + + + khơng phải sốchính phương với sốngun dương n

(Đề thi tuyển sinh vào lớp 10 chuyên trường ĐHSP TP Hồ Chí Minh năm học 2015 – 2016) Bài 31. Chứng minh phương trình : 15 15 15 2003 2003 2003

19

x + y +z = + + khơng có nghiệm nguyên

Bài 32 Tìm nghiệm nguyên dương phương trình x x( + +3) (y y+3) (= z z+3) với điều kiện x y, số nguyên tố

Bài 33. Chứng minh ( 2016 2016 2016)10

2013 +2014 −2015 106 Bài 34. Chứng minh 4 4

1k +2 k +3 k +4 k không chia hết cho

Bài 35. Chứng minh với số nguyên tố p tồn vơ số số có dạng 2nn , (n ∈N) chia hết cho p

Bài 36.Tìm hai chữ số tận 62001 26 Bài 37.Tìm số tựnhiên n cho n

3 +4n 1+ chia hết cho 10 Bài 38 Tìm số tự nhiên n nhỏ lớn cho

n +4n −20n−48 125 Bài 39. Cho sốnguyên a không chia hết cho Chứng minh rằng:

( )( )

a −1 a +15a +1 35 Bài 40 Chứng minh m n

(138)

CH

UY

ÊN

Đ

S

H

C

Bài 41 Chứng minh số tự nhiên thếnào có sốk cho k

2017 −1chia hết cho

10

Bài 42 Tìm n nguyên dương đểphương trình sau có nghiệm hữu tỉ:

( ) (n )n

n

x + x+2 + 2−x =0

Bài 43. Gọi a tổng chữ số số ( )29 1945 Gọi b tổng chữ số số a Gọi c

(139)

CH

IN

H

P

H

C

K

T

H

I H

C S

IN

H

GI

I C

P H

AI

A. KiÕn thøc cÇn nhí

1 Giải phương trình nghiệm nguyên.

Giải phương trình f(x, y, z, ) = chứa ẩn x, y, z, với nghiệm nguyên tìm tất số nguyên (x, y, z, ) thỏa mãn phương trình

2 Một số lưu ý giải phương trình nghiệm nguyên.

Khi giải phương trình nghiệm nguyên cần vận dụng linh hoạt tính chất chia hết, đồng dư, tính chẵn lẻ,… để tìm điểm đặc biệt ẩn số biểu thức chứa ẩn phương trình, từ đưa phương trình dạng mà ta biết cách giải đưa phương trình đơn giản Các phương pháp thường dùng để giải phương trình nghiệm nguyên là:

• Phương pháp dùng tính chất chia hết

• Phương pháp xét số dư vế

• Phương pháp sử dụng bất đẳng thức

• Phương pháp dùng tính chất số phương

• Phương pháp lùi vô hạn, nguyên tắc cực hạn

B. MỘT SỐ PHƯƠNG PHÁP GIẢI PHƯƠNG TRÌNH NGHIỆM NGUYÊN

I. PHƯƠNG PHÁP DÙNG TÍNH CHIA HẾT

Dạng 1: Phát tính chia hết ẩn

Bài tốn Giải phương trình nghiệm nguyên 3x 17y 159+ = ( )1

Hướng dẫn giải

Giả sử x, y số nguyên thỏa mãn phương trình (1) Ta thấy 159 3x chia hết cho

3 nên 17y 3 ⇒y 3 (do 17 nguyên tố nhau)

Đặt y 3t t Z= ( ∈ ) thay vào phương trình ta 3x 17.3t 159+ = ⇔ +x 17t 53.=

CH

Đ

(140)

CH

UY

ÊN

Đ

S

H

C

Do đó: x 53 17t(t Z)

y 3t

 = −

 =

 Thử lại ta thấy thỏa mãn phương trình cho

Vậy phương trình có nghiệm (x, y) = (53 – 17t, 3t) với t số nguyên tùy ý Bài tốn Tìm nghiệm ngun phương trình 2x 13y 156+ = (1)

Hướng dẫn giải

- Phương pháp 1: Ta có 13y 13 156 13 nên 2x 13 ⇒x 13 (vì (2,3) = 1) Đặt x 13k (k Z)= ∈ thay vào (1) ta được: y= −2k 12+

Vậy nghiệm nguyên phương trình là: x 13k (k Z)

y 2k 12

 =

∈  = − +

- Phương pháp 2: Từ (1) x 156 13y 78 13y

2

⇒ = = − ,

Để x Z 13y Z

2

∈ ⇒ ∈ Mà (13,2) = 1⇒y 2 Đặt y 2t(t Z)= ∈ ⇒ =x 78 13t−

Vậy nghiệm nguyên phương trình là: x 78 13t(t Z)

y 2t

 = −

∈  = −

Bài tốn Giải phương trình nghiệm ngun 23x 53y 109+ =

Hướng dẫn giải

Ta có x 109 53y 23(4 2y) 17 7y 2y 17 7y

23 23 23

− − + − −

= = = − +

Ta phải biến đổi tiếp phân số 17 7y 23

để cho hệ số biến y

Phân tích: Ta thêm, bớt vào tử số bội thích hợp 23

17 7y 17 7y 46 46 7(9 y) 46 2 7(9 y)

23 23 23 23

− − + − − − −

= = = − +

Từ x 2y 7(9 y)

23

= − + , Để x Z y Z

23

∈ ⇒ ∈ , (7,23) =

Đặt y 23t (t Z)− = ∈ ⇒ = −y 23t

Vậy nghiệm nguyên phương trình là: x 23t (t Z)

y 53t 16

 = −

∈  = − 

Chú ý: Phương trình có dạng ax by c+ = với a,b,c số nguyên

(141)

CH

IN

H

P

H

C

K

T

H

I H

C S

IN

H

GI

I C

P H

AI

-Rút gọn phương trình ý đến tính chia hết ẩn

-Biểu thị ẩn mà hệ số có giá trị tuyệt đối nhỏ (chẳng hạn x) theo ẩn - Tách riêng giá trị nguyên biểu thức x

- Đặt điều kiện để phân số biểu thức chứa x số nguyên t1, ta

phương trình bậc hai ẩn y t1

- Cứ tiếp tục làm ẩn biểu thị dạng đa

thức với hệ số nguyên

Bài tốn Tìm nghiệm ngun phương trình 11x 18y 120+ = ( )1

Hướng dẫn giải

Ta thấy 11 6x nên x6 Đặt x = 6k (k nguyên) Thay vào (1) rút gọn ta được: 11k + 3y = 20

Biểu thị ẩn mà hệ số có giá trị tuyệt đối nhỏ (là y) theo k ta được: 20 11

3

k y= −

Tách riêng giá trị nguyên biểu thức này:

3

k y= − k+ −

Lại đặt

3

k− = t với t nguyên suy k = 3t + Do đó:

4(3 1) 11

6 6(3 1) 18

y t t t

x k t t

= − + + = −

= = + = +

Thay biểu thức x y vào (1), phương trình nghiệm

Vậy nghiệm nguyên (10 biểu thị công thức:

18

3 11

x t

y t

= +

 = −

 với t số nguyên tùy ý

Chú ý: a) Nếu đề yêu cầu tìm nghiệm nguyên dương phương trình (1) sau tìm nghiệm tổng quát ta giải điều kiện: 18t t

3 11t 11

 + >

⇔ − < <  − >

Do t = t số nguyên Nghiệm nguyên dương (1) (x, y) = (6, 3).

Trong trường hợp tìm nghiệm nguyên dương (1) ta cịn giải sau: 11x + 18y = 120

Do y 1≥ nên 11x 120 18.1 102.≤ − =

Do x nguyên nên x 9≤ Mặt khác x 6 và x nguyên dương nên x = 6⇒ =y 3 b) Có nhiều cách tách giá trị nguyên biểu thức y= 20 11k− ,

3 chẳng hạn: k

y 4k

(142)

CH

UY

ÊN

Đ

S

H

C

1 2k y 3k

3

+

= − − (cách 2)

( ) k y 3k

3

= − + (cách 3)

Ta thấy: - Cách gọn cách ở cách hệsố k phân thức 1, sau đặt k t

3

= ta không cần thêm ẩn phụ

- Trong cách 3, nhờđặt thừa số chung mà hệsố k phần phân số -1, sau

đặt 1 k t

3

= cũng không cần dùng thêm thừa số phụ

Bài toán Tìm nghiệm nguyên dương phương trình: 6x2+5y2 =74

Hướng dẫn giải

Ta có: 6x2 +5y2 =74⇔6 x 4( 2− ) (=5 10 y− 2)( )2

Từ (2) suy 6 x 5( 2− ) , mặt khác ( )6,5 1= ⇒(x 52− ) ⇒x2 =5t t N+ ( ∈ )

Thay x 5t2− = vào (2) ta có: 30t 10 y= ( − 2)⇔y2 =10 6t−

Ta có: 2

4 t

5t 5

x 0,y t ,t N

10t t 5

3

 > − 

 + > 

> > ⇔ ⇔ ⇔ − < < ∈

− >

  <



Suy ra: t∈{ }0;1

Với t = khơng thỏa mãn u cầu tốn

Với t = ta có: x22 x

y

y

 =  = ±

 ⇔

  = ± =

 

 Mặt khác x, y nguyên dương nên x = 3, y =

Vậy phương trình có nghiệm (x, y) = (3, 2)

Dạng 2: Phương pháp đưa phương trình ước số * Cơ sở phương pháp:

Ta tìm cách đưa phương trình cho thành phương trình có vếlà tích biểu thức có

giá trị nguyên, vế phải số nguyên

Thực chất biến đổi phương trình dạng: A(x; y).B(x; y) c= trong A(x; y),B(x; y)

là biểu thức nguyên, c số nguyên

Xét trường hợpA(x; y),B(x; y) theo ước c * Ví dụ minh họa:

Bài tốn Tìm nghiệm ngun phương trình: 2xy x y 3− + =

(143)

CH

IN

H

P

H

C

K

T

H

I H

C S

IN

H

GI

I C

P H

AI

( ) ( ) ( )( )

− + = ⇔ − + =

⇔ − + − = −

⇔ − + =

2xy x y 4xy 2x 2y

2x 2y 2y

2y 2x

Ta gọi phương trình phương trình ước số: vế trái tích thừa số

nguyên, vế trái số Ta có x y số nguyên nên 2x + 2y – số

nguyên ước

(2x + 1) (2y - 1) ước số nên ta có:

2x + 1 -1 -5

2y -1 -5 -1

Vập phương trình có nguyện nguyên (x, y) = (3, 0); (-1, -2); (2, 1); (-3, 0)

Kinh nghiệm giải: Để đưa vế trái 2xy x y− + về phương trình dạng tích, ta biến đổi thành x 2y 1( ) (1 2y 1)

2

− + − cách nhân vế phương trình với để đưa vềphương trình ước số Luyện tập kinh nghiệm ví dụ2 sau đây.

Bài tốn Tìm nghiệm ngun phương trình: 5x 3y 2xy 11− = −

Hướng dẫn giải

( ) ( ) ( )( )

− = − ⇒ − + − − + =

  − +

⇔ −  + = ⇔ − = ⇔ − + =

 

3 15

5x 3y 2xy 11 x(5 2y) (5 2y) 11

2

3 2x

5 2y x 2y 2y 2x (*)

2 2

(2x + 3) (2y - 5) ước số nên ta có:

2x + -1 -7

2y - -7 -1

Vập phương trình có nguyện nguyên (x, y) = (-1, 6); (-2, -1); (2, 3); (-5, 2)

Nhận xét: Đối với nhiều phương trình nghiệm nguyên việc đưa vềphương trình ước sốlà rất khó khăn ta áp dụng số thủ thuật, bạn xem tiếp ví dụ3:

Bài tốn Tìm nghiệm ngun phương trình: x2 −2xy 3y 5x 0+ − + =

(144)

CH UY ÊN Đ Ề S Ố H Ọ C 2 2

2 2 2

(2y 5) (2y 5)

x 2xy 3y 5x x x(2y 5) 3y

4

2y 4y 20y 25 12y 28 2y 4y 8y

x x

2 4

+ − +

− + − + = ⇔ − + + + + + =

 +  − − − + +  +  + −

⇔ −  + = ⇔ −  −

   

2 2

2

2y 4(y 1) 2y

x x (y 1)

2 4

 +  + −  +  − ⇔ −  − = ⇔ −  − + =     ( ) ( ) ( ) ( )( ) ( )( ) − − − ⇔ − + = ⇔ − − − + = − ⇔ − − − − − − + + = − ⇔ − − − = − 2 2

2x 2y 7

(y 1) 2x 2y y

4

2x 2y 2y 2x 2y 2y 2x 4y 2x (*)

Vì x, y nguyên nên từ PT(*) ta có trường hợp sau:

1) 2x 4y

2x

 − − =  − = −  x y  = − ⇔  = −

 2)

2x 4y 7

2x

 − − = −  − =  x y  = ⇔  = 

3) 2x 4y

2x

 − − = −  − =  x y  = ⇔  =

 4)

2x 4y 7

2x

 − − =  − = −  x y  = ⇔  = − 

Vậy nghiệm nguyên (x; y) phương trình là: (-2; -3); (2; 1); (5; 1);(1; -3)

*Nhận xét: Trong cách giải ta sử dụng phương pháp biến đổi tam thức bậc hai(ax2+bxy cy ,ax+ 2+bx c+ )): trước hết ta chọn một biến để đưa về hằng đẳng thức (Bình

phương tổng, hiệu) chứa biến đó: ở đây ta chọn biến x là :

2

2 (2y 5)

x x(2y 5)

4

+

− + + , phần lại đa thức ta lại làm với biến y:

2

(2y 5) 3y 7

4

− +

+ + 4y 8y 32

4

+ −

= − 4(y 1) 72

4

+ − = − Các bạn có thểtư tìm hướng giải sau:

( ) ( )

2

x 2xy 3y 5x 0− + − + = ⇔x − 2y x 3y a a *+ + + + =

Xét phương trình: x2−(2y x 3y a * *+ ) + + + = ( )

Với a số chưa biết cần thêm vào, xác định a sau:

( ) ( ) ( )

2

**

2

2y 3y a

4y 20y 25 12y 28 4a

4y 8y 4a

∆ = + − + + = + + − − − = + − −

Chọn a để ∆( )** số phương nên 4a a

4

− − = ⇒ = :

( ) ( ) ( ) ( )

2

1

**

2y x 2y x 4y

4 x x ,x

2 2

+ − + + + + +

(145)

CH IN H P H Ụ C K Ỳ T H I H Ọ C S IN H GI Ỏ I C Ấ P H AI

Vậy: ( )* x x 4y 7 (2x 2x 4y 7)( )

2

 + 

 

⇔ −  − = − ⇔ − − − = −

  

Vì x, y ngun nên ta có trường hợp sau:

1) 2x 4y

2x

 − − =  − = −  x y  = − ⇔  = −

 2)

2x 4y 7

2x

 − − = −  − =  x y  = ⇔  = 

3) 2x 4y

2x

 − − = −  − =  x y  = ⇔  =

 4)

2x 4y 7

2x

 − − =  − = −  x y  = ⇔  = − 

Vậy nghiệm nguyên (x;y) phương trình là: (-2; -3); (2; 1); (5; 1);(1; -3)

Bài tốn Tìm nghiệm ngun phương trình x 12x y2+ = ( )1

Hướng dẫn giải

Phương trình tương đương với :

( )2 ( )( )

2 2

x 12x y+ = ⇔ x 6+ −y =36⇔ x y x y 6+ + − + =36

Suy (x + y + 6) (x – y + 6) ước 36

Mà 36 có 18 ước nên: (x y 6+ + ∈ ± ± ± ± ± ± ±) { 1; 2; 3; 4; 6; 9; 18; 36± }

Kết ta tìm nghiệm nguyên là:( ) (0,0 ; 12,0 ; 16,8 ; 16, ; 4,8 ; 4, 8− ) (− ) (− − ) ( ) ( − )

Nhận xét: Phương pháp đưa vềphương trình ước sốcó bước: Phân tích thành ước xét

các trường hợp Hai bước có thểkhơng khó trường hợp số phải xét có nhiều

ước số cần dựa vào tính chất biến (ví dụ: tính chẵn lẻ, số dư vế) để giảm số trường hợp cần xét

Trong trường hợp ví dụ ta có thể nhận xét sau:

Do y có số mũ chẵn nên y nghiệm – y nghiệm nên ta giả sử y 0≥ Khi

x y x y+ − ≤ + + ta giảm trường hợp:

6 6

, ,

6 36

6 36 6 18

, ,

6 18

6 12 6

, ,

6 12 6

6

6

x y x y x y

x y x y x y

x y x y x y

x y x y x y

x y x y x y

x y x y x y

(146)

CH

UY

ÊN

Đ

S

H

C

Bây có 10 trường hợp, ta lại thấy (x y+ + ) (+ x y+ − )=2y nên (x y , x y+ + ) ( + − )

cùng tính chẵn lẻ.Do ta cịn trường hợp:

x y x y 18 x y 6 x y 6

, , ,

x y 18 x y x y 6 x y 6

 + + = −  + + =  + + = −  + + =

 + − = −  + − =  + − = −  + − =

   

Tiếp tục xét hai phương trình x y 6, x y 6

x y 6 x y 6

 + + = −  + + =

 + − = −  + − =

  hai phương trình có nghiệm

y = ta có xét y = từđầu Ta có phương trình ban đầu: x x 12( + )=y2 , xét hai khảnăng:

Nếu y = x = x = - 12

Nếu y 0≠ x y x y+ − < + + áp dụng hai nhận xét ta phải xét trường hợp

x y x y 18

,

x y 18 x y

 + + = −  + + =

 + − = −  + − =

 

Giải kết luận phương trình có nghiệm ( ) (0,0 ; 12,0 ; 16,8 ; 16, ; 4,8 ; 4, 8− ) (− ) (− − ) ( ) ( − )

Dạng 3: Phương pháp tách giá trị nguyên

*Cơ sở phương pháp:

Trong nhiều tốn phương trình nghiệm ngun ta tách phương trình ban đầu thành phần có giá trị nguyên để dễdàng đánh giá tìm nghiệm, đa sốcác tốn sử dụng phương pháp này thường rút ẩn (có bậc nhất) theo ẩn cịn lại

*Ví dụ minh họa:

Bài tốn Tìm nghiệm ngun dương phương trình sau: xy 2y 3y 0− − + =

Hướng dẫn giải

Ta có xy 2y 3y 0− − + = ⇒y x 3( − )=2x 1.−

Ta thấy x = khơng nghiệm nên x 3≠ đó: = −

2x y

x

Tách phân thức −

2x

x giá trị nguyên:

( − )+ −

= = = +

− − −

2 x

2x

y

x x x

Do y số nguyên nên

5

x số nguyên, (x – 3) ước

+) x – = x = 4, y = + =

(147)

CH

IN

H

P

H

C

K

T

H

I H

C S

IN

H

GI

I C

P H

AI

+) x – = -5 x = -2 (loại)

Vậy nghiệm (x, y) (4, 7) , (8, 3)

Bài tốn Tìm số nguyên x y thoả mãn phương trình: x2 +xy 2y x 0− − − =

Hướng dẫn giải

Nhận xét: phương trình ẩn y có bậc nên rút y theo x Ta có: x2+xy 2y x 0− − − = ⇔y x 2( − )= − + +x2 x *( )

Với x = thì: ( )* ⇔ =0 (vô lý)

Với x 2≠ ta có: ( )* y x x 22 x

x x x

− + +

⇔ = + = − − +

− − −

Để y nguyên x 2( − ) Vậy (x – 2) ước đó:(x 2− ∈ − −) { 3, 1,1, 3}⇒ ∈ −x { 1,1,3,5}

Vậy phương trình có nghiệm: (x, y) = (3; - 1) ; (5; -5); (1; -5); (-1; - 1)

Bài tốn Tìm số ngun dương x, y cho 6x 5y 18 2xy+ + = (1) Hướng dẫn giải

Ta có:

( )

5y 18 10y 36

x 2x

6 2y 2y

66 2y 66 33

2x 2x

6 2y 2y y

− − − −

= ⇔ =

− −

− + − − −

⇔ = = + ⇔ = +

− − −

Như x muốn nguyên dương (3 – y) phải ước – 33 Hay

(3 y− ) {∈ ± ± ±1; 3; 11; 33 ± } Lại y 1≥ ⇒ − ≤ ⇒ ∈ ± − −3 y y { 1; 3; 11; 33− } Ta có bảng sau:

3 - y -1 -3 -11 -33

y 14 36

x 19 - 14

Thử lại ta cặp thỏa mãn (19, 4); (8, 6); (4, 14); (3, 36)

Nhận xét: - Dễ xác định phương pháp để giải toán này, biểu diễn x theo y

được x 5y 18

6 2y

− −

=

Ta thấy biểu thức khó phân tích ví dụ trên, nhiên để ý ta thấy

(148)

CH

UY

ÊN

Đ

S

H

C

- Bài toán giải phương pháp đưa vềphương trình ước số Do ởbài toán nhân ở x để biến đổi, phải có bước thửlại xem x, y có thỏa mãn phương trình cho hay khơng.

Bài tốn Tìm nghiệm ngun phương trình: 2y x x y x2 + + + = 2+2y2 +xy

Hướng dẫn giải

Ta có: 2y x x y x2 + + + = 2+2y2 +xy⇔2y x x x y x 1 12( − −) ( − −) ( − + =) ( )

Nhận thấy x = khơng nghiệm phương trình (1)

Chia vế (1) cho (x – 1) ta được: 2y x y2 0 2( )

x

− − + = −

PT có nghiệm x, y nguyên, suy

x 1− nguyên nên { }

x x 1;

x

 =

− ∈ − ⇒ 

= 

Thay x = x = vào phương trình để ý đến y nguyên ta y = Vập phương trình cho có nghiệm (2; 1) (0; 1)

II. PHƯƠNG PHÁP SỬ DỤNG TÍNH CHẴN LẺ CỦA ẨN HOẶC XÉT SỐ DƯ

TỪNG VẾ

* Cơ sở phương pháp: Chúng ta dựa vào tính chẵn lẻ ẩn xét số dư hai vế phương trình nghiệm nguyên với số nguyên dùng lập luận để giải tốn

*Ví dụ minh họa:

Dạng 1: Sử dụng tính chẵn lẻ

Bài tốn Tìm x, y ngun tố thoả mãn y2−2x2 =1

Hướng dẫn giải

Ta có y2−2x2 = ⇒1 y2 =2x 12+ ⇒y số lẻ

Đặt y = 2k + (với k nguyên).Ta có (2k + 1)2 = 2x2 +

⇔ x2 = k2 + 2k ⇒ x chẵn , mà x nguyên tố ⇒ x = 2, y =

Vậy nghiệm phương trình (x, y) = (2, 3)

Bài tốn Tìm nghiệm nguyên dương phương trình

(2x 5y 2+ + )( x + +y x2+x)=105

(149)

CH

IN

H

P

H

C

K

T

H

I H

C S

IN

H

GI

I C

P H

AI

Ta có: (2x 5y 2+ + )( x + +y x2+x)=105

Ta thấy 105 lẻ ⇒ 2x + 5y + lẻ ⇒ 5y chẵn ⇒ y chẵn, ( )

2x + +y x + =x 2x + +y x x+1 lẻ có x(x + 1) chẵn, y chẵn ⇒ 2x lẻ ⇒ 2x = ⇒ x =

Thay x = vào phương trình ta

(5y + 1) ( y + 1) = 105 ⇔ 5y2 + 6y – 104 = 0⇒ y = y = 26

5

− ( loại) Thử lại ta có x = 0; y = nghiệm phương trình

Vậy nghiệm phương trình (x, y) = (0, 4)

Dạng 2: Xét tính chẵn lẻ xét số dư vế

Bài tốn Chứng minh phương trình sau khơng có nghiệm ngun:

2 2

a) x −y =1998 b) x +y =1999

Hướng dẫn giải

a) Do x số nguyên nên x 2k= x 2k k Z= + ( ∈ ) x2 =4k2∨x2 =4k2+4k 1+

x chia ln dư Tương tự ta có 2 y2 chia dư

Suy ra: x2−y2 chia cho dư hoặc Mà 1998 chia cho dư phương

trình cho khơng có nghiệm ngun

b) Như chứng minh câu a ta có: x ,y2 2 chia cho dư nên x2+y2 chia cho 4

luôn dư hoặc Mà 1999 chia cho dư phương trình cho khơng có nghiệm ngun

Chú ý: Chúng ta cần lưu ý kết quảở toán này:

2

*) x −y chia cho không dư 2

2

*) x +y chia cho không dư 3

Bài tốn Tìm nghiệm ngun phương trình: 9x y+ = +y

Hướng dẫn giải

Ta có: 9x y+ = + ⇔y 9x y y 1+ = ( + )

Ta thấy vế trái phương trình số chia cho dư nên y y 1( + ) chia cho dư Do y 3k 1= + y+ =1 3k+2(kZ)

(150)

CH

UY

ÊN

Đ

S

H

C

Thử lại: x k k , y 3k 1= ( + ) = + thỏa mãn phương trình cho Vậy nghiệm phương trình ( )x,y =(k k ,3k 1( + ) + ) với k Z∈

Bài toán Tìm x, y số tự nhiên thoả mãn x 32 + y =3026

Hướng dẫn giải

Xét y 0= ⇒x2+30 =3026⇒x2 =3025 Mà x ∈ N ⇒ x = 55

Xét y > ⇒ 3y chia hết cho 3, x2 chia cho dư 1

3y

x

⇒ + chia cho dư mà 3026 chia cho dư (loại)

Vậy phương trình có nghiệm (x,y) = (55,0)

Bài tốn Chứng minh phương trình x 7y 513− = khơng có nghiệm ngun

Hướng dẫn giải

Xét x 7k k Z= ( ∈ ) x 7.3

Xét x 7k k Z= ± ( ∈ ) x3 chia cho dư 6.

Xét x 7k k Z= ± ( ∈ ) x3 chia cho dư 6.

Xét x 7k k Z= ± ( ∈ ) x3 chia cho dư 6.

Do vế trái phương trình chia cho dư hoặc cịn vế phải phương trình chia dư Vậy phương trình cho vơ nghiệm

Bài tốn Tìm nghiệm ngun phương trình x2−5y2 =27

Hướng dẫn giải

Do x số nguyên nên ta biểu diễn x dạng: x 5k= x 5k 1= ±

x 5k 2= ± với k Z∈

- Xét x = 5k x 5y2− =27⇔( )5k −5y2 =27⇔5 5k y( 2− 2)=27

Điều vô lý vế trái chia hết cho với k y ngun cịn vế phải khơng chia hết cho

- Xét x 5k 1= ± x 5y2− =27⇔(5k 1± )2−5y2 =27

( )

2 2

25k 10k 5y 27 5k 2k y 23

⇔ ± + − = ⇔ ± − =

(151)

CH

IN

H

P

H

C

K

T

H

I H

C S

IN

H

GI

I C

P H

AI

- Xét x 5k 2= ± x 5y2− =27⇔(5k 2± )2−5y2 =27

( )

2 2

25k 10k 5y 27 5k 4k y 23

⇔ ± + − = ⇔ ± − =

Điều vô lý vế trái chia hết cho với k y ngun cịn vế phải khơng chia hết cho

Vậy phương trình cho vơ nghiệm

III. PHƯƠNG PHÁP DÙNG BẤT ĐẲNG THỨC

Dạng 1: Sử dụng bất đẳng thức cổ điển

*Cơ sở phương pháp:

Trong nhiều toán ta thường sử dụng bất đẳng thức để chứng minh vế không nhỏ hơn (hoặc không lớn hơn) vếcịn lại Muốn cho phương trình có nghiệm dấu bất đẳng thức phải xảy nghiệm phương trình.

Một số bất đẳng thức Cổđiển thường sử dụng như: 1 Bất đẳng thức Cauchy (tên quốc tếlà AM – GM)

Nếu a ,a ,a , ,a1 2 3 n là số thực khơng âm thì: n

1 n

a a a a a a a a

n

+ + + +

Đẳng thức xảy a1 =a2 =a3 = a= n

2 Bất đẳng thức Bunhiacopxki với hai bộsố thực (a ,a ,a , ,a1 2 3 n)(b ,b ,b , ,b1 2 3 n)ta

( 2 2)( 2 2) ( )2

1 n n 2 3 n n

a a a a+ + + + b b b b+ + + + ≥ a b a b a b a b+ + + +

Đẳng thức xảy tồn số thực k (k 0≠ ) sao cho ai =kbi với i = 1, 2, 3,…, n.

*Ví dụ minh họa:

Bài tốn Tìm số nguyên dương x, y thỏa mãn phương trình: (x x2+ )( 2+y2)=4x y2

Hướng dẫn giải

Áp dụng bất đẳng thức AM-GM ta có:

2

x 2x+ ≥ Dấu “=” xảy x =

2

x +y ≥2xy Dấu “=” xảy x = y

Do x, y dương nên nhân vế bất đẳng thức ta (x x2+ )( 2+y2)≥4x y2

Dấu xảy x = y =

Bài toán Giải phương trình nghiệm nguyên dương sau:

( )3 ( )

6 2 2

(152)

CH

UY

ÊN

Đ

S

H

C

Hướng dẫn giải

Ta có: ( )1 ⇔x z6+ 3+(y2+5)3 =15x z 3x y z2 + 2 ⇔x z6+ 3+(y2+5)3 =3x z y2 ( 2+5)

Áp dụng bất đẳng thức AM-GM ta có: x z6+ 3+(y2+5)3 ≥3x z y2 ( 2+5)

Dấu “=” xảy x2 =y2+ =5 z

Từ x2−y2 =(x y x y− )( + )=5 giải nghiệm (x, y, z) = (3, 2, 9).

Bài tốn Giải phương trình nghiệm nguyên sau (x y 1+ + )2 =3 x y 1( 2+ 2+ )

Hướng dẫn giải

Áp dụng bất đẳng thức Bunhiacopxki ta có: (1 1 x y 1+ + )( 2+ 2+ ≥) (x y 1+ + )2

Dấu “=” xảy 1 x y

x y 1= = ⇔ = =

Vậy nguyệm nguyên phương trình (x, y) = (1, 1)

Bài toán Tìm nghiệm nguyên phương trình: x2+xy y+ =x y 2

Hướng dẫn giải

Với x 2≥ y 2≥ ta có:

( ) −

 ≥

 ⇒ ≥ + = + + + + + > + +

 ≥

 ≥

2 2 AM GM

2 2 2 2 2 2

2 2

x y 4x x y 2 x y x y x y x y 2 xy x y xy.

x y 4y

Vậy x 2≤ y 2≤

Nếu x = -2 x = phương trình khơng có nghiệm nguyên

Thử x = -1, 1, ta thấy phương trình có nghiệm (0;0), (1; - 1), (-1; 1)  Dạng 2: Sắp xếp thứ tự ẩn

*Cơ sở phương pháp:

Khi phương trình đối xứng với ẩn x y z, , , , ta thường giả sử x≤ ≤ ≤y z để giới hạn miền nghiệm phương trình bắt đầu tìm từ nghiệm bé trở

*Ví dụ minh họa:

Bài tốn Tìm nghiệm nguyên dương phương trình: 2xyz x y z= + +

Hướng dẫn giải

(153)

CH

IN

H

P

H

C

K

T

H

I H

C S

IN

H

GI

I C

P H

AI

Chia vế cho z dương ta 2xy 3≤ ⇒xy xy 1≤ ⇒ =

Do x = y = Thay vào phương trình ban đầu ta được: 2z = z + hay z = Vậy nghiệm phương trình cho (x, y, z) = (1, 1, 2); (1, 2, 1); (2, 1, 1)

Bài toán Giải phương trình nghiệm nguyên dương: 1 1

x y z+ + =

Hướng dẫn giải

Do x, y, z có vai trị nên ta giả sử: x y z≤ ≤

Khi đó: 1= + + ≤ ⇒ ≤ ⇒ ∈1 1 x 3 x {1; 2; x Z} ( ∈ +)

x y z x

Với x = phương trình cho vơ nghiệm

Với x = ta có: 1 1 y

2 y z y

= + + ≤ + ⇒ ≤ Mặt khác y x 2≥ = ⇒ ∈y {2,3,4}

+) y = phương trình vơ nghiệm +) y = z =

+) y = z =

Với x = ta có: 1 1 y

3 y z y

= + + ≤ + ⇒ ≤ Mặt khác y x 3≥ = ⇒ = ⇒ =y z Vậy phương trình có nghiệm (x, y, z) = (2, 3, 6); (2, 4, 4); (3, 3, 3)

Bài tốn Giải phương trình nghiệm ngun dương: 1 z

x y+ =

Hướng dẫn giải

Biến đổi thành: xyz x y= +

Do đối xứng x y nên giả thiết x y≤ Ta cóxyz x y y y 2y= + ≤ + = ⇒xz 2.≤

Ta lựa chọn nghiệm trường hợp sau: x = 1, z = 1; x = 2, z = 1; x =1, z = Ta suy nghiệm (x, y, z) (1, 1, 2) (2, 2, 1)

Nhận xét: Ở toán vai trị x, y, z khơng bình đẳng nên ta khơng giải sử

x y z≤ ≤ ta chỉ giảsử x y≤

Bài tốn 4. Tìm nghiệm ngun dương phương trình:

(154)

CH

UY

ÊN

Đ

S

H

C

Hướng dẫn giải

Ta giả sử x ≥ y ≥ z ≥ t ≥

Ta có: ( x + y + z + t ) + 10 = xyzt

3

5 5 10 30

2 t 15 t t

yzt xzt xyt xyz xyzt t

⇔ = + + + + ≤ ⇒ ≤ ⇒ = ∨ =

Với t = ta có:

( )

{ }

2 x y z 10 2xyz

5 5 15 30

2 z 15 z 1; 2;

yz xz xy xyz z

+ + + + =

⇔ = + + + ≤ ⇒ ≤ ⇒ =

Nếu z =1 ta có x y 20 2xy( ) (2x 2y 5)( ) 65 x 35 x

y y

 =  =

+ + = ⇔ − − = ⇒ ∨

= =

 

Ta nghiệm (35, 3, 1, 1) ; (9, 5, 1, 1) hoán vị chúng

Với z = 2, z = phương trình khơng có nghiệm ngun

Với t = ta có:

( )

( ) ( )( )

2 x y z 20 4xyz

5 5 20 35 35

4 z z t 8x 8y 265

yz xz xy xyz z

+ + + + =

⇔ = + + + ≤ ⇒ ≤ ≤ ≥ ≥ ⇒ − − =

Do x ≥ y ≥ z ≥ nên 8x – ≥ 8y – ≥ 11 ⇒(8x – 5) (8y – 5) = 265 vô nghiệm

Vậy nghiệm phương trình (x, y, z)= ( 35; 3; 1; 1); (9; 5; 1; 1) hoán vị

Dạng 3: Chỉ nghiệm nguyên

*Cơ sở phương pháp: Chúng ta xét khoảng giá trị ẩn thể dưới

dạng: vài số nghiệm phương trình, chứng minh phương trình khơng cịn nghiệm khác

*Ví dụ minh họa:

Bài tốn Tìm nghiệm nguyên dương phương trình sau: 3 4x + x =5x

Hướng dẫn giải

Chia hai vế phương trình cho 5 ta được: x

x x

3 1

5

   

+ =

   

   

(155)

CH

IN

H

P

H

C

K

T

H

I H

C S

IN

H

GI

I C

P H

AI

Với x 3≥

x

3

5

   

⇒  ≤     

x

4

5

  ≤ 

   

   

x x 2

3 4 1

5 5

       

⇒  +  <  +  =

       

Vậy x = nghiệm phương trình

Bài tốn Tìm nghiệm ngun dương phương trình sau: 2x +3x =35

Hướng dẫn giải

Thử thấy x = 0; x = 1; x = không thỏa mãn 2x +3x =35

Với x = 23 +33 =35 (đúng)

Với x ≥ 23 +33 >35

Vậy x = nghiệm phương trình

Dạng 4: Sử dụng điều kiện ∆ ≥0 để phương trình bậc hai có nghiệm

*Cơ sở phương pháp:

Ta viết phương trình f(x, y) = dạng phương trình bậc hai ẩn, chẳng

hạn x y tham số Điều kiện để phương trình có nghiệm nguyên ∆ ≥0

*Ví dụ minh họa:

Bài tốn Tìm nghiệm ngun phương trình x2+y2−2x y 9.+ =

Hướng dẫn giải

Ta xem phương trình cho phương trình ẩn x tham số y, ta viết lại sau:

( )

2

x 2x y− + + −y =0

Để phương trình cho có nghiệm :

( )

( )

2

2

' y y y y 10

4y 4y 40 2y 41

∆ ≥ ⇔ − + − ≥ ⇔ + − ≤

⇔ + − ≤ ⇔ + ≤

Do đó: (2y 1+ ) {2∈ 1;9; 25} Ta có:

2y+1 -1 -3 -5

2y -2 -4 -6

y -1 -2 -3

x Loại Loại Loại Loại -1 -1

(156)

CH

UY

ÊN

Đ

S

H

C

Hướng dẫn giải

Ta xem phương trình cho phương trình ẩn x tham số y, ta viết lại sau: ( )

2

x −2 y x 2y+ + −3y 0=

Ta có: ∆ =' (y 1+ )2−(2y 3y2 − )=y 2y 2y 3y2 + + − 2+ = − +y 5y 12 +

Để phương trình có nghiệm ngun thì:

∆ ≥ ⇔ − + + ≥ ⇔ − ≤ − ≤

− + − +

⇔ ≤ ≤ ⇔ < <

2 29 29

' y 5y y

2 2

5 29 y 29 y

2 2

Vì y ngun nên y∈{0,1,2,3,4,5} thay vào phương trình ta tính giá trị x Giải ta nghiệm phương trình (x, y) = (0, 0); (0, 2)

Nhận xét: Ởví dụ cốtình tính ∆' cho bạn thấy tính ∆' có dạng tam thức bậc : f x( )=ay2 +by c+ với a < ta mới áp dụng phương pháp này, nếu a > 0

thì áp dụng phương pháp đưa vềphương trình ước số

IV. PHƯƠNG PHÁP DÙNG TÍNH CHẤT CỦA SỐ CHÍNH PHƯƠNG

Dạng 1: Dùng tính chất chia hết số phương

*Cơ sở phương pháp:

- Số phương khơng thể có chữ tận 2, 3, 7, 8;

- Số phương chia hết cho số nguyên tố p chia hết cho p2

- Số phương chia cho có số dư 1; - Số phương chia có số dư 1; - Số phương chia cho có số dư 0,

*Ví dụ minh họa:

Bài tốn 1. Tìm nghiệm nguyên phương trình: 9x y y 1+ = ( + ) Hướng dẫn giải

Ta có:

( ) ( ) ( )

+ = +

⇔ + = +

⇔ + = + +

⇔ + = +

2

2

9x y y

36x 20 4y 4y

36x 21 4y 4y

(157)

CH IN H P H Ụ C K Ỳ T H I H Ọ C S IN H GI Ỏ I C Ấ P H AI

Số phương chia hết chia hết cho 9, ta lại có 12x + khơng chia hết 3(12x + 7) không chia hết cho Do phương trình vơ nghiệm

Cách khác: ( ) ( ) ( ) + = + ⇔ + − − = ∆ = + + = + = +

9x y y

y y 9x

1 9x 36x 21 12x

Ta có ∆

chia hết cho không chia hết khơng số phương khơng tồn y ngun Vậy phương trình vơ nghiệm

Dạng 2: Biến đổi phương trình dạng 2

1 2 n n

a A +a A + +a A =k,

( 1, , ) i

A i= n đa thức hệ số nguyên,ai số nguyên dương, k số tự nhiên

*Cơ sở phương pháp:

Sử dụng đẳng thức đáng nhớ

(a b+ ) , đưa phương trình dạng Sau

dựa vào tính chất a Ai, i để phân tích thành 2

1 2 n n

k=a k +a k + +a k (với ki∈),

dẫn đến giải hệ phương trình

2 1 2 2 2 n n A k A k A k  =  =     = 

*Ví dụ minh họa:

Bài tốn Tìm nghiệm nguyên phương trình x2+y2− − =x y 8

Hướng dẫn giải

Ta có:

( ) ( ) ( ) ( )

2 2

2

2

2 2 2

x y x y 4x 4y 4x 4y 32

4x 4x 4y 4y 34 2x 2y 34

2x 2y

+ − − = ⇔ + − − =

⇔ − − + − + = ⇔ − + − =

⇔ − + − = +

Ta thấy 34 có dạng phân tích thành hai số phương 32 5 2

Do đó: ( ) ( ) ( ) ( ) 2 2 2 2

2x 2x 3

2y

2y

2x

2x

2y

2y

 − =  − =     − =  − =    ⇒   − =  − =     − =  − =  

(158)

CH

UY

ÊN

Đ

S

H

C

Bài tốn Giải phương trình nghiệm ngun x2 −4xy 5y+ =2(x y)− .

Hướng dẫn giải

Ta có x2−4xy 5y+ =2(x y)− ⇔x2 −4xy 5y+ 2−2x 2y 0+ =

2 2

2 2

x 2x(2y 1) (2y 1) (2y 1) 5y 2y

(x 2y 1) y 2y (x 2y 1) (y 1) 2(*)

⇔ − + + + − + + + =

⇔ − − + − − = ⇔ − − + − =

Xét phương trình (*) ta có: (x 2y 1− − )2 ≥ ∀0 x,y⇒(y 1− )2 ≤2 Mà x nguyên nên (y 1− ) { }2∈ 0,1

* Với (y 1− )2 =0 (x 2y 1− − )2 =2 (loại)

* Với (y 1)2 y 1 y

y 1 y

 − =  =

− = ⇒ ⇔

− = − =

 

- y = (x 1)2 x x

x x

 − =  =

⇒ − − = ⇒ ⇒

− = − =

 

- y = (x 1)2 x 1 x

x 1 x

 − =  =

⇒ − − = ⇒ ⇒

− = − =

 

Vậy phương trình cho có nghiệm: ( ) ( ) ( ) ( ) ( )x,y = 6,2 ; 4,2 ; 2,0 ; 0,0 Bài toán Giải phương trình nghiệm nguyên 5x2−2xy y+ =17.

Hướng dẫn giải

Ta có 5x 2xy y2− + =17⇔(x y− )2+4x2 =17 ⇔(x y)− =17 4x− 2 (*)

Xét phương trình (*) ta có (x y)2 0, x,y 17 4x2 0 x2 17

4

− ≥ ∀ ⇒ − ≥ ⇒ ≤

Mà x số nguyên nên x2∈{0;1; 4}

-Với x2 = ⇒0 (x y)− =17 (loại)

-Với x2 = ⇒1 (x y)− =13 (loại)

-Với x2 = ⇔ = ±4 x 2,

Với x 2 (2 y)2 1 y y

2 y y

 − =  =

= ⇒ − = ⇔ ⇔

− = − =

 

Với x 2 (2 y)2 1 y y

2 y y

 + =  = −

= − ⇒ + = ⇔ ⇔

+ = − = −

(159)

CH

IN

H

P

H

C

K

T

H

I H

C S

IN

H

GI

I C

P H

AI

Bài tốn Tìm nghiệm ngun phương trình x y xy x+ + = +y2

Hướng dẫn giải

Biến đổi: x y xy x+ + = +y2 ⇔(x 1− ) (2 + y 1− ) (2 + x y− )2 =2.

Tổng ba số phương nên tồn số Trường hợp: x – = ta (1; 0), (1; 2)

Trường hợp: y – = ta được: (0; 1), (2; 1) Trường hợp x – y = ta được: (0; 0), (2; 2)

Vậy phương trình có nghiệm (x, y) = (1; 0), (1; 2), (0; 1), (2; 1), (0;0), (2; 2)

Bài tốn Tìm nghiệm nguyên phương trình 2x2 +4x 19 3y = −

Hướng dẫn giải

( ) ( ) ( )

+ = −

⇔ + = −

2

2

2x 4x 19 3y

2 x y *

Ta thấy 3 y 2( − 2) ⇒ −7 y 22 ⇒y

lẻ Ta lại có 7 y− ≥0nên y2 =1

Khi (*) có dạng x 1( + )2 =18 Ta được: x 1+ = ±3do đóx1 =2; x2 = −4

Các cặp số (2; 1), (2; -1), (-4; 1), (-4; -1) thỏa mãn (2) nên nghiệm phương trình cho  Dạng 3: Xét số phương liên tiếp

*Cơ sở phương pháp:

Phương pháp dựa nhận xét sau:

1 Không tồn n Z∈ thỏa mãn: a2 <n2 <(a 1+ )2 với a Z∈

2 Nếu a2 <n2 <(a 2+ )2với a,n Zthì n = a + Tương tự với lũy thừa bậc 3

3 Nếu x x x n( + ) ( + ) (<y y y n+ ) ( + ) (< x a x a x a n+ )( + + ) ( + + )

Thì y y y n( + ) ( + ) (= x i x i x i n+ )( + + ) ( + + ) với i 1,2, ,a 1∈{ − }

*Ví dụ minh họa:

Bài tốn 1. Tìm nghiệm ngun phương trình: 1 x x+ + 2+x3 =y3 ( )1

(160)

CH

UY

ÊN

Đ

S

H

C

Ta có:

2

2 11 19

x x x 0; 5x 11x x

2 10 20

   

+ + = +  + > + + =  +  + >

   

Nên

(1 x x x+ + 2+ 3) (− x x 1 x x x2+ + < + +) 2+ 3< + +(1 x x x2 + 3) (+ 5x 11x 2+ + )

Do đó: x3 <y3 <(x 2+ )3⇒y3 =(x + )3

Kết hợp với (1) ta có: (x 1)3 1 x x x2 x x 0( ) x .

x

 =

+ = + + + ⇒ + = ⇒ 

= − 

Nghiệm phương trình là: (0;1) (-1;0)

Bài tốn Giải phương trình nghiệm nguyên: x3−y 2y3− −3y 0− = ( )2

Hướng dẫn giải

( )2 ⇔x3 =y3+2y2 +3y 3+ ( ) Ta có: y2 ≥0; 5y2 + >2 0nên

(y 2y3+ 2+3y 1+ −) (5y2+2)<y 2y3+ 2+3y y 2y+ ≤( 3+ 2+3y y + +)

Do đó: (y 1− )3 <x3≤(y 1+ )3 ⇒x3 =y3 x3 =(y + )3

Nếu x3 =y3 kết hợp với (3) ta có: 2y2 +3y 0+ = ⇒ = − ⇒ = −y 1 x 1.

Nếu x3 =(y + )3 Phối hợp với (3) ta có y2 = ⇒ =0 y 0 , lúc x =

Vậy nghiệm phương trình cho (-1; -1) (1; 0)

Bài toán Giải phương trình nghiệm nguyên: x4 +2x3 +2x2 + + =x y2

Hướng dẫn giải

Ta có (x2 +x)2 <x4 +2x3 +2x2 + + <x 3 (x2 + +x 2)2

( )2 ( )2

2

x x y x x

⇔ + < < + +

mà (x2 +x )2 (x2 + +x ()2 hai số phương

( )2

2

1

y x x

(161)

CH

IN

H

P

H

C

K

T

H

I H

C S

IN

H

GI

I C

P H

AI

( )2

4 2

2

x x x x x x

⇔ + + + + = + +

2

2

x

x x

x

= 

⇔ + − = ⇔ 

= − 

Thay x 1= ta y= ±3 Thay x= −2 ta y= ±3

Vậy nghiệm phương trình (x ; y) ∈{(1; ; 1;3 ;− ) ( ) (−2;3 ;) (− −2; 3)}

Bài tốn Giải phương trình nghiệm ngun: x (x 1)2+ + =y (y 1)4+ +

Hướng dẫn giải

Biến đổi phương trình dạng

+ + = + + + + = + + = ∈

2 2 2

x x y (y 1) 2y(y 1) (y y 1) k ,k Z (1)

- Nếu x 0> ⇒x2 <x2+ + <x (x 1)+ ⇒x2 <k2 <(x 1)+ 2 khơng có số ngun k thỏa mãn

- Nếu x y2 y 1 1

x

 =

⇒ + + = ±

 = 

Ta có nghiệm ngun phương trình (0; 0), (0; -1), (-1; 0); (-1; -1)

- Nếu x< − ⇒1 (x 1)+ <x2+ + <x x2 ⇒(x 1)+ <k2 <x2 khơng có số ngun k thỏa mãn

Bài tốn Giải phương trình nghiệm nguyên

( )

4 2

x +x −y + +y 10 6= Hướng dẫn giải

( )6 ⇔y y 1( − =) x4+x 10 72 + ( )

Ta có: x4+x2 <x4+x 102 + <(x4+x 102 + ) (+ 6x2 +2 )

Do đó: ( ) ( ) ( )( ) ( ) ( )( )

( ) ( )( )

2

2 2

2

y x x x

x x y y x x

y y x x

 − = + +

+ < − < + + ⇒

 − = + +

Kết hợp với (7) ta suy ra: x22

x

 =  = 

Từ đó: x= ±2, x= ±1

Do ta tìm nghiệm phương trình (6)  Dạng 4: Sử dụng điều kiện là số phương

(162)

CH

UY

ÊN

Đ

S

H

C

Với phương trình nghiệm ngun có dạngf x,y( )=0 viết dạng phương trình

bậc ẩn chẳng hạn ẩn x, ngồi điều kiện ∆ ≥0 để phương trình có nghiệm ngun phải sốchính phương Vận dụng điều ta giải tốn

Chú ý: là số chính phương chỉ là điều kiện cần chưa đủ để phương trình có

nghiệm ngun, đósau tìm giá trị cần thửlại vào phương trình ban đầu

*Ví dụ minh họa:

Bài tốn Giải phương trình nghiệm nguyên 3x2+y2+4xy 4x 2y 0+ + + =

Hướng dẫn giải

Ta có: 3x2 +y2+4xy 4x 2y 0+ + + =

( ) ( )

2

y 2x y 3x 4x

⇔ + + + + + =

Coi phương trình (1) phương trình ẩn y tham số x ta có:

( )2 ( 2 ) 2 2 2

' 2x 3x 4x 4x 4x 3x 4x x

∆ = + − + + = + + − − − = −

Để phương trình có nguyện ngun ∆' phải số phương hay ∆ =' x n2− =

với n N∈

(x n x n− )( + )=4 giải ta x = x = -2 Với x = y =

Với x = -2 y = -5

Vậy phương trình có nghiệm (x, y) = (2, 3) ; (-2, -5)

Bài toán Giải phương trình nghiệm nguyên x y2 2−xy x= 2+2y 12 ( )

Hướng dẫn giải

Phương trình cho viết lại: (x y xy x2− ) 2− − =0 2( )

Do x nguyên nên (x 22− )≠0 coi phương trình (2) phương trình ẩn y tham số x ta có:

( ) ( )

2 2 2

x 4x x x 4x

∆ = + − = −

Để phương trình có nguyện ngun ∆ phải số phương

-Xét x = từ (1) suy y =

-Xét x 0≠ (4x 72− )phải số phương 4x m2 − = 2với m số nguyên, ta

(163)

CH

IN

H

P

H

C

K

T

H

I H

C S

IN

H

GI

I C

P H

AI

Với x = thay vào (2) ta được: y2+ − = ⇒ ∈y 0 y {1; − }

Với x = -2 thay vào (2) ta được: y2− − = ⇒ ∈ −y 0 y { 1; }

Nghiệm nguyên phương trình (x, y) = (2, 1); (2, -2); (-2, -1); (-2, 2)

Dạng 5: Sử dụng tính chất: Nếu hai số nguyên liên tiếp có tích số phương hai số ngun liên tiếp

*Cơ sở phương pháp:

Giảsửa(a + 1) = k2(1) với a Z,k N ∈ ∈

Giải sửa ≠ 0, a + ≠ k2≠ Do k số tựnhiên nên k > 0.

Từ(1) suy ra: a2+ a = k2

( ) ( )

⇒4a2 +4a 4k= ⇒4a2 +4a 4k+ = + ⇒1 2a 1+ =4k2 +1 2

Do k > nên 4k2 <4k2 + <1 4k2 +4k 1+ ( )3

Từ (2) (3) suy ( ) (2k < 2a 1+ ) (2 < 2k , vô lý + )2

Vậy a(a + 1) = k2 tồn hai số a, a +

*Ví dụ minh họa:

Bài tốn 1. Tìm nghiệm ngun phương trình: x2 +xy y+ =x y2

Hướng dẫn giải

Thêm xy vào hai vế: x2 +2xy y+ =x y2 +xy ⇔(x y+ )2 =xy xy 1( + ) ( )*

Ta thấy xy xy + hai số ngun liên tiếp, có tích số phương nên tồn số

Xét xy = Từ (1) có x2 + y2 = nên x = y =

Xét xy + = Ta có xy = -1 nên (x, y) = (1; -1), (-1; 1)

Thử lại ba cặp số (0; 0), (1; -1), (-1; 1) nghiệm phương trình cho

Bài tốn 2. Tìm nghiệm ngun phương trình: x2 +2xy 5y 6= + ( )1

Hướng dẫn giải

Ta có ( )1 ⇔x 2xy y2+ + =y2+5y 6+ ⇔(x 1+ ) (2 = y y 2+ )( + )

Do (y + 3) (y + 2) số nguyên liên tiếp mà có tích số phương nên số phải

(164)

CH

UY

ÊN

Đ

S

H

C

Vậy phương trình có nghiệm nguyên (x, y) = (-3, -1); (-2, -1)

Dạng 6: Sử dụng tính chất: Nếu hai số ngun dương ngun tố có tích một số phương số số phương

*Cơ sở phương pháp:

Giảsử ab = c2(1) với a,b,c N , a,b∈ * ( )=1

Giảsửtrong a b có số, chẳng hạn a, chứa thừa số nguyên tố p với sốmũ lẻthì số b

khơng chứa thừa số p nên c2 chứa thừa số p với sốmũ lẻ, trái với giả thiết c2là sốchính phương.

*Ví dụ minh họa:

Bài tốn Tìm nghiệm ngun dương của phương trình: xy z= ( )1

Hướng dẫn giải

Trước hết ta giả sử (x, y, z) = Thật ba số (x ,y ,z0 0 0) thỏa mãn (1) có ƯCLN d, giả sử x0 =dx ,y1 0 =dy ,z1 0 =dz 1 (x ,y ,z1 1 1) nghiệm

phương trình (1)

Với (x, y, z) = x, y, z đơi nguyên tố nhau, hai ba số x, y, z có ước chung d số lại chia hết cho d

Ta có z2 = xy mà (x, y) = nên x = a2, y = b2 với a,b N∈ * Suy z2 = xy = (ab)2 , z = ab

Như vậy:

 =  =   = 

2

x ta y tb z tab

với t số nguyên dương tùy ý

Đảo lại, hiển nhiên số x, y, z có dạng thỏa mãn (1) Công thức cho ta công thức nghiệm ngun dương (1) Bài tốn Tìm tất cặp số nguyên (x; y) thỏa mãn

4 2

x −2x +6x −4y −32x 4y 39 0+ + = Hướng dẫn giải

Ta có:

− + − − + + =

<=> − + − + = − +

<=> − + + = −

4 2

4 2

2 2

x 2x 6x 4y 32x 4y 39

x 2x 6x 32x 40 4y 4y

(165)

CH

IN

H

P

H

C

K

T

H

I H

C S

IN

H

GI

I C

P H

AI

Vì y số nguyên nên 2y – ≠ ⇒ x ≠

Vì (2y – 1)2 (x – 2)2 số phương khác nên x2 + 2x + 10 số phương

Đặt 2 ( *)

2 10

x + x+ =m mN suy (x 1) m+ + = ⇔ + −(x m)(x m)+ + = −9 *( ) Do (x + + m) > (m + – m) nên

( )

x m x

x m m

x m x

*

x m m

x m x

x m m

 + + =  =

 + − = −  =

 

 

 + + =  = −

 

⇔  ⇔ 

+ − = − =

 

 

 + + =  = −

 

 + − = −  =

 

•x = ⇒ (2y – 1)2 = 25 ⇒ y = y = –2

•x = –5 ⇒ (2y – 1)2 = 1225 ⇒ y = 18 y = –17

•x = –1 ⇒ (2y – 1)2 = 81 ⇒ y = y = –4

Vậy (x;y) nguyên thỏa yêu cầu toán (3;3),(3;–2),(–5;18),(–5;–17),(–1;5),(–1;–4)

V. PHƯƠNG PHÁP LÙI VÔ HẠN, NGUYÊN TẮC CỰC HẠN

Dạng 1: Phương pháp lùi vô hạn

*Cơ sở phương pháp:

Dùng để chứng minh phương trình f(x, y, z, ) ngồi nghiệm tầm thường

x = y = z = khơng cịn nghiệm khác Phương pháp diễn giải sau:

Giải sử (x ,y ,z , 0 0 0 ) là nghiệm phương trình f(x, y, z, ), nhờ phép biến đổi suy luận ta tìm nghiệm khác (x ,y ,z , 1 1 1 )sao cho nghiệm có quan hệ với nghiệm ban đầu tỷsốk Ví dụ x0 =kx ,y1 0 =ky ,z1 0 =kz1 ;

Rồi từ bộ (x ,y ,z , 2 2 2 )có quan hệ với (x ,y ,z , 1 1 1 )bởi tỷsốk đó.

Ví dụx1=kx ,y2 1 =ky ,z2 1=kz2 Q trình dẫn đến x ,y ,z , 0 0 0 chia hết cho ks vớs số tự

nhiên tùy ý, điều xảy x = y =z = Chúng ta đến ví dụ cụ thểnhư sau:

*Ví dụ minh họa:

Bài tốn Giải phương trình nghiệm nguyên sau x2 +y2 =3z2

Hướng dẫn giải

Gọi (x ,y ,z0 0 0)là nghiệm phương trình Xét (mod 3) ta chứng minh x ,y0 0

chia hết cho Thật rõ ràng vế phải chia hết cho suy ( 2)

0

x + y  Ta có

( ) ( )

2

0

x ≡0;1 mod ; y ≡0;1 mod ( 2) 2

0 0

(166)

CH

UY

ÊN

Đ

S

H

C

Đặt x0 =3 ;x y1 =3y1 vào rút gọn ta ( )

2 2

1 0

3 x +y =z ⇒z 3 ⇒z0 =3z1

Thế z0 =3z1vào ( 2)

1

3 x +y =z rút gọn ta được: 2

1 1

x +y =z Do

(x ,y ,z0 0)là nghiệm phương trình (x ,y ,z1 1)cũng nghiệm phương trình

trên Tiếp tục suy luận dẫn đến k

0 0

x ,y ,z 3 điều xảy x0 =y0 =z0 =0

Vậy phương trình có nghiệm (x, y, z) = (0, 0, 0)

Bài tốn Tìm nghiệm ngun phương trình: 3

3

xyz = (1)

Hướng dẫn giải

Giả sử (x y z0, 0, 0) nghiệm ngun phương trình x03đặt x0 =3 x1 thay

x = x vào (1) ta được: 9x13−y03−9z03= ⇒0 y03. đặt y0 =3y1⇒z03,khi đó:

3 3 3

1 1 0

9x −27y −3z = ⇒0 3x −9yz = ⇒0 z 3.đặt z0 =3z1 đó: x13−3y13−9z13 =0

Vậy 0, 0,

3 3

x y z

 

 

  nghiệm phương trình

Quá trình tiếp tục được: , 0,

3k 3k 3k

x y z

 

 

 là nghiệm nguyên (1) với k điều

này xảy x0 = y0 =z0 =0.Vậy ( 0, 0, ) nghiệm phương trình cho

Bài tốn Giải phương trình nghiệm nguyên sau: x2+y2+z2 =2xyz

Hướng dẫn giải

Gọi (x ,y ,z0 0 0)là nghiệm phương trình trên, ta có 2

0 0 0

x +y +z =2x y z suy

( 2 2)

0 0

x +y +z chẵn (do 2x y z0 0) nên có trường hợp xảy ra:

Trường hợp 1: Có số lẻ số chẵn khơng tính tổng qt giả sử x ,y0 0 lẻ, z0 chẵn

Xét mod ta có: 2 ( )

0 0

x +y +z ≡2 mod 2x y z 40 0 (do z0 chẵn) ⇒ Vô lý

Trường hợp 2: Cả số đề chẵn Đặt x0 =2x ,y1 0 =2y ,z1 0 =2z1 vào rút gọn ta có:

2 2

1 1 1

x +y +z =4x y z lập luận ta x ,y ,z1 1 1chẵn

Quá trình tiếp tục đến k( *)

0 0

x ,y ,z k N ∈ điều xảy x0 =y0 =z0 =0

(167)

CH

IN

H

P

H

C

K

T

H

I H

C S

IN

H

GI

I C

P H

AI

Dạng 1: Nguyên tắc cực hạn * Cơ sở phương pháp:

Về hình thức phương pháp khác với phương pháp lùi vô hạn ý tưởng sử dụng nhau, chứng minh phương trình ngồi nghiệm tầm thường khơng cịn nghiệm khác

Phương pháp bắt đầu việc giả sử (x ,y ,z , 0 0 0 ) nghiệm phương trình f(x, y, z, .) với điều kiện buộc với (x ,y ,z , 0 0 0 ) Ví dụ x0 nhỏ

0 0

x +y +z + nhỏ Bằng phép biến đổi số học ta tìm nghiệm khác(x ,y ,z , 1 1 1 )trái với điều kiện buộc Ví dụ tìm (x ,y ,z , 0 0 0 )với

0

x nhỏ ta lại tìm (x ,y ,z , 1 1 1 )thỏa mãn x1<x0 từ dẫn tới phương trình cho có nghiệm x0 =y0 =z0 =0

* Ví dụ minh họa:

Bài tốn Giải phương trình nghiệm nguyên sau 8x4+4y4+2z4 =t4 ( )1

Hướng dẫn giải

Giải sử (x ,y ,z0 0 0)là nghiệm phương trình với điều kiện x0nhỏ Từ phương trình (1) suy t số chẵn Đặt t 2t= 1 vào phương trình (1) rút gọn ta

được: 4 4

0 0

4x +2y +z =8t rõ ràng z0 chẵn Đặt 4 4

0 0 1

z =2z ⇒2x +y +8z =4t ⇒y chẵn

Đặt 4 4

0 1 1

y =2y ⇒x +8y +4z =2t ⇒x chẵn

Đặt 4 4 ( )

0 1 1 1 1

x =2x ⇒8x +4y +2z =t ⇒ x ; y ; z ; t nghiệm phương trình dễ thấy x1 <x0(vô lý) ta chọn x0nhỏ Do phương trình có nghiệm (x,y,z,t) (= 0,0,0,0 )

Tổng kết: Một tốn nghiệm ngun thường giải nhiều phương pháp, bạn

đọc nên tìm nhiều cách giải cho tốn đểrèn luyện kĩ Sau giải một tốn nhiều phương pháp để tổng kết

Bài toán Tìm nghiệm nguyên phương trình sau: x2+xy y+ =x y2 ( )1

Lời giải

(168)

CH

UY

ÊN

Đ

S

H

C

( ) ( )

( ) ( )

( )( )

2 2

2 2

2 2

2

2

x xy y x y

4x 4xy 4y 4x y

4x 8xy y 4x y 4xy

2x 2y 2xy 1

2xy 2x 2y

2xy 2x 2y 2xy 2x 2y

+ + =

⇔ + + =

⇔ + + = +

⇔ + = + −

⇔ + − + =

⇔ + + + + − − =

Sau giải phương trình ước số

Cách Dùng tính chất sốchính phương phương trình ước số

( )

( ) ( )

2 2

2 2 2 2

2 2 2

4x 4xy 4y 4x y

2x y 3y 4x y

2x y y 4x

+ + =

⇔ + + =

⇔ + = −

Nếu y = x = ta có (0, 0) nghiệm phương trình Nếu y 0≠ 4x 32− phải số phương

Ta có: 4x2− =3 k k N2( ∈ ) đưa (2x k 2x k+ )( − )=3

Ta tìm x = x = -1 từ tìm y Cách Đưa vềphương trình bậc x

(y x yx y2− ) 2− − =0 ( )2

Xét y = (2) có dạng: -x – = x = -1 Xét y = -1 (2) có dạng x – = x =

Xét y≠ ±1 (2) phương trình bậc hai x có:

( ) ( )

2 2 2

y 4y y y 4y

∆ = + − = −

Ta phải có ∆ số phương Nếu y = từ (2) suy x =

Nếu y 0≠ 4y2−3 phải số phương

Ta có 4y2− =3 k k N2( ∈ ) (⇒ 2y k 2y k+ )( − )=3,ta y= ±1 xét y= ±1

Cách Sử dụng bất đẳngthức

Không tính tổng quát giả sử x ≤ y , x2 ≤y ,xy xy y2 ≤ ≤ 2

Do đó: x y2 =x2+xy y+ ≤y2 +y2+y2 ≤3y2

Nếu y = x =

(169)

CH

IN

H

P

H

C

K

T

H

I H

C S

IN

H

GI

I C

P H

AI

Cách Sử dụng tính chất số phương

Thêm xy vào hai vế x 2xy y2+ + =x y2 2+xy⇔(x y+ )2 =xy xy 1( + )

Ta thấy xy (xy + 1) hai số ngun liên tiếp có tích số phương nên tồn số

Xét xy = từ (1) có x2 +y2 = ⇒ = =0 x y 0

Xét xy = -1 nên x = , y = -1 x = -1, y =

Thử lại thấy phương trình có ba nghiệm (0, 0); (1, -1); (-1, 1) C BÀI TẬP ÁP DỤNG

Bài 1: Tìm nghiệm nguyên phương trình 2xy− − =x y

Bài 2: Tìm nghiệm nguyên phương trình 2

2009

+ + =

x x y

Bài 3: Tìm nghiệm nguyên phương trình 2

5 10

x + y + z + xyxz=

Bài 4: Giải phương trình nghiệm nguyên

3x −2xy+ −y 5x+ =2

Bài 5: Tìm nghiệm nguyên phương trình 2

(x +y x)( +y )=(xy)

Bài 6: Giải phương trình nghiệm nguyên 3

2

xy = xy+

Bài 7: Tìm nghiệm nguyên dương phương trình 5(x+ + + =y z) 2xyz

Bài 8: Tìm nghiệm nguyên phương trình

a) 4

1+ +x x +x +x = y ;

b) 3

1+ +x x +x = y

Bài 9: Giải phương trình nghiệm nguyên 4x+9y=48

Bài 10: Tìm số tự nhiên lẻ n để 26n+17 số phương Bài 11: Tìm số nguyên x y z, , cho x4+y4 +z4 =2012

Bài 12: Tìm nghiệm nguyên dương hệ phương trình 213 22 22

13

x y z

x y t

 + =

+ =

Bài 13: Tìm nghiệm nguyên phương trình 3

3

xyz =

Bài 14: Tìm nghiệm nguyên phương trình

2 2

2 2 4

x + y + zxyyzz= −

Bài 15: Tìm nghiệm nguyên phương trình

( )( )( )

1 48

x + y + z + = xyz Bài 16: Tìm nghiệm nguyên hệ phương trình

2

2

9 16 12

x z y t xt yz

 + =  + = 

 + = 

Bài 17: Tìm nghiệm phương trình: x3+y x y xy3− − =5

Bài 18: Tìm nghiệm nguyên phương trình : x(x + 1)(x + 2)(x + 3) = y2 (1)

(170)

CH

UY

ÊN

Đ

S

H

C

Bài 20: Tìm tất số x, y nguyên dương thỏa mãn phương trình: 1 1

x y 617+ =

Bài 21: Giải phương trình nghiệm nguyên dương 1

x y p+ = p số nguyên tố

Bài 22: Tìm nghiệm nguyên dương phương trình: 1 1

x y 6xy 6+ + =

Bài 23: Tìm nghiệm nguyên của phương trình 6x 15 10z 3+ + =

Bài 24: Chứng minh phương trình sau khơng có nghiệm nguyên: ( )

+ + =

2 2

x y z 1999

Bài 25: Tìm nghiệm dương phương trình x+ y = 50

Bài 26: Giải phương trình nghiệm nguyên: y= x x 1+ − + x x 1− −

Bài 27: Giải phương trình tập số nguyênx2015 = y(y 1)(y 2)(y 3) 1+ + + +

(Chuyên Quảng Trung – Bình Phước 2015)

Bài 28: Tìm số tự nhiên x số nguyên y cho 2x+ =3 y2

Bài 29: Tìm số tự nhiên x, y thỏa mãn: (2 2x + )( x+2 2)( x +3 2)( x +4 5)− y =11879.

Bài 30: Tìm tất cặp (x, y, z) số nguyên thỏa mãn hệ phương trình: 3

x y z

x y z

 + + =

 + + = 

Bài 31: Tìm số nguyên x, y, z thỏa mãn đẳng thức: ( )

( )

2

x y z

2x xy x 2z

 − + =

 

− + − =



Bài 32: Tìm số thực a để nghiệm phương trình sau số nguyên: ( ) ( )

2

x ax a 2− + + =0

Bài 33: Tìm số nguyên dương x y thoả mãn phương trình:

( 2 2 )2 ( 4 4) 2

x 4y+ +28 −17 x +y =238y 833.+

(Chuyên Nguyễn Trãi – Hải Dương 2016 – 2017)

Bài 34: Tìm tất cặp số tự nhiên x, y thỏa mãn: 2 xx =9y2 +6y 16+

(Chuyên Hà Nội 2016 – 2017)

Bài 35: Tìm nghiệm nguyên phương trình: x y x y x y xy2 2( + )+ + = +

(Trích đềvào lớp 10 chuyên ĐHKHTN, ĐHQGHN năm 2014)

(171)

CH

IN

H

P

H

C

K

T

H

I H

C S

IN

H

GI

I C

P H

AI

Bài 37: Tìm nghiệm nguyên phương trình x2−y2 =xy 8+

(Trích đềvào Chuyên Bình Dương 2017)

Bài 38: Tìm nghiệm nguyên phương trình x 4y 3+ =

(Trích đề vào Chuyên Lê Hồng Phong – Nam Định) Bài 39: Tìm nghiệm nguyên phương trình sau x2+y2+5x y2 2+60 37xy=

(Trích đề vào Chuyên Bạc Liêu 2017)

Bài 40:Giải phương trình nghiệm nguyên y 2x x x 3− − = ( + )2 ( )1

(Trích đềvào Chuyên Hưng Yên 2017)

Bài 41: Giải phương trình nghiệm nguyên x2+2y2−2xy 4x 8y 1− + + = ( )

(Chuyên Lương Thế Vinh – Đồng Nai 2017)

Bài 42: Tìm x, y nguyên cho x+ y = 18

(Chuyên Bình Định 2015)

Bài 43: Tìm số nguyên x y thoả mãn phương trình9x y+ = 2+y

(Chuyên Phan Bội Châu – NghệAn 2014)

Bài 44: Tìm cặp số nguyên (x;y) thỏa mãn phương trình:2015(x2+y ) 2014(2xy 1) 252 − + =

(Chuyên TP HồChí Minh 2014)

Bài 45: Tìm nghiệm phương trình: x3+y x y xy3− − =5

(Chuyên Lam Sơn 2014)

Bài 46: 1) Tìm tất số nguyên tố p số nguyên dương x,y thỏa mãn

2

p 2x(x 2) p 2y(y 2)

 − = +

 − = +



2) Tìm tất số nguyên dương n cho tồn số nguyên dương x, y, z thoả mãn

3 3 2

x +y +z =nx y z

(Chuyên Hà Nội Amsterdam 2014)

Bài 47: Tìm nghiệm nguyên dương hệ phương trình: x y z3 3 2

x y z

 + =  

+ =



(Chun Hồng Văn Thụ - Hịa Bình 2015)

Bài 48: Tìm nghiệm nguyên phương trình:x2−2y(x y) 2(x 1)− = +

(Chuyên Hùng Vương Phú Thọ2015) Bài 49: Tìm số nguyênx,y thỏa mãn x4+x2−y2 − +y 20 0.=

(172)

CH

UY

ÊN

Đ

S

H

C

Bài 50: a) Chứng minh không tồn số nguyên (x, y, z) thỏa mãn x4+y4 =7z4+5

b) Tìm tất nguyện nguyên thỏa mãn đẳng thức (x 1+ ) (4− x 1− )4 =y3 (Chuyên KHTN Hà Nội 2011)

Bài 51: Tìm tất cặp số nguyên ( ; )x y thỏa mãn 2x2+5y2 =41 2+ xy

(Chuyên Nam Định 2018-2019) Bài 52: Tính tất cặp số nguyên dương (x y; ) thỏa mãn: x2019 = y2019−y1346−y673+2

(Chuyên Lam Sơn – Thanh Hóa 2018-2019)

Bài 53: Cho phương trình 3

2 9!(1)

x + y + z = với x y z; ; ẩn 9! Là tích số nguyên dương liên tiếp từ đến

a) Chứng minh có số nguyên x y z; ; thỏa mãn (1) x y z, , chia hết cho b) Chứng minh không tồn số nguyên x y z, , thỏa mãn (1)

(Chuyên Vĩnh Phúc 2018-2019)

Bài 54: Tìm nghiệm nguyên phương trình

2

xxy+ = +x y

(Chuyên Bến Tre2018-2019) Bài 55: Tìm số nguyên x y z, , thỏa mãn đồng thời: x2+4y2+ +z2 2xz+4(x+ =z) 396

2

3

x +y = z

(Chuyên Đăk Lăk2018-2019) Bài 56: Tìm cặp số nguyên (x y; )thỏa mãn điều kiện 2x2−4y2−2xy−3x− =3

(Chuyên Đồng Nai 2018-2019)

Bài 57:Tìm nghiệm nguyên phương trình:

3x −2xy+ −y 5x+ =2

(Chuyên Tuyên Quang 2018-2019) Bài 58: Tìm x y, nguyên dương thỏa mãn: 16(x3−y3)=15xy+371

(Chuyên Thái Nguyên 2018-2019) Bài 59: Tìm cặp số nguyên x y, thỏa mãn x2−2y2 =1

(Chuyên Bắc Ninh 2018-2019) Bài 60: Tìm nghiệm nguyên phương trình x2−xy y+ =2x−3y−2

(Chuyên Vĩnh Long2018-2019) Bài 61: Tìm tất cặp số nguyên ( ; )x y thỏa mãn đẳng thức x y2 2−x2−6y2 =2 xy

(Chuyên Quảng Nam 2018-2019) Bài 62: Tìm tất cặp số nguyên x y, thỏa mãn y2+2xy−3x− =2

(Chuyên Lào Cai 2018-2019) Bài 63: Tìm tất số nguyên ( )a b; thỏa mãn ( 2) ( )

3 a +b −7 a+b = −4

(173)

CH

IN

H

P

H

C

K

T

H

I H

C S

IN

H

GI

I C

P H

AI

(Chuyên Toán Lam Sơn – Thanh Hóa 2019-2020)

Bài 65: Tìm tất nghiệm nguyên phương trình:

5

xxyx+ y+ =

(Chuyên Tin Lam Sơn – Thanh Hóa 2019-2020) Bài 66: Tìm tất nghiệm nguyên dương phương trình

2

xy −(y 45)− +2xy x 220y 2024 0+ − + =

(Chuyên Hưng Yên2019-2020) Bài 67: Tìm tất số tự nhiên n để phương trình x2 −n x n 02 + + = (ẩn số x) có

nghiệm số nguyên

(Chuyên Bình Thuận 2019-2020)

Bài 68: Tìm tất cặp số nguyên (x, y) thỏa mãn x y2 85

x y 13

+ = +

(Chuyên Phú Yên2019-2020)

Bài 69: Tìm số nguyên không âm a, b,n thỏa mãn:

2

3 2

2

n a b

n a b

 = + 

+ = +



(Chuyên Quảng Nam 2019-2020) Bài 70: Tìm tất cặp số nguyên (x; y) thỏa mãn 2020(x2 +y ) 2019(2xy 1) 52 − + =

(Chuyên Cần Thơ2019-2020)

Bài 71: Tìm tất nghiệm nguyên dương phương trình 2

2x y− =1 x +3y

(Chuyên Đăk Nông 2019-2020)

Bài 72: Tìm nghiệm nguyên dương phương trình x+ + + =y x+ y

(Chuyên Quảng Ngãi 2019-2020) Bài 73: Giải phương trình nghiệm nguyên 4y2 = +2 199 x− −2x

(Chuyên Bình Phước 2019-2020) Bài 74: Tìm tất cặp số nguyên dương  x y; thỏa mãn: (xy x y x+ + )( +y2 +1)=30

(Chuyên Bắc Ninh 2019-2020) Bài 75: Tìm tất cặp số nguyên ( )x y; thỏa mãn

( )( )

2x+5y+1 2x− + +y x +x =65

(Chuyên Tiền Giang 2019-2020) Bài 76: Tìm tất cặp số nguyên dương (m;n) thỏa mãn phương trình

2m.m2 = 9n2 -12n +19

(Chuyên Bà Rịa Vũng Tàu 2019-2020)

Bài 77: Tìm tất cặp số nguyên thỏa mãn 2

(x − +x 1)(y +xy)=3x−1

(174)

CH

UY

ÊN

Đ

S

H

C

(Chuyên Sư phạm Hà Nội 2019-2020)

Bài 79: Tìm x, y thỏa mãn: x y 2( + − )= x.y

(HSG Lớp An Giang năm 2015-2016) Bài 80: Tìm tất nghiệm nguyên phương trình x2 +xy y+ =x y2

(HSG Lớp Thanh Hóa năm 2015-2016) Bài 81: Tìm tất cặp số nguyên (x;y) thoả mãn x5+y2 =xy 12+

(HSG Lớp TP Bắc Giang năm 2016-2017) Bài 82: Tìm số nguyên dương x, y, z thỏa mãn: 3x 18y2 − +2z2+3y z 18x 272 2− =

(HSG Lớp Hải Dương năm 2014-2015) Bài 83: Tìm nghiệm nguyên phương trình x y x y x y x 2 2( + )+ = + ( − )

(HSG Lớp Thanh Hóa 2018-2019) Bài 84: Tìm tất nghiệm nguyên dương phương trình:

2

xy +2xy 243y x 0− + =

Bài 84: Tìm số nguyên x, y thỏa mãn đẳng thức x2 =y2+ y 1+

Bài 85: Giải phương trình nghiệm nguyên y2 = +1 9 x− 2−4x

Bài 86: Tìm số nguyên a để phương trình sau có nghiệm ngun dương 4 3a a− = −

Bài 87: Tìm tất cặp ( )x; y nguyên thỏa mãn x y2 2+(x 2− ) (2+ 2y 2− )2−2xy x 2y 4( + − )=5

(HSG Lớp Lạng Sơn năm 2018-2019) Bài 88: Tìm nghiệm nguyên dương phương trình : 4y4+6y x2− =

(HSG Lớp Bình Phước năm 2018-2019) Bài 89: Tìm tất cặp số nguyên (x; y) thỏa mãn phương trình

(x y x y 6xy y x y− − )( + − )+ + 2( − − ) (=2 x y 1+ )( + )

(HSG Lớp Nam Định năm 2018-2019) Bài 89: Tìm tất cặp số nguyên thỏa mãn: (x 2018− )2 =y 6y 11y 6y4− 3+ 2−

(HSG Lớp Hưng Yên năm 2017-2018) Bài 90:Tìm nghiệm nguyên phương trình y 5y 62 (y 2)x (y 6y 8)x.2− + = − 2+ 2− +

(HSG Lớp Thanh Hóa năm 2017-2018) Bài 91: Tìm cặp số nguyên (x y; ) thỏa mãn: 2x2 +2y2+3x 6y 5xy 7.− = −

(HSG Lớp Hải Dương năm 2016-2017)

(175)

CH

IN

H

P

H

C

K

T

H

I H

C S

IN

H

GI

I C

P H

AI

(HSG Lớp Hưng Yên năm 2016-2017) Bài 93: Tìm số nguyên x, y thoả mãn phương trình (x y x 2y+ )( + )= +x

(HSG Lớp TP HồChíMinh năm 2016-2017) Bài 94: Tìm cặp số nguyên (x y; ) thỏa mãn: x x x 1.( 2+ + =) y−

(HSG Lớp Vĩnh Phúc năm 2015-2016)

Bài 95: Tìm nghiệm nguyên dương phương trình: 3 171 yx+ = 2

(HSG Lớp NghệAn năm 2015-2016) Bài 96: Tìm nghiệm nguyên ( )x; y phương trình: 54x y 3+ =

(HSG Lớp Thanh Hóa năm 2015-2016) Bài 97: Tìm nghiệm nguyên (x; y) phương trình: 5 x( 2+xy y+ 2)=7 x 2y( + )

(HSG Lớp Thanh Hóa năm 2014-2015) Bài 98: Tìm cặp số nguyên ( )x; y thỏa mãn: x x x( + + 2)=4y y ( − )

(HSG Lớp Vĩnh Phúc năm 2014-2015) Bài 99: Tìm số nguyên dương x, y, z thỏa mãn x2 =2x yzz4+

(HSG Lớp Khánh Hòa năm 2014-2015)

Bài 100: Tìm x,y,z N∈ thỏa mãn x 3+ = y+ z

(HSG Lớp Thanh Hóa năm 2012-2013)

Bài 101: Tìm nghiệm nguyên phương trình: 2

2xy + + + =x y x + 2y + xy

(HSG Lớp Bình Định năm 2018-2019) Bài 102: Tìm số nguyên x y, thỏa mãn 4x = +1 3y

(HSG Lớp Quảng Trịnăm 2018-2019) Một số toán từđề thi học sinh giỏi toán lớp 10!

Bài 103 Tìm tất số tự nhiên x, y thỏa mãn phương trình: ( )4

x− y =3361− 11296320

(Đềđề nghịTHPT TP Cao Lãnh – Đồng Tháp)

Bài 104 Tìm nghiệm nguyên phương trình: ( )

2

4x 6y 9x 6y

313

x y

− + −

=

+

(Đềđề nghị THPT Bạc Lưu)

Bài 105 Tìm nghiệm nguyên phương trình: x2+ + =x 2xy y+

(176)

CH

UY

ÊN

Đ

S

H

C

Bài 106 Chứng tỏ số: 444444 303030 3+ không viết dạng (x y 3+ )2 với x,y Z∈

(Đềđề nghịChuyên Quang Trung – Bình Phước) Bài 107 Tìm tất số nguyên dương x, y thỏa mãn phương trình:

( 2 ) ( )

9 x +y +2 3xy 2008+ − =

(Đềđề nghịTHPT Hùng Vương – Lê Lai) Bài 108 Tìm nghiệm nguyên phương trình: x x y xy3+ + 2+y3 =8 x( 2+xy y 1+ + )

(Đềđề nghịChuyên Lương Văn Chánh – Phú Yên)

Bài 109 Tìm nghiệm nguyên phương trình

( )

2

x 17y+ +34xy 51 x y+ + =1740

Bài 110 Tìm tất cặp (x, y, z) số nguyên thỏa mãn hệ phương trình 3

x y z

x y z

 + + =

 + + = 

Bài 111 Tìm tất số nguyên x, y, z thỏa mãn phương trình:

2 2 2

3x +6y +2z +3x y 18x 0.− − =

Bài 112 Tìm tất cả cặp số nguyên dương (a,b) thỏa mãn đẳng thức:

3 2

a −b +3(a −b ) 3(a b) (a 1)(b 1) 25+ − = + + +

Bài 113 Tìm tất cặp số nguyên dương ( )x; y thoả mãn phương trình:

( 2 2 )2 ( 4 4 2 )

x 4y+ +28 =17 x +y 14y+ +49

Một sốbài tốn phương trình nghiệm ngun tạp trí tốn học tuổi trẻ

Bài 114 Tìm nghiệm nguyên phương trình x y xy x y 1.2( − −) = − +

Bài 115 Tìm số nguyên (a.b,c,d) thỏa mãn hệ ac 3bd

ad bc

 − =

 + =

Bài 116 Một tam giác có số đo cạnh số nguyên x, y, z thỏa mãn

2 2

2x +3y +2z −4xy 2xz 20 0.+ − = Chứng minh tam giác tam giác

Bài 117 Chứng minh phương trình sau khơng có nghiệm ngun dương ( ) ( )2

2

x +y = x y+ + xy

Bài 118 Tìm nghiệm tự nhiên phương trình

2 3 2

(177)

CH

IN

H

P

H

C

K

T

H

I H

C S

IN

H

GI

I C

P H

AI

Bài 119 Tìm tất số nguyên x y, thỏa phương trình 2x2+y2 +xy=2(x+y)

(Trích đề học sinh giỏi lớp tỉnh An Giang 2017-2018)

Bài 120 Tìm nghiệm nguyên phương trình 2 x y 2

7 x xy y

=

− +

Bài 121 Tìm số x y, nguyên dương thỏa mãn phương trình: ( 3)

16 xy =15xy+371

(Trích đề học sinh giỏi lớp tỉnh Bến Tre 2017-2018)

Bài 122 Tìm nghiệm nguyên phương trình: (xy)(2x+ + +y 1) (9 y− =1) 13

(Trích đề học sinh giỏi lớp tỉnh Bình Định 2017-2018)

Bài 123 Tìm tất cặp số nguyên (x y, ) thỏa mãn phương trình

2

7 13

x y x xy y

− =

+ +

(Trích đề học sinh giỏi lớp TP Hà Nội 2017-2018)

Bài 124 Tìm số nguyên dương a, b, c, (b>c) thỏa mãn

( )

2 2

2

b c a a b c bc

 + = 

 + + =



(Trích đề học sinh giỏi lớp Hà Tĩnh 2017-2018)

Bài 125 Tìm số thực x cho x+ 2018 2018

x− số nguyên

(Trích đề học sinh giỏi lớp Hải Dương 2017-2018)

Bài 126. Tìm cặp số nguyên ( )x y; thỏa mãn ( )2

2018 11

x− =yy + yy

(Trích đề học sinh giỏi lớp Hưng Yên 2017-2018)

Bài 127.Tìm tất số nguyên dương a b c, , thỏa mãn a b c+ + =91

b =ca

(Trích đề học sinh giỏi lớp Phú Thọ2017-2018)

Bài 128 Chứng minh với số nguyên n cho trước, không tồn số nguyên dương

x cho x x( + =1) (n n+2)

(Trích đề học sinh giỏi lớp huyện Ba Vì 2019-2020)

Bài 129 Tìm số thực x để biểu thức 3

1+ x + 1− x số nguyên

(Trích đề học sinh giỏi lớp quận Ba Đình2016-2017)

Bài 130.Tìm số nguyên dương x, y, z thỏa mãn: 2 2

3x −18y +2z +3y z −18x=27

(Trích đề học sinh giỏi lớp tỉnh Hải Dương 2014-2015)

Bài 131. a) Tìm số nguyên dương x, y thỏa mãn 3 ( 2)

95

(178)

CH

UY

ÊN

Đ

S

H

C

b) Tìm số thực x, y thỏa mãn x2 −4+ y2 −4+ =8 4( x− +1 y−1)

x y

(Trích đềvào 10 Chuyên Sư Phạm 2016-2017)

Bài 132. Tìm tất cặp số nguyên ( )x; y thỏa mãn (x y 3x 2y+ )( + )2 =2x y 1+ −

(Trích đềvào 10 Chuyên Khoa học tự nhiên Hà Nội 2018-2019)

Bài 133. Chứng minh không tồn số nguyên x, y thỏa mãn đẳng thức

2

12x 26xy 15y  4617

(Trích đềvào 10 Chuyên Khoa học tự nhiên Hà Nội 2017-2018)

Bài 134. Tìm tất cặp số nguyên  x y; thỏa mãn đẳng thức sau

2

x + x = y

(Trích đềvào 10 Chuyên Khoa học tự nhiên Hà Nội 2016-2017)

Bài 135. Tìm cặp số nguyên x y;  thỏa mãn: 5x28y220412

(Trích đềvào 10 Chuyên Khoa học tự nhiên Hà Nội 2013-2014)

Bài 136. Tìm tất cặp số nguyên  x y; thỏa mãn đẳng thức

x y 1xy x y   5 2x y 

(Trích đềvào 10 Chuyên Khoa học tự nhiên Hà Nội 2012-2013)

Bài 137. Tìm tất số ngun khơng âm (x, y) thoả mãn đẳng thức (1+x2)(1+y2)+4xy+2(x+y)(1+xy)=25

(Trích đềvào 10 Chuyên Khoa học tự nhiên Hà Nội 2010-2011)

Bài 138. Tìm số nguyên a để phương trình sau có nghiệm ngun:

a) ( )

5 (1)

x − +a x+ a+ =

b)

198 (2)

x +ax+ =a

Bài 139 Tìm số nguyên dương x, y thỏa mãn phương trình: 3

1

x +y + = xy

Bài 140 Tìm nghiệm ngun khơng âm phương trình : ( )4 4 ( )2 2

1 (1)

y+ + y = x+ +x

( Vòng 2,THPT Chuyên – Đại học Sư phạm Hà Nội, năm học 2006 – 2007)

Bài 141 Tìm số nguyên x, y thỏa mãn: ( ) ( 2)

7 x+y =3 xxy+y (1)

Bài 142 Tìm nghiêm nguyên phương trình: 2 ( )

17 34 51 1740

x + y + xy+ x+y =

(Vòng 1, THPT Chuyên - Đại học Quốc gia Hà Nội, năm học 2005 - 2006)

Bài 143 Tìm nghiệm nguyên dương phương trình: ( 2 )

(179)

CH

IN

H

P

H

C

K

T

H

I H

C S

IN

H

GI

I C

P H

AI

Bài 144 Tìm cặp số nguyên( , )x y thỏa mãn điều kiện:(x+ +y 1)(xy+ +x y)= +5 2(x+y)

( Đề thi tuyển sinh vào lớp 10 trường THPT chuyên KHTN – ĐHQG Hà Nội , 2014)

Bài 145 Tìm cặp số nguyên ( , )x y thỏa mãn điều kiện: 4x2+8xy+3y2+2x+ + =y

( Đề thi tuyển sinh vào lớp 10 chuyên toán tin Amsterdam , 2018)

Bài 146 Tìm cặp số nguyên( , )x y thỏa mãn điều kiện:x3−y3 =91

Bài 147 Tìm nghiệm nguyên phương trình:

1

xxy+ = yx

Bài 148 Tìm nghiệm nguyên phương trình: 3 ( )

8 *

xy =xy+

Bài 149 Tìm số nguyên dương x y z, , thỏa mãn: x2− =2 (xy+2)z

Bài 150 Tìm cặp số nguyên ( )x y; thỏa mãn điều kiện: ( ) (2 )

2 26

x+ y− +xy + =

Bài 151 Tìm số nguyên dương ( )x y; thỏa mãn: 3 ( 2)

95

xy = x +y

(Trích Đề tuyển sinh lớp 10 chuyên Toán ĐHSP Hà Nội, năm 2016)

Bài 152 Tìm số nguyên tố x y, thỏa mãn điều kiện: (x2+2)2 =2y4+11y2+x y2 2+9 Bài 153 Tìm số nguyên dương x y, thỏa mãn: x3−y3 =13(x2+y2)

Bài 154 Tìm tất cả cặp số tự nhiên ( )x y; thỏa mãn phương trình:

( )

4

2

16 14 49 16

17

x y y

x y

+ + + =

+ +

Bài 155 Tìm cặp nghiệm số nguyên ( ; )x y thỏa mãn x2−xy+y2 =x y2 −5

(Đề tuyển sinh vào lớp 10 Trường THPT chuyên KHTN – ĐHQG Hà Nội, 2015).

Bài 156 Tìm tất số nguyên dương x y z, , thỏa mãn x+ − =y z 3x2+2y2−z2 =13

(Đề tuyển sinh Chuyên Tin Amsterdam, 2017)

Bài 157 Tìm tất số nguyên x y, thỏa mãn điều kiện:x2(x+y)= y2(xy)2

(Trích Đề tuyển sinh vào lớp 10 Trường THPT chuyên KHTN – ĐHQG Hà Nội, 2016)

Bài 158 Tìm tất cặp số nguyên ( )x y; thỏa mãn 2 2

5

xxy+y =x y

(Đề tuyển sinh lớp 10 Trường THPT chuyên KHKT – ĐHQG Hà Nội, 2015)

Bài 159 Tìm nghiệm nguyên phương trình 2( )

3

x y x+y + + = +x y xy

Bài 160 Tìm nghiệm tự nhiên phương trình:

8x−37= y

Bài 161 : Tìm nghiệm ngun khơng âm phương trình: x+ x+ x+ + x = y

(180)

CH

UY

ÊN

Đ

SỐ

H

C

A. KiÕn thøc cÇn nhí

1 Định nghĩa

Ta biết rằng, số thực x đều biểu diễn dạng: x= +n t với nZ

0≤ <t

Ví dụ: 6, 7= +6 0, ; 6, 7− = − +7 0,

Sự biểu diễn Ta gọi số nguyên n phần nguyên x ; t gọi phần lẻ x. Từđây ta đến định nghĩa

Phần nguyên số thực x số nguyên lớn khơng vượt q x, kí hiệu [ ]x Ta

có [ ]x ≤ <x [ ]x +1

Thí dụ: 21 2; 0;[ 7, 2] 8; 1;

2

 =  = − = −  =

     

   

Phần lẻ của số thực x hiệu x với phần ngun nó, kí hiệu { }x

Ta có { }a = −a [ ]a , 0≤{ }a ≤1

Thí dụ { }2,1 0,1; 1;{ 7, 2} 0,8;

2

 

=  = − =

 

2.Tính chất

1) a∈ ⇔ [ ]a =a { }a =0

2) n∈ n≤ < + ∈ ⇔a n  [ ]a =n

3) { }a ={ }[ ]a =0

4) Nếu n∈ [n+a]= +n [ ]a ;{n+a} { }= a

5) Nếu [n+a]=n n∈ 0≤ ≤a

6) a≥ ⇒b [ ] [ ]ab

7) [ ] [ ] [a + ba b+ ]

Tổng quát [ ] [ ]a1 + a2 + + [ ] [ana1+a2 + + an], 8) [ ] [ ] [aba b− ]

9) { } { } {a + ba b+ } { } { } {; aba b− }

10) Nếu [ ] [ ]a = b a b− <1

11) [ ] [ ]2

a +a+ = a

 

CH

Đ

7 PHẦN NGUYÊN

(181)

CH

IN

H

P

H

C

K

T

H

I H

ỌC

S

IN

H

GI

I C

ẤP

H

AI

12) Nếu n∈* [ ] [ ] [ ]na n a ; a a

n n

    ≥    =    

13) Nếu a số nguyên [ ] [ ]− = −a a ;

Nếu a không số nguyên [ ] [ ]− = −a a −1;

Chứng minh tính chất:

Các tính chất 1) đến 5) chứng minh dễ dàng dựa vào định nghĩa phần nguyên

6) Vì ab nên tồn số c≥0 cho a= +b c Do a=[ ]b +{ }b +c, suy [ ] [ ]a = b +{ }b +c Mà { }b +c≥0 nên [ ] [ ]ab

7) Viết a=[ ]a +{ }a b, =[ ]b +{ }b Khi

[ ] { } [ ] { } [ ] [ ] { } { }

[a b+ =]  a + a + b + b = a + b + a + b 

Mà { } { }a + b ≥0 nên

[a b+ ≥] [ ] [ ]a + b

8) Áp dụng tính chất ta có

[a− +b] [ ] [ba− +b b] [ ]= a nên [ ] [ ] [aba b− ]

{ } { } [ ] [ ] ( ) ([ ] [ ]) [ ] { } { } { } { }

9) a + b = −a a + −b b = a+ba + b ≥ + −a b a+b = a+ba + ba+b

Vậy { } { } {a + ba+b}

{ } { }ab = −a [ ] [ ]a + b − =b (ab)−([ ] [ ]ab )≤(ab)−[ab]={ab}.⇒{ } { } {abab}

Vậy { } { } {abab}

[ ] [ ]

10) a = b suy a−{ }a = −b { }b Khơng giảm tính tổng quát, giả sử ab

Nếu a=b a b− =0;

Nếu a>b từ a b− ={ } { } {aba b− }

Suy a b− = − ≤a b {a b− }<1

Vậy a− <b

11)Đặt { }a =d 0≤ ≤d

• Nếu

d

≤ < [ ] [ ] [ ];

2 2

a a d a d a

 +  = + + = + + =

     

     

[ ]2a =2([ ]a +d)=2[ ] [ ] [ ]a + 2d =2 a Từđó suy điều phải chứng minh

• Nếu 1

2≤ <d [ ] [ ] [ ]

1 1

1;

2 2

a a d a d a

 +  = + + = + + = +

     

     

[ ]2a =2([ ]a +d)=2[ ] [ ] [ ]a + 2d =2 a +1 Suy điều phải chứng minh

12) Ta có [ ]na =n a([ ]+{ }a )=n a[ ]+ n a{ }, mà n a{ }≥0 nên [ ] [ ]nan a

1

a a a a a

n a n

n n n n n

   ≤ < + ⇒  ≤ <  +

(182)

CH

UY

ÊN

Đ

SỐ

H

C

[ ] [ ]a

a a a a

n a n

n n n n n

 

        ⇒  ≤ <  + ⇒ ≤ < +

        

Vậy [ ]a a

n n

   

=

   

 

 

13) Nếu a số nguyên [ ]− = − = −a a [ ]a

Nếu a khơng ngun 0<{ }a <1, nên − < −1 { }a <0, suy −{ }a = −1

Ta có [ ]− = −a  ([ ]a +{ }a ) = − [ ]a + − { }a = −[ ]a −1

B CÁC DẠNG TOÁN THƯỜNG GẶP

Dạng 1: Tìm phần nguyên số biểu thức

* Cơ sở phương pháp:Để tính giá trị biểu thức chứa phần nguyên, ta cần sử dụng

tính chất phần nguyên, kết hợp với kĩ thuật tính tốn khác đặc biệt Phương pháp “kẹp”

Đánh giá số hạng để “kẹp” số cần tính phần nguyên hai số nguyên liên tiếp: Đưa

biểu thức dạng z≤ < +A z 1và kết luận [ ]A =z; * Ví dụ minh họa:

Bài tốn Tìm [ ]x biết:

1.2

x= +

3

1 +

1 + +

) (

1

+

n n

Hướng dẫn giải

Ta cần số nguyên y cho: y< < +x y 1để: [ ]x = y

Để ý 1 1 1 1

2 1

x

n n n

      = −  + − + + − = −

+ +      

Suy 0< < ⇒x [ ]x =0

Bài tốn Tìm phần nguyên số: 6+ 6+ + 6+ (có 100 dấu căn)

(Nâng cao phát triển lớp tập – Vũ Hữu Bình)

Hướng dẫn giải

(183)

CH

IN

H

P

H

C

K

T

H

I H

ỌC

S

IN

H

GI

I C

ẤP

H

AI

Ta có a1 = <3

2 6 3

a = +a < + =

3 6

a = +a < + = …

100 99 3

a = +a < + <

Hiển nhiên a100 > 6>2a100  62 Như 2<a100 <3, [ ]a100 =2 Bài tốn Tính phần nguyên của: A= n n( +1)(n+2)(n+3 ) với n số tự nhiên

Hướng dẫn giải

Ta có: ( )( )( ) ( 2 )( 2 ) ( 2 )2 ( 2 )

1 3 3

A= n n+ n+ n+ = n + n n + n+ = n + n + n + n Để ý rằng:

( 2 )2 ( 2 )2 ( 2 ) ( 2 )2 ( 2 )

3 3 3

n + n < n + n + n + n < n + n + n + n +

Suy 2

3

n + n< <A n + n+ Vậy [ ]A =n2 +3 ,n nN

Bài tốn Tìm [ ]x biết: x= 4n2 + 16n2 +8n+3 với n số tự nhiên

Hướng dẫn giải

Thật ta có: ( )2 2 ( )2

4n+1 <16n +8n+ <3 4n+2

4n 16n 8n 4n

⇒ + < + + < +

[ ]

2 2 2

2

4 4 16 4

2 16 2

2

n n n n n n n n n

n n n n n

x n

⇒ + + < + + + < + + < + + ⇒ + < + + + < +

⇒ = +

Bài tốn Tính tổng sau:

1 24

S =    +   + + + 

(184)

CH

UY

ÊN

Đ

SỐ

H

C

( ) ( ) ( 15 ) ( 16 24 ) S =     +   +  +  + +  +  + +  +  + + 

Theo cách chia nhóm trên, nhóm có ba số, nhóm có năm số, nhóm có bảy số,

nhóm có chín số

Các số thuộc nhóm 1, số thuộc nhóm 2, số thuộc nhóm 3, số thuộc nhóm

Vậy A=1.3 2.4 3.7+ + +4.9=70

Dạng 2: Chứng minh đẳng thức chứa phần nguyên

* Cơ sở phương pháp: Chứng minh hệ thức chứa phần nguyên thực chất coi

chứng minh tính chất phần nguyên Để chứng minh hệ thức chứa phần nguyên ta phải sử dụng tính chất nêu phần lý thuyết, kết hợp với kĩ thuật đại số số học

* Ví dụ minh họa:

Bài tốn Chứng minhrằng với số nguyên dương n ta ln có:

2

2

n n

n

+   +  =        

Hướng dẫn giải

Nếu nchẵn, tức n = 2k 2 [ ]

2 2

k k

k k k k k n

+

  + = + +  = + = =      

     

Nếu n lẻ, tức n = 2k + thì: 2 1 [ 1]

2 2

k k

k k k k k n

+ + +

  +  = + + + = + + = + =      

     

Vậy toán chứng minh

Bài toán Cho n số tự nhiên, chứng minh:

4n 4n

 +  = +     

Hướng dẫn giải

Đặt k = 4n+2 ; m= 4n+1  Ta có: km

Do k = 4n+2 nên k≤ 4n+ ⇒2 k2 ≤4n+2

Giả sử

4

k = n+ , điều vơ lý số phương chia cho khơng thể dư Từ

suy ra: 2

4 4

k < n+ ⇒kn+ ⇒ ≤k n+ ⇒ ≤k  n+ =m

(185)

CH

IN

H

P

H

C

K

T

H

I H

ỌC

S

IN

H

GI

I C

ẤP

H

AI

Bài toán Chứng minh với nlà số ngun dương bất kì, ta có:

2

nn

 +  = − +    

   

Hướng dẫn giải

Đặt ;

2

k = n+  m= n− + 

   

Khi đó: 1 2

1

2 2 4

kn+ < + ⇔ − ≤k k n< + ⇔k k − + ≤ <k n k + +k

nngun dương nên phải có 2

1

k − + ≤ ≤k n k +k

Chứng minh tương tự:

2 2

3 1

1

4 4

mn− + < + ⇔m m − + ≤ − <m n m + + ⇔m m − + ≤ ≤m n m +m Vậy phải có k = m

Dạng 3: Phương trình chứa phần nguyên

1) Phương trình có dạng: f x( )=a (a∈)

* Cơ sở phương pháp: f x( )=a (a∈)⇔ ≤a f x( )< +a

* Ví dụ minh họa:

Bài tốn 1. Giải phương trình [ ]2 [ ]

3 x +5 x − =2

Hướng dẫn giải

Đặt [ ]x = y y, ∈Z Phương trình trở thành: 3y2 +5y− =2

Suy y= −2

y= − (

3

y= − loại yZ )

Do [ ]x = = −y Suy − ≤ < −2 x

Vậy tập nghiệm phương trình [− −2; 1)

Bài tốn 2. Giải phương trình 2 2

5 26

x x

 +  −  + = −

   

Hướng dẫn giải

Ta có: 2

7

x x

 +  = + +    

Do đó: 2

5 26

x x

(186)

CH

UY

ÊN

Đ

SỐ

H

C

( )

2

2

2

2

5 26

5

x x

x x

   

⇔  +  −  + + = −    

⇔  +  −  + + =

Đặt

5 5,

x y y y Z

 + = ⇒ ≥ ∈

  Phương trình trở thành: y2 −9y+ =8

Suy y=8 y=1 (y=1 loại y<5 )

Do

5

x y

 + = =

  Suy 8≤ x2 + < ⇔ ≤5 x2 < ⇔4 3≤ <x

Vậy tập nghiệm phương trình  3; 2)

Bài tốn 3. Giải phương trình 17

2

x x

   + =

       

Hướng dẫn giải

Trước hết ta ước lượng giá trị x

Do [ ]xx nên 17

x x x

≤ + = , suy x≥20, (1)

Do [ ]x ≥ −x nên 17 1

2

x x

x

   

> − +  − = −

    , suy x≤22,8 ( )2 Do x số nguyên nên từ (1) (2) suy x∈{21; 22 }

Thửvào phương trình cho, ta x=21

2) Phương trình có dạng: f x( )=g x( )

* Cơ sở phương pháp: Đặt g x( )=t (t nguyên), biểu diễn f x( )=h t( ) đưa phương

trình h t( )= ⇔ ≤t t h t( )< +t hay 0≤h t( )− <t

Tìm t, sau từ g x( )=t tìm x

* Ví dụ minh họa:

Bài tốn Giải phương trình 5

5

x x

− −

  =

 

 

Hướng dẫn giải

Đặt 5 ( )

7

x

t t

= ∈ 3; 21

5 25

t x t

x= + − = −

Ta có 21 21

25 25

t t

t t t

− −

  = ⇔ ≤ < +

 

(187)

CH IN H P H Ụ C K Ỳ T H I H ỌC S IN H GI Ỏ I C ẤP H AI 20

25 21 25 25

46 46

t t tt

⇔ ≤ − ≤ + ⇔ < ≤

Do t nguyên nên t=0 Suy x=1

Vậy phương trình có nghiệm x=1

Bài tốn Giải phương trình [ ]

9

xx + =

Hướng dẫn giải

Biến đổi phương trình dạng [ ]

9

x

x = +

Đặt ( *)

9

x

t t

+

= ∈ x= 9t−8 (do x>0) Ta có 9t t t 9t t

 − = ⇔ ≤ − < +  

2

2

1

7 13

2

7 13

t t

t t t

t t t ≤ ≤   −   − + ≤   ≤ ⇔ ⇔ − + ≥   +  ≥  

Do t số tự nhiên nên t∈{1; 6; 7;8 } Do x∈{1; 46; 55;8 }

Vậy tập nghiệm phương trình {1; 46; 55;8 }

Bài tốn Giải phương trình

3

xx+ x

  + =

   

   

Hướng dẫn giải

Áp dụng tính chất: [ ] [ ]2 ,

a +a+ = a

  ta có

2 2 1

3 3

xx+ xxx

  +  =  + +  = 

         

         

Nên phương trình cho trở thành

4

3

xx

  =

 

 

Đặt ( )

3

x

t t

− = ∈

 4;

5

t x t

x= + − = + Suy

{ }

4

0 2; 1; 0;1;

5

t t

t t t t

+ +

  = ⇔ ≤ − < ⇔ − < ≤ ⇔ ∈ − −

 

 

(do t nguyên), tương ứng tìm 7; ; ; ;

5 5

x∈ − 

(188)

CH UY ÊN Đ Ề SỐ H Ọ C

3) Phương trình có dạng: f x( )  = g x( )

* Cơ sở phương pháp:

Đặt f x( )  = g x( )=t suy f x( ) ( )−g x <1, dẫn đến a< <x b

Với a< <x b suy ( )

( )

1

2

,

a f x b

a f x b

< < 

< <

 từđó tìm t

Ứng với giá trị t nguyên, giải hệ ( )

( )

f x t

g x t

 =    =     

để tìm x

Tập hợp giá trị x tìm từ hệ nghiệm phương trình

* Ví dụ minh họa:

Bài tốn Giải phương trình 1

3

xx+

  = 

   

   

Hướng dẫn giải

Theo tính chất 10 [ ] [ ]a = b a− <b

Đặt 1 ( )

3 x x t t − +   = = ∈    

     Theo tính chất chứng minh ta có

2 1

1 1 11

3

x x x

x

− − + < ⇔ − < − < ⇔ − < < Khi đó

1

1 0 5

0

2

2

1

3 x x x x   +  +

 < < ≤ ≤     ⇒    − 

−   − < < − ≤ ≤

 

 

   

Suy t∈{0;1; 2;3; 4;5}

Với t=0

2

1

0

2

2 1 3

0

1

3 2

1 1 x x x x x x x

 ≤ < 

 ≤ <

− +  

  = = ⇔ ⇔ ⇔ ≤ <

 

    +

     ≤ < − ≤ <

 

Với t=1

2

7

1

2

2 1

1 2

1 3 2 x x x x x x x

 ≤ < 

 ≤ <

− +  

  = = ⇔ ⇔ ⇔ ≤ <

 

    +

     ≤ <  ≤ <

 

Với t=2

2

7

2

5

2 1

2

1

3 2

3 x x x x x x x

 ≤ < 

 ≤ <

− +  

  = = ⇔ ⇔ ⇔ ≤ <

 

    +

     ≤ <  ≤ <

 

Với t=3

2

11

3

5

2 1 11

3

1

3 2

5 x x x x x x x

 ≤ < 

 ≤ <

− +  

  = = ⇔ ⇔ ⇔ ≤ <

 

    +

     ≤ <  ≤ <

(189)

CH IN H P H Ụ C K Ỳ T H I H ỌC S IN H GI Ỏ I C ẤP H AI

Với t=4

2

13

4

8

2 1

4

1 x x x x x x x

 ≤ < 

 ≤ <

− +  

  = = ⇔ ⇔ ⇔ ≤ <

 

    +

     ≤ <  ≤ <

 

Với t=5

2

19

5

8

2 1 19

5

1

3 2

9 11 x x x x x x x

 ≤ < 

 ≤ <

− +  

  = = ⇔ ⇔ ⇔ ≤ <

 

    +

     ≤ <  ≤ <

 

Vậy tập nghiệm phương trình [0, 5;1)∪[2;3)∪[3, 5;5, 5] [∪ 7;8)∪[9;9, )

Bài toán Giải phương trình [x−2, 3] [= 4−x]

Hướng dẫn giải

Theo tính chất 10 [ ] [ ]a = b a− <b suy ra:

[ 2, 3] [4 ] ( 2, 3) (4 )

1 6, 2, 65 3, 65

x x x x

x x

− = − ⇒ − < − − − <

⇔ − < − < ⇔ < <

Suy 0, 35< −x 2, 3<1, 35 Do [x−2, 3]=0hoặc [x−2, 3]=1

Vì 2, 65< <x 3, 65 nên 0, 35< − <4 x 1, 35 suy [4−x]=0hoặc [4−x]=1

Trường hợp 1: [x−2, 3] [= 4−x]=0

Ta có [4−x]= ⇔ ≤ −0 x 2, 3< ⇒1 2, 3< <x 3,

Kết hợp hai điều kiện ta được: 3< <x 3,

Trường hợp 2: [x−2, 3] [= 4−x]=1

Ta có: [x−2, 3]= ⇔ ≤ −1 x 2, 3< ⇔2 3, 3≤ <x 4, 3;

[4−x]= ⇔ ≤ − < ⇔ < ≤1 x 2 x

Khơng có x thỏa mãn hai điều kiện

Từhai trường hợp ta có nghiệm phương trình 3< <x 3,

4) Phương trình chứa nhiều dấu phần nguyên

* Cơ sở phương pháp:

Sử dụng tính chất phần nguyên, phân tích đa thức thành nhân tử, đặt ẩn phụ

(nếu cần) đểđưa vềphương trình phần ngun

* Ví dụ minh họa:

Bài toán Giải phương trình:

2 x x x    + =        

Hướng dẫn giải

(190)

CH UY ÊN Đ Ề SỐ H Ọ C 6

2 3

5

2 3

x x a r a r

x a r

r r r r

a a r a r

+ +    + = ⇔   + = +                         ⇔ +   + = + ⇔ = − +   +        

Lần lượt cho r 0,1, 2, 3, 4, ta

r

a 1 1 1 1 2

x 5 4 3 2 7

Cách khác:

Ta dễ dàng chứng minh tính chất [ ] [ ] [ ] { } { }

[ ] { } { }

0 1;

1

x y khi x y

x y

x y khi x y

 + ≤ + <

+ = 

+ − ≤ + <



Áp dụng tính chất ta được:

5

2 3

x x x x x

    + = +   =

       

       

5

1

2 3

x x x x x

    + = + − = −

       

       

Vậy x nghiệm phương trình

2

x x

x

   + =

   

   

5 x x   =  

 

5 x x   − =     Tức

5 x x x Z  ≤ − < 

 ∈

( )

5

0 1

6

x x x Z  ≤ − + < 

 ∈

hay − < ≤6 x 0hoặc −12< ≤ −x Vậy −12< ≤x

Do x nguyên nên x −11; 10; 9; 8; 7; 6; 5; 4; 3; 2; 1; 0.− − − − − − − − − −

Thay vào phương trình thử lại, ta được: x= − − − − −7; 5; 4; 3; 2;

Bài toán Giải phương trình 224

1! 2! 3!

x x x

     + + =

           

Hướng dẫn giải

Ta có [ ] 224

4

x x

x +      + =

   

Trước hết ta ước lượng giá trị x

Do [ ]xx nên 224

2

x x

x x

(191)

CH IN H P H Ụ C K Ỳ T H I H ỌC S IN H GI Ỏ I C ẤP H AI

Do [ ]x ≥ −x nên 224 ( 1) 1

2

x x

x     x

> − + − +  − = −

    , suy x≤136, ( )2

Do x số nguyên nên từ (1) (2) suy x∈{135;136 }

Thửvào phương trình cho, ta x=135

Bài toán Giải phương trình [ ] [ ] [ ]x + 2x + 3x + + [2009x]=4036082

Hướng dẫn giải

Nhận xét

[ ]x ≤ < +x [x 1] suy k x[ ]≤kx<k x[ ]+k nên k x[ ] [ ] [ ]≤ kxk x + −k 1(kZ+)

Do thay k =1, 2, , 2009 cộng theo vế ta có

[ ] [ ] [ ] [ ] [ ]

[ ] [ ]

2019045 2009 2019045 2017036

2019045 4036082 2019045 2017036

x x x x x

x x

≤ + + + ≤ +

≤ ≤ +

Lại có 4036082=2019045 2017037.+ Do phương trình vơ nghiệm

Bài tốn Giải phương trình 2

x x x −      − = −        

Hướng dẫn giải

Nếu a số nguyên [ ]− = − = −a a [ ]a

Nếu a khơng ngun 0<{ }a <1, nên − < −1 { }a <0, suy −{ }a = −1

Ta có [ ]− = −a  ([ ]a +{ }a ) = − [ ]a + − { }a = −[ ]a +1

Do đó: 2 2 , 1, x x x x x   − ∈    − =    − − ∉      

• Nếu

x sốnguyên phương trình cho trở thành

2 1

0

3

x x

x

− −

  = ⇔ ≤ < ⇔ ≤ <

 

 

x số nguyên nên x∈{1; 2; }

• Nếu

x khơng sốngun phươngtrình cho trở thành

2 1

1 1

3

x x

x

− −

  = − ⇔ ≤ + < ⇔ − ≤ <

 

 

x không nguyên nên phải loại 1, ( 1; 0) 0;1

x= − x= ⇒ ∈ −x ∪  

 

Vậy tập nghiệm phương trình ( 1; 0) 0;1 {1; 2; }

 

− ∪ ∪

(192)

CH

UY

ÊN

Đ

SỐ

H

C

5) Phương trình dạng hỗi hợp

*Cơ sở phương pháp:

Có phương trình chứa phần nguyên phần dư, phần nguyên với

các phép toán khác (lũy thừa, thức,…) ta xếp chúng vào dạng phương trình hỗn hợp

Giải chúng nói chung khó, cần kết hợp nhiều suy luận kĩ thuật khác nhau, dùng

định nghĩa, chia khoảng, sử dụng tính chất số nguyên [ ]x tính chất 0≤{ }x <1,

các tính chất x nguyên { }x =0hoặc x=[ ]x , phương pháp đại sốnhư

đặt ẩn phụ, biến đổi tương đương hệphương trình,

* Ví dụ minh họa:

Bài tốn Giải phương trình tập sốdương: 2 [ ]2

x x

  =  

Hướng dẫn giải

Xét n≤ < +x n 1hay [ ]x =n, nlà số tự nhiên (có thể 0) Ta có

2 2

2

nx <n + n+ Do   x2 nhận giá trị

2 2

; 1; 2; ;

n n + n + n + n

Nhưng [ ]2 2

x =n nên phương trình cho

[ ]2

2

x x n

  = =

  , tức

2 2

1

n x n

n x n

 ≤ < + 

≤ < +

 hay

2

1

n≤ <x n +

x > nên ta có 0< <x n≤ <x n2 +1,n=1, 2, 3, 4,

Bài tốn Giải phương trình: [ ] { }

2

x x x

  + = +

 

Hướng dẫn giải

Từ giả thiết ta suy { } [ ]

2

x =  x + x − Vế phải số nguyên, mà vế trái 0≤{ }x <1

nên { }x =0 Vậy x số nguyên Do

x số nguyên Suy 2

x x

  =

 

[ ]x =x Phương trình cho trở thành

2

x + − =x

Phương trình có nghiệm x = -2 x =

Bài tốn Tìm số x y z, , thoảmãn cảba phương trình sau

[ ] { } 1,1

xyz ; y [ ] { }zx 2, 2 ; z [ ] { }xy 3,

(193)

CH

IN

H

P

H

C

K

T

H

I H

ỌC

S

IN

H

GI

I C

ẤP

H

AI

Cộng vếcác phương trình cho x   y z 3,

Cộng vếhai phương trình đầu

[ ] { } [ ] { } 3,

x  y zzyx  Suy [ ] { }yx 0 (chú ý [ ] { }zzz )

Do { }x số nguyên, suy { }=0x Vậy [ ]y 0 x [ ]x Từ x [ ] { }yz 1,1 [ ]y 0 suy x { }z 1,1

Do 0{ }z 1 x [ ]x nên x 1, { }z 0,1 Từ y[ ] { }zx 2, { }=0x suy y[ ]z 2, Ta lại có [ ]y 0 nên 0 y 1, y 0, 2,[ ]z 2 Vậy z [ ]+{ }z z 2,1

Dạng 4: Bất phương trình chứa phần nguyên

* Cơ sở phương pháp: Khi giải bất phương trình có chứa dấu phần nguyên, ta thường đặt

biểu thức f x( )=t(t nguyên) để chuyển giải bất phương trình khơng cịn chứa dấu

phần ngun, vận dụng định nghĩa tính chất phần nguyên để tìm nghiệm

của bất phương trình

* Ví dụ minh họa:

Bài tốn Giải bất phương trình [x+ >2]

Hướng dẫn giải

Cách 1. Nhận xét [ ]a >b (b nguyên) a≥ +b

Ta có [x+ >2] x+ ≥2 Do x≥4

Cách 2.Đặt [x+ =2] t (t số ngun) có t>5 Do t∈{6; 7;8; }

Từ [x+ =2] t suy t≤ + < +x t suy t− ≤ < −2 x t 1,t∈{6; 7;8; }

Vậy x≥4 Bất phương trình có vơ số nghiệm x≥4

Bài tốn Giải bất phương trình [ ]2 [ ]

2 x −9 x+ +1 16<0 Hướng dẫn giải

Ta có [x+ =1] [ ]x +1 Biến đổi bất phương trình thành [ ]2 [ ]

(194)

CH

UY

ÊN

Đ

SỐ

H

C

Đặt [ ]x =t (t số ngun) có 2t2− + <9t suy 1< <t 3, mà t nguyên nên

{ }2;3

t

Với t=2 [ ]x =2 suy 2≤ <x

Với t=3 [ ]x =3 suy 3≤ <x

Vậy tập nghiệm bất phương trình [2; )

Bài toán Giải bất phương trình [ ] [ ]2x > x

Hướngdẫn giải

Cách Đặt [ ]x =t (t số nguyên) t≤ < +x t suy 2t≤2x< +2t Do [ ]2x =2t

hoặc 2t+1

• Với [ ]2x =2t 0≤{ }x <0, 2t> ⇔ >t t 0, mà t nguyên nên t số nguyên

dương Dẫn đến x≥1

• Với [ ]2x = +2t 0, 5≤{ }x <1 2t+ > ⇔ > −1 t t 1, mà t nguyên nên t số

nguyên dương Dẫn đến x≥0

Kết hợp với 0, 5≤{ }x <1 dẫn đến x≥0,

Cách 2. Nhận xét [ ] [ ]a > b a>b [ ] [ ]ab

Ta có [ ] [ ]2x > x ⇔2x>x [ ] [ ]2xx ⇔ >x [ ] [ ]2xx Trước hết ta tìm x cho [ ] [ ]2x = x

Đặt [ ] [ ]2x = x =t (t nguyên) ta có

2x− < ⇔x x <1 suy 0< <x nên [ ]x =0

Với t=0 [ ] [ ]x = 2x =0 suy 0≤2x<1 nên 0≤ <x 0,

Vậy nghiệm bất phương trình x≥0,

Bài toán Giải bất phương trình [ ]x { }x < −x

Hướng dẫn giải

Bất phương trình [ ]x { }x < −x 1tương đương với [ ]x { }x <[ ]x +{ }x −1 hay [ ]x ({ }x − <1) { }x − ⇔1 ([ ]x −1)({ }x − <1)

Do { }x − <1 0nên [ ]x >1 hay x≥2

Vậy nghiệm bất phương trình x≥2

Dạng 5: Phần nguyên chứng minh số dạng toán số học

* Cơ sở phương pháp: Phần nguyên ứng dụng nhiều giải toán số

(195)

CH

IN

H

P

H

C

K

T

H

I H

ỌC

S

IN

H

GI

I C

ẤP

H

AI

* Ví dụ minh họa:

Bài toán Cho a>0 số n nguyên dương Chứng minh số số nguyên dương

là bội số n không vượt a a

n      

Hướng dẫn giải

Ta viết a=nq r+ , q số tự nhiên, 0≤ <r n

Rõ ràng bội số n không vượt a n, , ,n qn tổng cộng có q số Mặt khác a q

n   =  

  Từđó suy kết luận toán

Bài toán Số 2012! có tận số 0?

Hướng dẫn giải

Vì 10=2.5 nên để biết 2012! có tận bao nhêu chữ số 0, ta cần phải tính

sốmũ phân tích 2012! thừa số nguyên tố

Theo Ví dụ 1, Sốmũ phân tích 2012! thừa số nguyên tố

2

2012 2012 2012 2012

402 80 16 501

5 5

  +  +  + = + + + =

       

        (Do

5

2012<5 )

Do mũ phân tích 2012! thừa số nguyên tố nhiều 501

Vậy 2012! Có tận 501 chữ số Nhận xét. Nếu

5k ≤ <n 5k+ số chữ số tận bên phải số n!

2

5 5k

n n n

   + + + 

     

     

Bài toán Tìm số tự nhiên k lớn cho (2011!)2012 chia hết cho 2012 k Hướng dẫn giải

Ta có

2012=2 503

Sốmũ cao 503 có 2011! Là

2011 503

  =

 

  (do

2

2011 503< )

Vậy 2011! chia hết cho

503 không chia hết cho

503 , hiển nhiên 2011! chia hết

cho

4 Do 2011! chia hết cho 20123 không chia hết cho 2012

Muốn ( )2012

2011! chia hết cho 2012k k≤3.2012=6036

Vậy maxk =6036

(196)

CH

UY

ÊN

Đ

SỐ

H

C

2010 2011 2012

n n n

  =  = 

     

      (1)

Hướng dẫn giải

Viết n=2010k+r(0≤ ≤r 2009, ,k r có số tự nhiên) Thay vào (1) ta có

2010 2011 2012

2010 2011 2012

k+r k+ −r k k+ −r k

  =  = 

     

     

2

0

2011 2012 2011 2012

r k r k r k r k

k k  −  k  −   −   − 

⇔ = + = + ⇔  = =

       

Suy 0≤ −r 2k nên 2k≤ ≤r 2009, 0≤ ≤k 1004

Vậy n=2010k+r(0≤ ≤k 1004; 2k≤ ≤r 2009)

Do có 105 giá trị k (từ0 đến 1004) Với k r nhận giá trị từ 2k đến

2009 Vậy sô nghiệm tự nhiên n (1)

( )

1004

0

2010 1011030

k

k

=

− =

Bài tốn Tìm tất số nguyên tố x cho

2

1  x 1

  + + + −

      số nguyên tố

Hướng dẫn giải

Nhận xét

( )2

2

1 1 ,

n nnn n

  = + = = + − = ∈

 

      

Đặt 2 2 ( )2 ( ) 2

1 1 2

n

S = n   + n + + +  n+ − = n+ n= n +n

 

Do ( )

2

1

4

1

6 x

x x x

y=    + + + x − = +S S + +S − = − −

Nên ( )

6y=x 4x −3x−1 , suy 6y x , mà x y, số nguyên tố suy x∈{2;3;y}

Nếu x=2 y=3 (thỏa mãn); x=3 y=13 (thỏa mãn); x= y y= −1

hoặc

4

y= (loại)

Vậy toán có hai nghiệm x=2 x=3

Bài tốn Cho a= +2

a) Tính

a

   

b) Tính

a

   

c) * Chứng minh n

a

 

 là số tự nhiên lẻ với số n nguyên dương

(197)

CH IN H P H Ụ C K Ỳ T H I H ỌC S IN H GI Ỏ I C ẤP H AI

Hướng dẫn giải

a) Cách (tính trực tiếp)

( )2

2

2

a = + = +

Ta có 6<4 3<7 nên 13<a2 <14 Vậy   = a2 13 Cách (tính gián tiếp)

Ta có

7

a = + Đặt b= −2 b2 = −7 Suy

2

14

a +b = (1)

Ta có 0< <b nên 0<b2 <1 (2)

Từ (1) (2) suy

13<a <14 Vậy   = a2 13

b) Cách (tính trực tiếp)

( )3

3

2 12 18 3 26 15

a = + = + + + = +

Ta có 25<15 3<26 nên 51<a3 <52 Vậy   = a3 51 Cách (tính gián tiếp)

Ta có

26 15

a = + Đặt b= −2 b3 = −8 12 18 3+ − =26 15 3− Suy

3

52

a +b = (1)

Ta có 0< <b nên 0<b3 <1 (2)

Từ (1) (2) suy

51<a <52 Vậy

51

a

  =  

c) Đặt b= −2 Theo khai triển (x+ y)n, ta

(2 3)

n n

a = + = +A B với A B, số tự nhiên

(2 3)

n n

b = − = −A B

Suy an +bn =2A (3)

Ta có 0< <b nên 0<bn <1 (4)

Từ (3) (4) suy 2A− <1 an <2A Vậy    =an 2A−1, tức   an số lẻ

Chú ý: Trong cách tính gián tiếp, để chứng tỏ n

a

 

  số nguyên m, ta chứng minh

1 n n

a +b = +m 0<bn <1, m<an < +m 1,   = am m Cách khác:

Đặt 2 3 x x  = −   = +

 ,

1 2 , x x x x x x + =  ⇒  =

 nghiệm phương trình

2

4

xx+ =

Đặt

n n n

(198)

CH

UY

ÊN

Đ

SỐ

H

C

( )( )

2

1 1 1

2

2 2 2

4

4

n n n

n n n

x x x x x

x x x x x

+ + + +

− + = ⇒ − + = − + = ⇒ − + =

Cộng (1) (2) ta được: Sn+2 +4Sn+1+Sn =0 ( )3

Ta có S0 =2,S1 =4 nên từ (3) suy Snlà số nguyên chẵn với nN Ta có 0< −2 3<1nên 0<x1n < ⇒1 x2n +(x1n − <1) x2n <x2n +x1nSn − <1 (2+ 3)n <Sn

(2 3)n Sn

 

⇒ + = −

  số lẻ với nN

Dạng 6: Chứng minh bất đẳng thức có chứa phần nguyên

* Cơ sở phương pháp: Để chứng minh bất đẳng thức phần nguyên ta phải sử dụng

linh hoạt tính chất nêu phần lý thuyết * Ví dụ minh họa:

Bài toán 1. Chứng minh

[ ] [ ] [x + yx+y]

(Nâng cao phát triển lớp tập – Vũ Hữu Bình)

Hướng dẫn giải

Cách Ta có [ ]xx; [ ]yy nên [ ] [ ]x + y ≤ +x y

Suy [ ] [ ]x + y sốnguyên không vượt x+ y (1)

Theo định nghĩa phần nguyên, [x+ y]là số nguyên lớn không vượt x+y ( )2

Từ (1) (2) suy [ ] [ ] [x + yx+ y]

Cách 2.Theo định nghĩa phần nguyên, ta có

[ ] [ ]

0

0

x x

y y

≤ − < ≤ − <

Suy 0≤(x+y)−([ ] [ ]x + y )<2

Xét hai trường hợp:

- Nếu 0≤(x+ y)−([ ] [ ]x + y )<21

[ ] [ ] [x + y = x+ y] (1)

- Nếu 1≤(x+ y)−([ ] [ ]x + y )<2 0≤(x+y)−([ ] [ ]x + y + <1) nên [x+y] [ ] [ ]= x + y +1 (2)

(199)

CH

IN

H

P

H

C

K

T

H

I H

ỌC

S

IN

H

GI

I C

ẤP

H

AI

Bài toán 1. Cho x y, ∈R Chứng minh

[ ] [ ] [ ] [ ] [2x + 2yx + y + +x y]

(Chuyên đề bồi dưỡng học sinh giỏi toán THCS Số học – Nguyễn Vũ Thanh)

Hướng dẫn giải

Ta có: [ ]2x =2[ ]x +2{ }x =2[ ]x + 2{ }x ;

[ ] [ ] { } [ ] { }

[ ] [ ] [ ] { } { }

2y y y y y ;

x y x y x y

 

= + = +   + = + + + 

Bất đẳng thức cần chứng minh tương đương với:

2{ }x   + 2{ }y   ≥ { } { }x + y  ( )1

Vì 0≤{ } { }x + y <2 nên ta có hai trường hợp sau:

● Nếu 0≤{ } { }x + y <1 (1) ln đúng vế trai lớn 0, vế phải ● Nếu 1≤{ } { }x + y <2 { } { }x + y =1khi { }

2

x ≥ { }

2

y ≥ Giả sử

{ } { } { } { }

2 2

2

x ≥ ⇒ x ≥ ⇒ x   + y ≥ (đpcm)

C BÀI TẬP ÁP DỤNG

Bài 1: Tìm [ ]x biết:

3

x− < − <x Bài 2: Tìm [ ]x biết : x< − < +5 x 0,5 Bài 3: Tìm [ ]x biết:

6

1 1

1

2 10

x= + + + +

Bài 4: Tìm phần nguyên biểu thức :

2+ + + 2+ , (với n dấu căn)

Bài : Tìm phần nguyên biểu thức :

3

6

6+ + + (với n dấu căn)

Bài 6:Tính tổng sau:

S = 1.2.3.4  + 2.3.4.5  + 3.4.5.6+ +  n n( +1)(n+2)(n+3)

Bài 7: Chứng minh rằng, với số nguyên n ta có:

[n+x]= +n [ ]x

(200)

CH UY ÊN Đ Ề SỐ H Ọ C

[ ]x + [ ]y ≤ [x+y] ≤ [ ]x + [ ]y +

Bài 9: Cho n số nguyên dương, chứng minh:

    n +     +

n = n

Bài 10: Cho n số tự nhiên, chứng minh:  4n+1  = 4n+2

Bài 11:Cho n số tự nhiên, chứng minh:  n+ n+1  = 4n+2

Bài 12:Chứng minh rằng: [ ]x +  + 

2

x = [ ]2x , (với xlà số thực bất kỳ)

Bài 13: Tính tổng: S =  + 

2 n +     + 2 n +     + n +     +

n + + 2020 n+      

Bài 14:Chứng minh rằng: m[ ]x ≤ [ ]mx ≤ m[ ]x + m - (với giá trị m nguyên dương)

Bài 15: Chứng minh : Không tồn x thoả mãn:

[ ]x + [ ]2x + [ ]3x + + [100x] = 313096

Bài 16:Giải phương trình:[x+0,7] = -

Bài 17: Giải phương trình: [ ]x+1 + [x+2] + [x+3] =

Bài 18: Giải phương trình 4[ ]x = 3x

Bài 19:Giải phương trình:  + 

8

5 x =

5 15x

Bài 20:Giải phương trình:  − 

3 2x +  +  4x = 5xBài 21: Giải phương trình: [ ]x { }x = x -

Bài 22:Giải phương trình: x - 3 

2

x = 2

Bài 23: Giải phương trình: [ ]x−1 =  +1

x

Bài 24:Giải phương trình: x4= 2x2 + [ ]

x

Bài 25: Giải phương trình: x3 - [ ]

x =

Bài 26: Giải phương trình: 2

3

2

x xx

− +  = +    

Bài 27: Với k > 3, Chứng minh 2n n n

k k k

+     ≥ +             

Bài 28: Cho *

1 , , , , n

n k k kN Chứng minh rằng:

( )

1

1

n

n

k k k

n k k k

n

+ + +

  + − ≤ + + +  

Ngày đăng: 24/02/2021, 07:00

Từ khóa liên quan

Tài liệu cùng người dùng

  • Đang cập nhật ...

Tài liệu liên quan